You are on page 1of 422

VALE UNsVERS

School Mathematics Study Group

Intermediate Mathematics

Unit 18
Intermediate Mathematics
Sti4dent's Text, Pdrt II

Prepared under the supcrvjsion of


the Panel on Sample Textbooks
of the School Mathematics Study Group:

Frank B. Allen Lyons Township a g h School


Edwin C.Douglas Taft School
Donald E.Richmond Wilham College
Charles E. Rickart Yale University
Henry Swain New Trier Townsllip k g h School
Robert J. Walker Cornell University

New Haven and London, Yale University Press

UNIVERSITY OF CHlCAQO
LAEORATORY SCHOOL
SCHOOL UBRAR'I
Copyright o 1960,1961 by Yale University.
Printed in the United States of America.

All rights reserved. This book rnav not


be rcproduced, in whole or in parc, in
any form, without wrirten permission from
the publishers.

support for the School Mathematics


Fin~t~cial
Study Group has been provided by the National
Science Fo undacion.
Below are l i s t e d the names of a l l those who participated In any of t h e w r i t i n g s e s a l o n a
a t which the f o l l o w i n g SMSO texts were prepared: First Course i n Algebra. Geometry,
Intermediate Mathematics, Elementary Functions, and Introduction to Matrix U ~ e b r a .

H.W. klexander, Earlham College R . C . Jurgensen, Culver Military Academy,


F . B . Allen, Lyons Township High School, La Culver, Indiana
Orange, I l l i n o i a Joseph Lehner, Wohlgan State University
Alexander Beck, Olney High School, Phila- Marguerite Lehr, Bryn Mawr College
delphia, Pennsylvania Kenneth Leisenring, U n i v e r s i t y of Michigan
E.F. Beckenbach, University of C a l l i o r n i a Howard Levi, Columbia University
.
a t Lon Angeles
E .G Begle, School Mathematlca Study Group,
Yale University
Eunice Lewle, Laboratory High School,
Univeralty of Oklahoma
M.A. L i n t o n , William Penn C h a r t e r S c h o o l ,
Paul Berg, Stanford Universlty Philadelphia, Pennsylvania
hi1 Berger, Monroe High School, S t . P a u l , A.E. Livingston, University of Waahington
Minnesota L .H. Loomis, Harvard Unlverslty
Arthur Bernhart, University of Oklahoma R .V. Lynch, Phillips Exeter Academy,
R.H. Blng, University of Wisconaln Exeter, New Hampshire
A.L. Blakers, University of Western W .K . McNabb, Hockaday School, D a l l a s ,
Austral l a Texas
A . A . Blank, New York University X .G . Mchael s , North Haven High School,
S h i r l e y Boselly, A a n k l i n H i g h School, North Haven, Connecticut
Seattle , Washington E .E . Molae, University of Michigan
K .E. Brown, DepaPtment of Health, Educa- E.P. Northrop, University of Chicago
t i o n , and Welfare, Washington, D.C. 0 .J. P e t e r s o n , Kansas S t a t e Teachers
J.M. Calloway, C a r l e t o n College College, Emporia, Kansas
Hope Chipman, University H l g h School, Ann B . J . P e t t i s , Unlversity of North Carolina
ArSor, Michigan R . S . Pieters, Phillips Academy, Andover,
.
R .R C h r i s t i a n , University of B r i t i s h Massachusetts
Columbia H.O. P o l l a k , B e l l Telephone Laborntorles
R . J . Clark, S t . P a u l ' s School, Concord, Walter Prenowltz, Brooklyn College
New Hampshire O.B. P r i c e . U n i v e r s i t y of Kansas
P . H . Daua, University of California at Los A.L. Putnara, University of Chicago
Angeles Persts 0 . Redgrave, Norwich Free Academy,
R.B. Davis, Syracuse University NolWich, C O M ~ C ~ ~ C U ~
Charles k P r i m a , C a l i f o r n i a Institute of Mina Rees, Hunter College
Technology D . E . Richmond, Williams College
Wry D o l c i a n i , Hunter College C .E. Rlckart, Yale University
Edwin C . Douglas, me Taft School, Water- Harry Rudeman, Hunter College High School,
t o m , Connecticut New Y@rk C i t y
Floyd h w n e , East High School, Denver, J.T. Schwartz, New York U n i v e r a i t y
Colorado O.E. Stanaltia, S t . Olaf College
E.A. Dudley, North Haven High School, North R o b e r t Starkey, Cubberley H l g h Schools,
Haven, Connecticut Palo Alto, C a l i f o r n i a
L i n c o l n Curst, The Rice Institute Phillip Stucky, Roosevelt High School,
Florence E l d e r , Weat Hempstead Hlgh School, S e a t t l e , Washington
Weat Hempstead, Me* York Henry Swain, N e w Trier Township H f g h
W .Em Ferguson, Newton HZ@ School, Newton- School, Winnetka, Illinois
v i l l e , Maasachusetts Henry Syer, Kent School, Kent, Connecticut
N.J. Fine, University of Pennsylvania G.B. Thomas, Mamachusetts Institute of
Joyce D. Fontalne, North Haven High School. Technology
North Haven, Connecticut A.W. Tucker, P r i n c e t o n University
F.L. Friedman, Haasachuaetts I n s t i t u t e oi H.E. Vaughan, University of Illlnois
Technolorn John Wagner, U n i v e r s i t y of Texas
Esther Oassett, Claremore High School, R.J. Walker, Cornell Univeroity
Claremore, Oklahoma A.D. Wallace, Tulane U n i v e r s i t y
R.K. aetoor, University o f Washington E .L. Halters, W i l l i a m Penn Senior High
V.H. M a g , F r a n k l i n md Marahall College School, York, Pelmsylvania
R .R. Hartman, Edina-Morningaide Senior High Warren mite, North High School, Sheboygan,
School , Edina , Mime ~ oa t Uisconaln
M.H. Heina, University of I l l i n o i a D.V. Widder, Harvard University
Edwin H e w i t t , University of Waahington W i l l i a m Wooton, Pierce J u n i o r College,
Martha Hildebrandt , Proviao Tomship High Woodland hill^, C a l i f o r n i a
School, Maywood, Illinois J.H. Zant, Oklahoma S t a t e Unlversity
-.- - , ' . " ' , , ' , , ' , , " ' , " ' " . ' , ,

0.70 0.80 0.90 1 .oo


1
Figure 9-lg. Graph of y = z.
1
Figure 9 - l h . Graph of y = x.
It can be shown by counting squares t h a t In 10 l a approxi-
mately 2 . 3 (a five-place t a b l e g i v e s this value as 2.302591,
Experience has shown t h a t one of the most useful logarithms I s t h e
common logarithm loglOx; i t s value is 1 at x = 10. To show
how p o i n t s needed to draw t h e graph o f y = loglox can be obtained
let us consider once more t h e hyperbola y = k/x. (see Fig. 9-lj) .
Every o ~ d i n a t eon t h e graph of t h i s equation ia k times the
1
corresponding ordinate on t h e graph of y = 3;. ( ~ o t i c ethat the
h e i g h t at each x for y = In x is approximately 2.3 times
the h e i g h t of the ordlnate f o r the graph y = loglOx) .

Figure 9-lj

Therefore, for any interval 1 to x (x > 0), the area


under the graph of y = k is k times the corresponding area

under the grapn of y = z. In F i g . 9-1j t h i s means that the area


under arc PIQ1 l a k times t h e area under are PQ.
But, by o u r definition, these areas are log x and ln x respec-
t i v e l y , therefore we can write
9-1 log x = k In x,
where k is the constant in the equateon of the hyperbola used in
definlng log x , Clearly, the value of l o g x depends on k. To
f i n d an approximate value of k which makes log 10 equal to one,
we substitute x = 10 in (9-1) and make use of the fact that
In 10 is approximately 2.3 as shown in Fig. 9-11;

Of course,
1 is a b e t t e r approximation because it is based
on the more accurate value of In 10 given above. The exact
value is

This number is denoted by M; it I s an Irrational number (siml-


lar t o T ) . Its value, correct to 20 decimal places, is

The common logarithm of x, denoted by loglOx, is thus the


area under the hyperbola

from 1 to x. The values of loglox c a n be computed in the


same way that the values of In x were computed. Fig, 9-lk con-
tains the graph of y = M/x from x = 1.00 to x = 1.35. Areas
under this curve c a n be computed by counting squares. The graph
of y = loglox is shown in F i g . 9-li, and Table 9-1 contains a
brief t a b l e of values of loglOx. It follows from the definition
of common logarithms t h a t
As a matter of notation, y = log x will be used to denote
the general logarithm function obtained from the hyperbola
y = k/x, where the value of k is general and unspecified except
that k > 0.

Figure 9-lk. Qraph of -


y = M
x
464

Table 9-1. A Brief Table of Common Logarfthms.


Exercises 9-1
1. Use the graphs in Figs, 9-lg and 9-lh to estimate the value
of In x for those values of x lieted In the first column
of the following t a b l e . Compare your estimated values w i t h
the correct values given in t h e l a s t column. It should be
observed that these logarithms are natural logarithms rather
than the common logarithms given in Table 9-1.

x Estimated Ln x Correct ln x
0.70 -0.35667
0.82 -0.19845
O. go -0.10536
1.12 0 11333
1.18 0,16551
1.23 0.20701
1.24 0.21511
1.26 2.23111
1.28 0.24686
1.29 0,25464
1-31 0.27003

1.32 0.27763
;.33 o. 28518
1.34 0.29267
2. Use the graph in Fig. 9-lk to estimate the values of logl#
for those values of x l i s t e d in the first column of the
following table. Compare your estimated values with the
correct values given in the last column,

x Estlmted logL# Correct loglox

1.12 0.0492
1.16 0.0645
1.18 0.0719
1.21 0,0828
1.23 0.0899
1.24 0.0934
1.26 0.1004
1.28 0.1072

1.29 0,1106
1.31 0.1173
1.32 0.1206

1 33 0.1239
1.34 0.1271

3 Draw an accurate graph of the comon logarithm function


y = logl# on a large sheet of graph paper. Use Table 9-1
a8 the table of valuea fop drawing the graph. Campare y o u r
graph with the graph of y = logl# in Fig. 9-li.
4. Use sheets of graph paper similar to those In Fig. 9-lg and
9-lh t o extend the graph of y = l/x both to the right and
t o the left. The c l a s s might undertake a cooperative proJect
of drawing the graph from x = 0.1 to x = 10, This graph
can be used to make a table of logarithms for a l l numbers
from x = 0.1 to x = 10. Observe that the logarithms ob-
tained are natural logarithms and not common logarithms as
given in Table 9-1.
5. Use sheets of graph paper similar to those In F i g , 9 - l g and
9-UI to extend the graph of y = M/x in F i g . 9-lk both to
the right and to the left. The c l a s s might undertake as a
cooperative p r o j e c t the task of drawing the graph from
x = 0.65 to x = 10. This graph can be used to make a table
of common logarithms f o r all numbera from x = 0.65 to
x = 10. Compare the values of logL# obtained with the
values given in Table 9-1.
6. If In x is t h e natural logarithm o f x, then M ln x
Show that thls r e l a t i o n can be used to compute a
-
logl#.
table of common logarithms from a table of natural logarithms.
7. DeteMne k In the equation of the hyperbola y =, k so that
log 2 = 1, We call thls log function log2x. Find t h e

value of and

[aec. 9-11
9-2. - -
A n Important Formula For a -
x. The purpose of this s e c t i o n
is to prove a theorem which s t a t e s an important p r o p e r t y of log x,
Theorem . If y = l o g x is the logarithm f u n c t i o n de-
rived from the hyperbola y = k/x, and if a and b are any two
positive numbers, then
9-2a log ab = l o g a 4- log b.
Before we undertake to prove Theorem 9-2 Let us v e r i f y Equa-
tion 9-2a in a number of special cases. Table 9-2a gives t k
values of loglOab and (loglOa + loglob) f o r a number of dlff-
erent values of a and b. In three cases the two numbers d i f f e r
by one in the f o u r t h decimal place. A small difference of this
size i s to be expected occasionally s i n c e t h e logarithms in our
table are c o r r e c t t o only f o u r decimal places.

Table 9-2a. Comparison of logl@ and loglOa + loglob.


F i g u r e 9-2a

Fig. 9-2a. The area under the hyperbola y = k/x from x = a to


x = ab is equal t o t h e area under the hyperbola from x = 1 to
x = b.

Consider the proof of Equatlon 9-2a. The graph on the left in


F i g . 9-2a shows that the area under the hyperbola from x = 1 t o
x = ab is l o g ab, and t h a t thfs area is equal to t h e area from
x = 1 to x = a plus t h e area from x = a t o x = ab. Since
the area from x = 1 to x = a is log a by definition, the
proof of Equation 9-2a w i l l be complete Zf we can show t h a t the
area under the hyperbola from x = a to x = ab is t h e same as
the area from x = 1 to x = b. This fact w i l l be proved in a
s p e c i a l case; the proof in the general case can be given in the
same way.
The proof will be given f o r a = 2 and b = 3 . In this case
we are asked to prove that the area under the hyperbola y = k/x
from x = 2 to x = 6 is equal to the area from x = 1 t o
x = 3 . Approximate the l a t t e r area by f o u r rectangles as shown in
Flgure 9-2b. Approximate Figure 9 - 2 c . Approximate area
k
area under t,he h y p e r b o l a under the hyperbola y = - X
y = -
k from x = 2 t o x = 6 .
X
from x = 1 to x = 3.

Flg. and approximate t h e former area by four corresponding


9-Zc,
rectangles as shown in Fig, 9-2b. The altitude o f each rectangle
can be found by calculating y from y = k/x f o r the appropriate
value of x , The c a l c u l a t i o n s are shown in T a b l e 9-2b. Observe
that the area of each rectangle in Fig. 9-2b is exactly equal

Table 9-2b. Computation of the Areas in Figs. 9-2b and 9-2~.

~ i g . 9-2b FIE. 9-2~

Rec- Length *Ititude Area Length Altitude Area


tangle of Base tangle o f Base
k k
1 1 F 1 0.5 T3
2 1 k k 2 0*5 -
k k
4 7 1.5 5
3 1 $ 3 0.5 -k
2.0

k k k
4 1 4 0 5 =5 5
7
[ s e c . 9-21
to the area of the correapondlng rectangle in Fig. 9-2c. Thus the
sum of the areas of the rectangles in Fig, 9-2b is equal to the
sum of the areas in Fig. 9-2c. The same result will be found re-
gardless of the number of rectangles used to approximate the areas.
If a large number of rectangles is u s e d , the sum of their areas ie
very close to the area under the curve. From these considerations
it follows t h a t the area under the hyperbola y = k/x from x = E
to x = 6 is equal to t h e area from x = 1 t o x = 3 .
A proof of Equation 9-2a for-the general case can be given in
e x a c t l y the same way.
Equation 9-2a has m a n y applications. For example, Table 9-1
does not give 1ogl028, but it doe3 give log104 and log107.
Therefore, by Equation 9-2a,

2 1.4472.
Observe also that 11 = ( f i l )( a) .
Therefore,

so that

log,, JZT = g1 logloll'


2 g (1.0414)
Exercises 9-2.
1. Verify Equation 9-2a in a number of speclal cases by complet-
i n g the following t a b l e , Use t h e common logarithms given in
Table 9-1.

2. Use Equation 9-2a and Table 9-1 to calculate the values of


the f o l l o w i n g l o g a r i t h m s :

(a) log1021 (g) logld2 (m) log1044 (s) 10g1057


(b) log1024 (h) 10gl$3 (n) log1048 (t) loglo63
(c) log1022 (1) logl$4 (0) 1 0 8 ~ ~ 4 9 (u) log10125
(d) log1026 (J) logl$6 (P) 10g1051 (v) log10144
(e) log1027 (b) logl@ (4) log1054 (w) log10250
(f) log1028 (1) log1042 (r) log& (x) 10g~~1000.
j. Prove t h a t log a* =2 l o g a. Use this f a c t to compute the
following logarithms:

4, Prove that l o g abc = l o g a + log b + log c and thus that


2
log a b = 2 l o g a + log b and

l o g a3 = 3 log a.
Use these facts to compute the f o l l o w i n g l o g a r i t h m s :

( a ) log1042 (f log10147 ("1 loglo p


(b) 10glOIOOl (9) 10g10126*?5 (1) loglO

(c) l0glO255 (h) logl$43 (m) loglO

(dl l0g~~26.25 (1) l0gl01728 (n) loglO ViG


(el l ~ g ~ ~ ( 3 * 5 ) ~ (f Jl ) loglo p (0) loglO 3-
5. Use t h e definition of log x as an area to show t h a t

k X-1 < log x


( < k(x-l) where k > 0 and x > 1.
X

Is this inequality true when 0 <x < I?


9-3. Properties Of l o g 5,
Corresponding t o the hyperbola y = k/x t h e r e is a logarithm
function y = l o g x . This f u n c t i o n was defined in Section 9-1,
According to D e f i n i t i o n 9-1, l o g x for each k > 0 has the
following p r o p e r t i e s :
9-3a l o g 1 = 0,
log x > 0, x > 1,
log x < 0, 0 < x < 1.
F!urthemnore, it was shown in Sectfon 9-2 that

9-3b l0g(x1 xg) = l o g x 1 + l o g xg'*


In this section some additional p r o p e r t i e s of the logarithm
function will be established.
In Equation 9-3b let xl be x, and l e t x2 be z.
1 Then

l o g x(,)1 = log x + 1
l o g ji.

But since x (1 ~ )= 1, and l o g 1 = 0 by Equation 9-Ja, the l a s t


equation becomes
0 = log x + log z.
1

Thus it follows that

9-3c -
l o g X1 = - l o g x.

Next, consider the logarithm of .X2 This quotient can be

thought of as a product. Thus,

Then by Equations 9-3b and 9-3c,

X1 l o g X1 + l o g -,
1
lo% = X2
or
X,
lo&- J.
= log XI - l o g x2.
2
It w i l l be shown next that if n is any positive integer,
then,

9-3e log xn = n log x,

log ,-
I

X
= n l o g x.

The first statement in Equation 9-3e f o l l o w s from repeated


a p p l i c a t i o n of Equation 9-3b, f o r
log x 2 = log(x x)
= log x + log x
= 2 log x;
log XJ = log(x2 x)
= log x2 + log x
= 2 log x + l o g x
= 3 log x.
The f i r s t statement in Equation 9-3e can be established by
continuing in this fashion. The second statement follows from t h e
first statement and from Equation 9-3c, f o r
log -
1 = - log xn
xn
= - n log x.

The symbol q~ where x > 0 and q is a p o a l t i v e integar


means a positive number whose qth power is x. Thus
( G
' )' = x. For example, 3J8=2, = 2, 6a
4a = 2,

3m3= 5 . It w i l l now be shown that if p and q are any pos-


itive integers, then
It w i l l be shown f i r s t t h a t log 4
1 l o g x.
! = (a) By the firat
statement in Equatlon 9-3e,
l o g ( q ~ j t ) q= q l o g 4
-
,
or
log x = q l o g qfl.
By solving this equation f o r log q~ we obtain

log %= $ l o g x.
From this r e s u l t and the f i r s t statement in Equation 9-3e it
follows that
log ( g f i ) p = ;log x,
and the proof of Equation 9-3f is Complete.

The next property of l o g x to be established i s the


f ollow~ng:

9-3g If x1 < x2, then log xI< l o g x2.


This property f o l l o w s from the definition of l o g x (~efinltion
k
9 ; f o r , if xl < x,, the area under the hyperbola y = ji
from 1 to xl is less than the area under the curve from 1 to

A similar argwnent establishes

9-3g ' If l o g xl < log x2, then xl < x2.


Note that statements 9-3g and 9-3g' can be expressed as one
statement as follows:

X1< X2
if and only If log xl < log x2
The next p r o p e r t y of log x to be established is the follow-
ing :
9-3h If l o g xl = log x2, then x, = x2,

There are only three possibilities: either xy < x2, xl > x2,

or x3. = x2. But the f i r s t two are impossible, since xl( x2


implies log xl ( l o g xg, and xl > x2 implies l o g xl > l o g x:,
by Equation 9-jg. Thus, xl < x2 and xl > x2 must be rejected
since both l e a d to contradictions. Therefore, xX = x2, and the
proof Is complete.
Again, a similar argument establishes

9-3h ' If xI = x2, then l o g xl = l o g x2

The f i n a l property of l o g x which is desired is the following:


9-31 The graph of y = log x is a contfnuous curve.
This f o l l o w s from t h e f a c t that the graph has no breaks or Jumps
in it. An important consequence of this property is the following:
I f x1 < x2 a d c is any number such t h a t l o g xl < c < l o g x2,
t h e n there is a number xo such that xl < xo < x2 and
log XO = C*

The following is a summary o f the properties of log x:


9-3a l o g 1 = 0,
log x > 0 x > 1,
log x < 0 O<x<l.
If x1 and x2 are any two positlve numbers, then

log(xl*x2) = l o g Xl + l o g xp.
1
log -
X
= - log x.
If x1 and x2 are any two p o s i t i v e numbers, then

log -
X1
x2
= log XI - l o g x2.

If n is any p o s i t i v e i n t e g e r , t h e n

log xn = n l o g x,

log -nI = -n l o g x.
X

If p and q are any two p o s i t i v e integers, then

log (q 3 ) P = E
q
log x.

~f xl ( x2, then log xl < l o g x2.

If l o g x1 < l o g x2, then xl ( x2

[x, < x2 If and o n l y if l o g xl < log x d

If l o g xl = l o g x2, then xl = x2.

If x1 = x2, then l o g xl = l o g x2

t l = X2 if and only if l o g xl = l o g x2]

9-31 The graph o f y = log x is a c o n t i n u o u s curve.

Some appli cations of t h e s e properties of the f u n c t i o n log x


will be i l l u s t r a t e d by examples:
Example 9-3a. Use Table 9-1 to f i n d t h e f o l l o w i n g
logarithms:
(b) According to (9-3f),

2 $(0.6990)
2 1.1650
&ample 9-Jb, If N = a3b
-, express log N in terms of the
6
logarithms of a, b, and c.

solution: log Qb- = l o g A - log f i


F
- log a3 + l o g b - l o g f i
= 3 log a + log b - p1 l o g c

Exmple 3 Solve for x: 1 l o g x I- log 3 = log 5

Solution: log x I- 3 log 3 = 3 log 5


log x = 3 l o g 5 - 3 log 3
log x = l a g 53
= - log33
1% x = log 5
3
Example - 3 . Solve f o r x: l o g (x-3) + l o g x = l o g 28
Solution: Note t h a t l o g a is not defined when a 5 0. This
means that we must have ( x - 3 ) > 0 for our equation t o be mean-
i n g ~ u l . We can write log[(x-3)xl = log 28. Since x I = x2 if l o g

x1 = log xg, we have (x-j)x = 28 or x2 - jx - 28 = 0. The mots


of this quadratic are found to be 7 and -4. We ob-
serve that 7 satisfies the o r i g i n a l equation and t h a t -4 must
be rejected f o r t h e reason we have Indicated.

mercises 9-3.
1. Use T a b l e 9-1 and the propertfes of log x stated in t h i s
section to f i n d the following logarithms:
2. Find the value of 2 x 7 by t h e use of logarithms.
Solution: By Equation 9-3b and Table 9-1,
loglo( 2 x 7 1 = logl02 + log 107

2 0.~010+ 0.8451
2 1.1461.
Table 9-1 shows that
14 2 1.1461.
log10
It follows from Equation 9-3h that 2 x 7 = 14.

3. Find the value of


2. o x 18.00
by using logarithms.

4. If N =
1 x 8 ,
5 find N by means of logarithms.
3

5. &press the logarithms of each of the f o l l o w i n g expressions


in terms of the logarithms of the l e t t e r s involved as in
Example 2:
6. Solve each of the following logarithndc equations for x:
(a) loglO x = 3 loglO 7

(b) loglO x + loglO 13 = loglO 182

(c) 2 loglO X - loglO 7 = loglO 112

(d) log, (x-2) + loglO 5 = 2

(e) loglO x + loglO (x + 3 ) = 1

(f) 21 loglO x = - loglO 64

(g) loglO (x-2) + loglO (x + 3 ) = logl0 14

7. Write without " log" :

(a) loglO V = loglO 4 + logl0 w t 3 loglO r - loglO 3


1 1
(b) loglO P = loglO t + 7 loglO g

(c)
1
loglO s = - [logl0 + loglO (5-a)

8. S h o w b y logarithms that, if a > 0, and p, q, and n are


natural numbers, then

(b) nqn
qfl = Hint: Use Property 9-3f

[ s e c . 9-31
9. Express as a single logarithm:
( a ) loglO x + loglO Y - loglO z

(b) loglO (x + 3) - logl0 (x-2)

(c) 4 loglO t - 3 loglO s

(f) loglO X + 4 logl0 (x-2) + 31 loglO X2

'10, Suppose we denote the area under the . h e c u r v e y = 3x2 in t h e


first quadrant between the ordinates at 1. and x as "lug xl'
Are there any properties of log x which are a l s o true f o r
II
lug x"? In particular is it true t h a t lug ab = lug a +
l u g b?

9-4. -
The Graph -
of = log&.
Fig. 9-11 contains graphs of y = ln x and y = logl+

These graphs exhibit many of the characteristic features and im-


portant properties of all logarithm functions. This section w i l l
be devoted to a study of the properties of the graph of the general
logarithm function y = log x.
The first important property of the graph of y = log x is
this: The ordinate y always increases as x increases. It waa
proved in 9-3g that, if xl < x 2 , then log xl < log x2. The fact
t h a t y always increases as x increases on the graph of
y = l o g x is a consequence.
It f o l l o w s from the definition of l o g x t h a t log 1 = 0 (see
also Equation 9-3a). Thus the graph of y = log x crosses the
x-axis at 0 The graph does not cross the x-axis at any other
p o i n t because y always Increases as x increaaes.
It has been explained already t h a t the graph of y = log x
is a continuous c u r v e ( s e e P r o p e r t y 9-31), The graphs of
y = In x and y = log
lox in Fig. 9-11 are continuous curves, and
the graph of every logarithm function y = log x has this same
property.
Another important property of the graph of y = l o g x is t h e
following: As x increases without l i m i t , y a l s o Increases
without l l r n i t . By 9-3g, we lolow that if xl < x2, then log
xI < log x2, Since 1 < 2, log 1 < log 2, and 0 < log 2,
Consider log 2". -
Since l o g 2n- n log 2, it follows t h a t
log 2n increases without limit as n increases without limit.
Thus, the point (2n, n log 2) is on the graph of y = l o g x,
-and the ordinate of this point is a r b i t r a r i l y large if n is suffi-
c i e n t l y large. Since y always increases as x increases, it
f o l l o w s t h a t y increases without limit as x increases on the
graph of y = l o g x.
A c l r s e l y r e l a t e d p r o p e r t y is the following: As x decreases
toward zero, y decreases without llimi on the graph of y = log x
Another way t o state this property is the following: the graph of
y = log x is asymptotic t o the n e g a t i v e y-axis. It follows from
Equation 9-312 that l o g n
2
- 1
- - - n log 2.
Thus, the p o i n t (I-p-
is on the graph of y = log x.
n log 2)
II
As n Increases without limit,
the abscissa of this point de-
creases toward zero, and the
o r d i n a t e decreases without limit.
Since y always decreases as x
decreases, the graph of y = l o g x
ia asymptotic to t h e negative
Figure 9-4a. The graph
y-axis as stated. of y = log x crosses
The final property of the every line y = c once
and o n l y once.
graph of y = l o g x is the
following: If c is any real
number, then the graph of
[ s e c . 9-41
y = log x c r o s s e s the line y = c at one and only one point.
This property is an important consequence of the fact t h a t the
graph of y = l o g x is continuous, and Fig. 9-4a g i v e s a
g r a p h i c a l proof of it. The figure shows the graphs of y = log x
and y = c. If the graph of y = l o g x crosses the graph of y = c
once, then the curved line cannot cross the s t r a i g h t line a second
time because the o r d i n a t e y oL the graph of y = l o g x always
increases as x increases. Thus, the proof w i l l be complete if
it can be shown that the graph of y = l o g x crosses t h e line a t
least once. It has been shown already in this section t h a t there
is a p o i n t on the graph of y = l o g x above the l i n e y = c and
another point below thfs Une. Since the graph of y = l o g x is
conlinuous, the graph crosses the llne in passing from the p o i n t
below the line y = c t o t h e point above this l i n e . The proof is
complete.
Another statement of the property proved in the last para-
graph is the following: If c is any real number, then there Is
exactly one positive real number xo such that log xo = c.
The following is a sumnary of t h e properties of the graph of
y = log x established in this section.

9-4a On the graph of y = l o g x, the o r d i n a t e y always


increases as the abscissa x increases.
9-4b The graph of y = log x crosses the x-axis at x = 1
and a t no o t h e r point.
9-4c The graph of y = log x is a continuous curve.
9-4d As x increases without l i m i t , y a l s o increases with-
out 1ird.t on the graph of y = l a g x.
9-4e As x decreases toward zero, y decreases without limit
on the graph of y = log x,
9-4f If c is any real number, then the graph of y = l o g x
crosses the line y = c at one and only one p o i n t .
1. FTnd the coordinates of a point P on the graph of
y = loglox which s a t i s f i e s each of the following conditions:

(a) The ordinate of P is greater than 100.


(b) The o r d i n a t e of P is Less t h a t -5.
(c) The ordinate of P is greater than 1 and less than 2.

Hint for ( a ) : Recall t h a t loglOIO = 1 and t h a t

2. D r a w an a c c u r a t e graph of y = loglox on a large sheet of


graph paper ( s e e Exercise 9-1-3). Use t h i s graph to find the
approximate solutions of the following equations: ( ~ o t ethat
graph must extend to at l e a s t x = 100).
(a) loglox = .5 (f) logl+ = 1.2

(4 log 10x = -2.5

3. We l a b e l the log function whose value a t 10 is 1 w i t h the


symbol loglO x. S i m i l a r l y , the log function whose graph
passes through the point (t,l) is called logt x. Find the
value of k associated with logt x . Show also that
logt tn = n, where n is any p o s l t l v e integer,
4. If the graph of y = log x passes through the p o i n t (t,~)

where t > 1, show that log t" = ns f o r any p o s i t i v e


integer n. A l s o find the value of k such t h a t the graph
of y = log x passes through ( t , s ) .
5 Show that t h e r e exists a number x such that loglO x = tl
Show also that this number is greater than 1000 and leas
than 1000 m.
6. Sketch a curme which has the property that it is symmetric to
the graph of y = log x with respect to the l i n e y = x.
Suppose t h a t the equation of t h i s new c u r v e is y = ~ ( x,)
(a) Re-state properties 9-4a, 4b, 4c, 4d as they apply
to the graph of y = ~ ( x ) .
(b) Which of the following a r e true?
(1) The functions log x and ~ ( x )are so related
t h a t the domain of either functlon I s the range of
the other.
(2) If ~ ( a , b ) l i e s on elther graph, then t h e p o i n t
&(b,a) lies on the o t h e r .

9-5. Tables -
of Common hgarithms; Interpolation.
It was shown in Table 9-1 that the common logarithms of a few
numbers are integers; for example, 10g~~0.01 = -2, loglOl = 0,
and loglolo = 1. The common logarithms of some numbers are

r a t i o n a l fractions; f o r example, loglO = $ ( see Equation


9-3f). The common logarithms of many numbers are irrational

numbers; f o r example, the number


- . 10f i will be defined later,

and it w i l l be shown that loglOIO


f i is the irrational number

fi The usual tables of logarithm express approximate v a l u e s


o f the logarithms of numbers in decimal form correct to four,
five, or seven decfmal places.
[set. 9-51
Four-place t a b l e s w i l l be used in this section and the n e x t .

Table 9-ga. Approxinations to a Few Common Logarithms a n d T h e i r


Representation in Standard Form.

x loglox logl# in Standard Form


0.00231 - 2.6364 -3 + ,3636
0.0231 - 1.6364 - 2 + ,3636
0.231 - 0,6364 - 1 + .3636
2.31 0.3636 o + ,3636
23.1 1.3636 1 -t -3636
231.0 2.3636 2 + .3636
2310.0 3 3636 3 + 03636
23100.o 4.3636 4 +- .3636

It has been shown that the logarithms of numbers greater than


1 are positive, and t h a t the logarithms of numbers less than 1
are negative. The second column of Table 9-5a gives the comon
logarithms of numbers listed in the first column. The t h i r d
column shows the logarithms w r i t t e n In standard form. It will be
observed that loglOx, when wrltten in standard form, is the awn
of an i n t e g e r ( p o s i t i v e , negative or zero) and a non-negative
decimal f r a c t i o n less than 1. The integer is called the charac-
teristic o f the logarithm, and the decimal f r a c t l o n is c a l l e d t h e
mantissa. Thus, the standard form for wr3ting the common log-
arithm of a number a is
Definition 9-5a. l o g a = n + m, where
n is a p o s i t l v e or negative integer or
zero, and
O<m<l.
We i l l u s t r a t e t h e meaning of this d e f i n i t i o n with some examples:
Example 9-5a. Find the characteristic n and t h e mantissa
m of logjO a for each of t h e following values:
(a) loglOa = ,4829
Solution:It is important to observe t h a t the characteristic
n can be zero as it Ls in t h i s case, We c a n write:

log10a = ,4829 = 0.4829 = 0 + .4829 where


n = 0 and m = .4829, Note 0 Lrn < 1.

(b) loglOa = 3.3122 + 1.5040


Solution: Clearly loglOa = 4.8162 = 4 + .8162, therefore,
n = 4 and rn = .8162. Again, 0 < m < 1,
(c) loglOa = -2.4163
Solution: If we write l o g 10a = -2 -k (-.4163), we observe
t h a t the decimal f r a c t i o n is negatfve and therefore cannot be re-
garded as a mantissa which, by definition, fs a non-negative number
less than one. In this case log a is larger than -3 and less
than -2. This means that loglOa can be expressed as -3 plus
some positive number less than one, This positive number I s our
mantissa m.
ioglOa = -3 + m or -2.4163 = -3 + m. rn = .5837. mis
gives loglOa = -3 + ,5837. We see that n = -3 and t h a t
0 i r n < 1. Note t h a t we could have obtained this result more
quickly by adding and subtracting 3:

EKample 9-5b. Find the characteristic n and the mantissa


m for
loglOa if 5 loglOa = 2 logl# - 3 loglOy, where
loglox = 0.1962, and loglOy = 0.7343 - 2.
Solution: 5 logloa = 2 x (0.1962) j(0.7343 - - 2)
= 0.3924 -
3(-1.2657)
= 0,3924 + 3.7971
= 4.1895
logloa = 0.8379. n = 0 and rn = .8379.
O i m < l
I

mercisea 3-5a
Find the characteristic and the mantissa f o r loglOa in Ecercises
1-12 :
1. loglOa = 3.8383

3 loglOa = -.4431 Hint: loglOa = -1 + ?

7. log 10a = .2727 - 3.8122

2 '[ 3
loglOa = 3 logl+ + logl# - 51 loglO z] where

loglox = 0.3163, log y = -.888?,


10
loglCz = - 7.4175
13. L e t n represent t h e characteristic and m the mantissa f o r
loglOa, a > 0 . Is the following statement true?: If
loglOa = 0, then m = 0 and n = 0.

14. Are these statements true?

(8) If loglOa and loglob have the same mantissas, then


they differ by an integer.
(b) If logl@ and loglob d i f f e r by an i n t e g e r , then t h e i r
mantissas are equal.
k t us now consider two p o s i t i v e numbers whose decimal
r e p r e s e n t a t i o n s d l f f e r o n l y in the position of the decimal p o i n t .
We see t h a t 73.18 and ,07318 a r e a p a i r of numbers of this type.
In t h i s case, we note t h a t 73.18 = .07318 x 103 .
The sample of logarithms given in Table 9-5a suggests t h a t the
common logarithms of any two numbers whose decimal representations
d i f f e r o n l y in the positions of the decimal points have the same
mantissas. This fact w l l l be proved In the f o l l o w i n g theorem:
Theorem a If a and b are any two p o s i t i v e numbers
whose decimal representations d i f f e r only in t h e positions of the
decimal p o i n t s , t h e n loglOa and loglob have-the same mantissas.

The proof employs the p r o p e r t i e s of logarithms established in


Section 9-3, For convenience, assume that a > b; a similar proof
can be g i v e n if b > a , men there exists a positive integer n
such t h a t
a = lonb.

Recall t h a t l o g (xl xp) = log xl + log x2, log xn = n log x, and


loglolo = 1.
492

Then,

Thus, loglOa is obtained by adding the integer n to logmb,


and the mantissas of loglOa and loglob are t h e same. The proof
is complete.
It follows from Theorem 9-5a t h a t the logarithms of all num-
bers can be obtained from a table which gives the logarithms of
numbers from 1 to 10, Comon logarithms are preferred to nat-
ural logarithms for ordinary computation because of Theorem 9-5a.
The mantissas of common logarithms a r e obtained from a table, and
characteristics are obtained by i n s p e c t i o n as indicated in the
next Theorem ( 9 - 9 ) .
Before we consider this theorem, let us recall the meanings
that have been assigned to such expressions as lo0, and l o 3
and locn where n is a positive integer. We have long known
t h a t 1 9 X lo2 = (10 x 10 x 10) (10 x 10) = 105 and, more gener-
ally, that
(i) 1
pX 10" = where m and n are positive i n t e g e r s
called exponents when used in this way.
Zero and negative integral exponents were defined so that this law
(i) remains true. Suppose n = 3 and m = 0. We have
ld ' = lGtO = I.$.
x 1
0 Evidently 1 0
' must be assigned the
value 1 in order for the statement t o be true, We can mite
loo = 1,
Fig. 9-5b gives a schematic diagram which indicates how the graph
of y = loglox can be approximated by a straight l i n e in such a
way as to give a solution to t h e problem, S i m l l a r triangles give
t h e equation

Pig, 9-5b. mplanation of linear interpolation


f o r finding a number when i t s loga-
rithm is given.
Thus, z is approximately 3 , and the number whose cornon loga-
rithm is 1,7940 Is approximately 62.23,
The problem just explafned c a n be solved a l s o by finding the
equation of the straight l i n e AE in Fig. 9-5b. This l i n e passes
through the p o i n t s whose coordinates are (62.2, 1.7938) and
(62.3, 1.7945). The equation of this l i n e is

If y = 1.7940 on this l i n e , then x = 62.23. Thus, if


logl# = 1.7940, then x 2 62.23.
Table 9-5d. FOUR-PLACE TABLE OF COMMON LOGARITmS
lhercisea g-5c.
Use T a b l e 9-5d with the f o l l o w i n g exercises.
1. Find the logarithm of each of the following numbers:
( a ) 342.0
(b) 38.4
( 4 *735
( 4 e0945
( 4 58900
(f) 21.4
(g) 349.0

2. Find the logarithm of each of the following numbers. Inter-


p o l a t i o n is required.
( a ) 684.2
(b) 9,484
( c ) .06254
(a) *7328
( e ) 271.6
( f ) 1.647
3, Find the numbers that have the following l o g a r i t h m :

( a ) 2 + -4425 (f) 0 + .3522


(b) 2.4425 (g) 1 + *2330
( c ) -2 + .a274 ( 1 -3 + ,6839
(a) -2.7167 (1) -3.2924
(e) 4 + .6646 (9) 3.7135
4. Flnd the numbers that have the followlng logarithms. Inter-
p o l a t i o n is required.
(a) 2 + ,4505 (f) -2.4748
(b) -1 + ,9156 (g) -2 + h7592
(c) 4 + ,1320 (h) 1 + .8487
(a) 5.3328 (i) -1 + ,6329
( e ) -2 + .4748 ( 9 1 3 + *4279
5. D r a w an accurate graph of y = loglox on a large aheet of

graph paper. Use Table 9-5d as a t a b l e of values for plott-


l n g the graph.

96. Computation With Common Logarithms.


Computation wlth common logarithms rests on two simple f a c t s :
(a) A number can be found (by using tables) if Its logarithm
1s known.
(b) By using the properties of logarlthms established in
Sectlon 9-3, it is frequently possible to find t h e
logarithm of a complicated expression q u l t e simply from
the logarlthms of the individual numbers in the express-
ion.
The procedure is best explained by means of examples. Since a l l
logarithms in this section are common logarithms,the subscript 10
has been o m i t t e d from the symbol loglOa in order to simplify

Example a Find the value of 27.43 x 71.64.


Solution:
L e t a denote the value o f t h e expression. Then by the
propertles of logarithms established in Section 9-3,
l o g a 2 l o g (27.43 x 71.64)
2 log 27.43 + l o g 71.64.
In o r d e r to make the addition easy, the work may be arranged in
tabular form as followa:
log 27.43 2 1.4383
log 71.64 2 1.8551
log (27.43 x 71.64): 3,2934
mample 9-6b. Find the value of 71.64
25.m
Solution: By the properties of logarithms established in
Section 9-3,

log 71m64 = l o g 71.64 - log 25.64.

log 71.64 Z 1.8551


log 25.64 1.4089

l o g 71.64 2 0.4462

2 2.794.

&ample 9 . Find the value of 27.43 X (71.641~


(25. 64)3
Solution: k t a denote t h e value of the expression. Then
by t h e properties of logarithms established in Section 9-3,

log a = l o g L27.93 x (71.64)2] - log ( ~ 5 . 6 4 ) ~


= l o g 2 7 - 4 3 + 2 l o g 71.64 - 3 log 25-64
In order t o make the additions and s u b t r a c t i o n s easy, t h e work
should be arranged I n tabular form as follows:

log 27.43 2 1.4383


2 log 71.64 23.7102 l a g 71.64 2-1

3 l o g 25.64 2 4.2267
1 log 25.64 2 1.4089 1
27.43 ( 7 1 . 6 4 ~ ~ 2 0.9218 3 log 25-64 4.2267
log
(25. 64)3
Example 9-6d. Flnd t h e value of J&.
2 .8

Solutton: It was shown in S e c t i o n 9-3 t h a t l o g % = $ :log a.


x,1
Also, l o g - = l o g x - l o g x2. Thus,
"2 1
2' .8
log & = $ ( l o g 25.8 - l o g 6'1.8)

l o g 25.8 2 1.4116
l o g 64.8 2 1.8116

log J&
2 ,8
:$(-,4000) = -0.2000.

Again, we observe that t h e number -0.2000, being negative, camot


be regarded as the mantissa of a logarithm, because all mantfssas
are, by definition, non-negative numbers l e s s than one. Moreover,
we have no negative entries in o u r t a b l e of mantissas (9-5d).
Therefore, we must write t h e number -0.2000 in standard form,
where the decimal f r a c t i o n part is positive, as we did in some of
t h e exercises following D e f i n i t i o n 9-5a.
-
We have -0.2000 = -0.2000 + 1 1 = 0.8000 - 1

. . log
' Js 2 -1 + .8000

and J s 2 0.6310
Example 9-6e. Flnd the value of (0.08432)5.
Solution: It was shown i n Section 8-3 that log (0.084~2)~=
5 log (0.08432). From the Table 8-5d and the rules f o r
characteristics,
log (0.08432)
-- - 2 + .9259,
l o g ( 0 . 0 8 4 ~ 2 } ~-2 10 + 4.6295
-
2 6 + 0.6295
Then
N o t e t h a t it is o f t e n advantageous t o keep the decimal frac-
tion p a r t of the logarithm p o s i t i v e , For t h i s reason we did not
express log 0.08432 in the equivalent form -1.0741 although it
would n o t have been wrong t o do so. In f a c t , if we use this value,
we have
log (0.08432)~ 2 -5.3705
which is correct, but, because t h e decimal fraction p a r t is nega-
tive, is not useable w l t h our table, If we add and subtract 6,
we have
-5.3705 + 6 - 6 = 0.6295 -6 as shown above.

Example 9-61. Find the value of v(0.078461~


Solution: The c a l c u l a t i o n i s carried out as follows:

4 log 0,07846 2 -4.4212


1
3
14 l o g 0,07846]2 -1.4737 = -1.4737 + 2 - 2

m a log J
'- 2 -2 + .5263

Exercises 9-6,
Use Table 9-5d t o compute t h e value of t h e unknown in each of
the following expressions:
1. x = 53.89 x 0.7394 .
2. y = (141.6)(0.299) .
If d = 7 , find H when d = 2.166 and R = 1200

~f t = w find t whenl = 95.8, and 13 = 980.

Use = 3-14.
2 log x + log

24. If A = (I i-rln, find


(a) k when n = 30 and r = 0.03 ,
(b) r when A = > and n = 4 0 ,
(c) A when r = -0.05 and n = -20 .

9-7 Lo~arithms--
with an A r b i t r a r y Base. -
I n Section 9-1, t h e r e was d e f i n e d f o r each k > 0 a loga-
rithm functlon as the area associated with t h e hyperbola y = k z.
For p a r t i c u l a r values o f k like k = 1 and k = M = 1
In=
0.43429 ... ( s e e 9-la), we o b t a i n the natural logarithm f m c -
t i o n and the common logarithm function r e s p e c t i v e l y . In general,
as stated in Equation 9-1, the logarithm Pwction associated with
a particular p o s i t i v e value o f k has the property that

If a is any positive number which is not equal to one, then


the ratio
log x k k x = -l n x
l o g a - k l n z h a
is independent o f t h e particular k used t o d e f i n e the l o g func-
t i o n . In other words, the r a t i o of t h e values l o g x to log a
depends only on the numbers x and a and not on the p a r t i c u l a r
logarithm function used. Thus, t h e function fa d e f i n e d by

is independent of k, For example


log
flo (XI = -0 = loglOXl

since LoglOIO = 1.
Definition a . For a > 0 and a # 1, the function fa
-
defined by (9-7a) Is called the logarithm function --
w i t h base -
a.
We write f a (x) as logax. Thus

Hence, for each positive a # 1, we have associated a loga-


rithm function with base a, In particular, the equation preced-
ing D e f i n i t i o n (9-7a) t e l l s us t h a t this new logarithm function
wlth base 10 is our o l d friend the common logarithm function, If
we denote that value x f o r which In x = 1 by the l e t t e r e,
then, by Definition ( g a l a ) , the logarithm function with base e

loge x - 1 0 x~.
log e
lo x
But since A
log e = In e for any logagarit function,

9-7c loge x = I n x ,
That is, t h e natural logarithm functlon is precisely t h e logarithm
f'unction with base e . The number e t h e r e f o r e , takes on a
s p e c i a l significance. It is an i r r a t i o n a l number whose value,
correct to 10 decimal places is given by
e 2 2.7182818285.
Notice that logarithms w i t h base 1 are excluded from De-
finition (9-7a) because log 1 = 0.
The motivation for defining logax as that ratio lo x is
that this ratio depends only on x and a, and n o t on the p a r t i c -
ular p o s i t i v e number k used to define log x. As a matter of

lWbX
fact, the ratio -
logp
is independent of b. Note that

lo x
logbx = &,. and logba =

so t h a t t h e i r ratio is precisely
lo x which, by definition is
&=

log, x.

[sec; 9-7 J
This simple relation,

f a r p o s i t i v e a and b different from one, is called the change


----
of base law for logarithms,
Two particular bases a r e interesting. Let x = b in (9-7d);
since logbb= log b = 1, we have

9-7e log b =
a
-
3.
logb&*

But logla = = -1 and (1) becomes


-a
-
log a

9-7f logax = - loglx.


-a

We write down several simple properties of the logarithm


functions with arbitrary bases. The proofs of these properties
follow inmediately from the f a c t t h a t logax = l o g a w.
me proofs
are left as exercises.
9-7e log, 1 = 0.

9-7h log, a = 1.

9-71 log, an = n, f o r any integer n.

9-7 J l0g,(xl .x2 )= log, xl + log, x2'


Two other propertlea which w i l l p l a y an important part in the
next section arc
9-7k loga xl fag, x2 lr and only if xl x2.

9-7p For each r e a l number a, the equation log, x 5 s


has a unique solution.
To prove (9-7k), we obaerve that

loga xI - log
l o g a x2 or-1OPI a fi
log x2
-1 if and only

If log x l - log x2. Moreover, according t o (9-3h) and (9-3hh'),


log xl - log x2 I P and only Zr xl= x2 and our proof is complete.

To prove (9-71))we observe that log, x - s l a equivalent


to t h e equation log x = s log a, which has a unique aolution
according to (9-4f) if' we consider e t o be the ma1 number
s log a.

The fallowing examples illustrate the a p p l l c a t l o m of some of


the r e l a t l o n a dereloped'in t h i s sectlon.
F i r s t , we compute aoms logarithms with various basea
[~~harnples 9-7a t o 9-7a).

=ample -7. Compute lagh 32.

Solution: log4 32 - Og
&
log p2
, s j 3,
Example .
9 - 7 ~ Compute log5 10.
Solution: Thls t h e we use (9-7d) instead of usisle: (9-7b)
aa we did in the first t w o examplea.

Nota t h a t it is possible to avoid this long divlaion by using


1
logarithms. Let t =-,. Then,

Solution: logl 10
5
- +- - lo
lo& 5
10
log 5
= -10% 10: -1.431.

Of course t h i s answer could have been obtained by applying (9-71)


t o the results of Example 9-7c.
Example 9-7. Find N if log3 N - 4.

Solution: 10% N - - 4

log N - 4 log 3 log 34 - log €31

m p l c
integer.
9 - Show that log, x" - n log, x i f n i s an

Solution: We b o w that log xn - n log x by (9-3=). By

m * ;OqL.
a OF,
-. log, x .
k - 2
=ample 9-76. The logaklf hm function corresponding t o
~ o l n c i d a swZth the logarithm k r n c t l o n w i t h what baae?

Solution: U@ h o w that 10% x 15 t h a t logarithm functkon


whose value a t b l a I. According to m a t i o n 9-1,
log x - k In x . Sbce lee b = 1, we have
1 1 2 I n b tor k - 2 *
.*. Inb-F
1 Lhrt,by(g-7c),

log b - .I.
log e - l o g n (9-3fg p - -
1, q 2)

... b - f l , because x1 - x2 if log x l - log x2.

mercis.@s9-7a
1. Flnd the value of t h e following logarfthms wikhout t h e use
of t a b l e s :

(h) loglO 0.01


(b) 10%~ 9 x (r) logb 9 f i 5
-
(c) leg
9
N I
i (=) log 1 N
x
- -0.75
(dl logJ5 K 1 -4 (h) logb g 1.5

3. of Table 9-5d t o compute the flellowing l o g a ~ l t h m s


M a k s l use
correct to the nearest thousandth:

(b) log, 200 (e) log2 10

-
4. Show that:
(a) log5 2 x l o g 2 5 1 (b) 10% 2 + logl 2
5
- O

5. Salve for x:
(a) 10% x - 1.17 (b) logl x
w
- -0.301

6. Pmre the follotdng statements:


(a) 9-78: lo& 1 - 0 . (0) 9-71 loga a" - n ror any
integer n .

7. If l o g x N = s , and logxb-t, find logt,~.


8. Complete the following table:

9. The logarithm M e t i o n cosreapanding to k 1 5 colncAdes


Hith the logarithm kulctlon w i t h what base?
10. Compare the results of the preceding e x e r c i s e wlth &ample 7
and f i n d t h e base of the logarithm function colncZding w i t h
the logarithm k v l c t i o n corresponding t o any k > 0.
11. Show t h a t the solution of the equation log, x = s is the
erne as t h e solution of the equation logb x - 8 logb a
provided a and b a r e pasitfve numbers not equal to one,
and s I s a real number.
k t
defined by y
US

- -
eXamine the graphs of several logarithm functions
log, x.
Xr a 10, we have the familiar graph or the c o m n loga-
rithm function shown In Figwe 9 - l i . If a m 100, we c a n sketch

To do this, we let a 1 110 and b -


the graph of loglOO x by comparing I t w l t h the graph of loglO x .
LOO in Equatldn 9-Td and
mi tc
loglOO x
1
- log3oo 110 x l o g l 0 x .
1 loglO x.
Now, loglOO 10 -7, so we obtain loglOO x = 5 From
thia we see that e v e v ordinate of the graph of g = loglOOx is
ont-half the corresponding oMinatc of t h e graph of - loglO x,
-
y
Similarly, each ordinate of the graph of y logl x is the neg-
rn
a t l v e of tho cormspon8ing ofdinate of the graph of y rn laglO x ;
eund each ordinate of y log x is the negatlve of t h e corres-
m
pending ordinate of the graph of y loelm X. All four of theao
graphs are sketched In F i g , 9-7a.
Fig. 9-7a
Theae graphs indicate that:

(11) ~f a < 1, > o ir x < 1 and log, x < o ir


x > 1.
9-7m
For
(ill) a > 1, logog,xl < log, xp if and only if

( (iv) or a < I, log xl


a
< 1 0 4 xg 11 only if

These statements are indeed trme and they fallow cUrect1.y


from (9-7b) and the corresponding properties of the l o g function
glven i n Section 9-3.
1. Sketch the g ~ n p h sof y = x and y - logl x on the
E o
same s e t of axes,
2. If n is a ratural number, ahow t h e t t each ordinate of the
graph or y = log 1 t b t e s the eorrtapor,dlng ordinate
x is -
n
of the graph of
an
loga x.-
(a) IP 1 <a and, a < b then logab > 1.
(b) If' 0 < a < 1, and a <b then logab < 1.

4. Prove properties (i), (ii), (iii). (IV) for loga x by


rnaklng use of the corresponding propertlea of Log x and
without reliance on the p a p h s shown in Figure 9-7a,
5. Show t h a t if l <a<b and x >1 then logax > logbx.
If, hordver, O <x <l then lohx ( 10%~.

The f'ollontng is a s m a r y of properties of logarithm


tkons with an arbitrary base:

Definition 9-78. r a > 0 and a # 1, t h e logarithm


function wlth bast a I s deflned by the function

log, x - lagbn
10€fbX
' B L b + l , X > O , e > O , b >
9-71. log,an - n, ror any Integer n. a Z 1 , a > 0.

9-73. loga X1*X2 - loga x1 + loge X*, &I, s>O, xl>O. x2)0

9-7*- 1 0 ~ ~ log&
5 if and only if xl - xp,a#2, a)O,xl>O,xg>O.

9
-
7
!
. For each r e a l number s, the equation logax - s
has n untque s o l u t i o n , a ) 0, a + 1.
9-73' logax" - n logax, Tor any integer n, a+, a>O, X>O.

9-7m l o g , x l < l o g a x g l ~ a n d o n l y i f xl<x2, afl, a s , xl>O.

--
9-8. mponential b c t l o n a - - h w s of mponents
b?t us look once again a t the graph or t h e fhmcticm defined
by y = loga x , ( a > 0. a # 1, x > 0 ) .

The domain of thia function cansists of all poaitive nmbers


and i t s range consist8 of all real numbers. We have seen that; m y
horizontal l i n e y = s will intersect this graph Isl ono and only
one polnt (~1- 9-&a), In other words the equation log x m s
a
haa a unique solution. Accordfng to our dkscussion of invepae
functions I n Chapter 3, Section 8, t h e logarithm function h a s an
inverse function which we w3ll call, f o r the moment, Ea. Thia
Inverse t h m t l b n 19 then defkned by tho equation,

-
Y E,(x) a

We should note that E,(x) is not defined for a 0 or 1 be-


cause loga x l a not defined for these values of a.
Agdn drawlrig om Chapter 3, we recall that inveroe functions
have the property t h a t t h e i r graphs are symmetric in the lLne
y = x. T h i ~ract enables us t o sketch the graph of y - E,(x).

This could be accomplished by drswlne the graph of y I logax in


Ink and then f o l d i n g the paper along the l i n e y = x so that an

of y -
lmpmaaion is made while the ink I s s t i l l wet. The resulting graph
E,(x) is shorn in Figure 9-8b.

Figure 9-8b
F r o m the graph I t I s clear t h a t the domain of E, is a41 real
numbere and the range of E, is all positive numbers.
Sinue Ea and log, are inverse functions, we l a o w from our
d i s c w s l o n in Chapter 3, Section 8, t h a t each of them "undoes" what
the other one does. T h i s means that
9-8a log^^ 1 - s or (i) l o g a s is the unique solution of
E,(x) - s, and

9-8b ~O&~[E,(U)1 - u or (11) E,(u) l a the d q u e solution oP


logax - u.

Thlg l a t t e r fact, (li), enables ua t o oompute ~,(n) when n is


an integer. We aak f o r the solution of
log,(x) - n, where n is an integer.

Since Ea and loga are inverse f u n u t i o m , we k n o w that

log,[~,(n) ] - n. However, acconling to (9-71).

logaan - n. where n is an integer. Therefore,

9-8c -
~ ~ ( n B)n , because xl Q x2 if log, x, loga x2
according t;o (9-7k).
In particular, If n 0, we heve
9-8d, -
~ ~ ( 01,) and if n - 1

Equation 9-& furnishes u a with a compelling and pennantnt


notation ror the function E,, The function is c a l l e d the exponen-
-
tial function ---
with base a, and E is m i t t e n as, when a

La called t h e base and a is c a l l e d the exponent. The symbol a3


th
is read as "a to the s power", or simply "a to the st'.
k t w now review what m b o w about the f u n c t i o n Ea '

[I) E,(s) is defined Tor all reel numbers 5.

(11) E,(s) 1s the unique s o l u t i o n of logax - a.

E,(s)
(ill) has the some value as as when s 1s an

The f i r s t two statements f o l l o w directly f m m the fact t h a t E,


was defined as the Inverse of log,; the t h i r d statement I s another
w a y of saying Equation g-8c.
These etatements, (i), (ii), (111) suggest t h a t as
might be defined in t e r n of E,(s), i.e, as a unique s o l u t i o n

of the equation logax - s. If t h i s la done, we will have a sorv-


l c e a b l e d e f l n i t i o n for n9 when s is % r e a l number, whereas
until now a" has been defined only f o r the case when s l a an
integer. Moreover, the new d e f i n i t i o n , while much broader, agrees
wlth OW previous Interpretation of a".
Aocordingly, w t adopt the following d e f l n i t i o n :
k f i n i t i o n 9-$a. a > 0, a # 1, ma s is a reell
number, as is that m a 1 number x which i a t h e unlque positive
s o l u t i o n of the equation logax a.
Since we n o w write as for E,(B), Equatlans 9-8a and 9-8b
become reepectivtly

9-BL' a
lw,u
- u for a l l u > 0. (log, u
solution of ax
is t h e unique
- u)
9-&3 logaa3 - s f o r a l l real s. (aS Is t h e unique solution
of l o g x = s )
a
Equatlone (9-8f) and ( 9 - 8 g ) together are equivalent to this
statement: If a > 0 and a r( 1 thcn ax and l o h x are
inverse functions.
The meaning of o u r new definition (9-8a) and the equivalence
or E,(s) and as are illustrated by t h e following examples and
exercises :
-3,
I
manplt 9-8a. kraluate 3/. -
3tj-, or %(f),
Solution: Aceoxling to our d e f i n i t i o n ,
the unique p o s i t i v e solution of the equatlon lo x - 1
A
5'
la

Butp

and x 2 l.&h2-
Example 9-83,. Use cbrmKIn loearithms Zo approximate the
value of

solution: $( 0)or 3 Jf 1
. defined as the positive
solution of
log7x -fl . Applying (9-76) re have

and x 2 66.10 . 1
ample 9-8c. Find t h e value of z2 by sketching t h e graph
or $(XI.
Solution: We seek ~ ~ ( 3Me) .can obtain an approximate
value Imm the graph of' E*(x). WO laow that E~(x) I s the in-
verse or the function l o ~ x . Therefore, we can obtain bhe graph
or ~ ~ ( xby) first grapNng y u log2x and cncr~reflecting this
graph in t h e l l n e y x , as we d i d In Figure (g-~b). F i r s c , we
make a table f o r y - log$.

Tne corresponding table for y R $(x) is obtained by interchang-


1 1 31
end y.
-
m g x I h e r e i o ~ e ,p i n t s A(-4,T6), B(-3,g), c(-2,ri)r
etc.. l i e on the graph of y ~ ~ ( x )SLnce,
. lo+x has a con-
timow graph, its Inverse function E ~ ( X ) also has e continuous
graph. He obtain this graph by drawing a smooth curve through
A, B, C,
1
...
as shown in F i g u r e ( 9 - 8 c ) . The ordinate correspond-
Ing to x = is approximately 1.4.
Exercises 9-8a.
1. Evaluate t h e following by means of Definltlon 9-8a and Equa-
tions 4-81 and 9-8g:

2. Use common logarithms to find the approximate value of each


of the expressions l i s t e d below. It w i l l be necessary t o
make c e r t a i n approximations and the answer you o b t a i n w i l l be
o n l y an approximate answer. W s approximate answer should
be as accurate as t h e use af a f o u r place logarithm t a b l e
w i l l permit.
(a) 31*T2
) 40.48
(c) *& (k) 10J 2
-0.48
(d) (11 (1.251
(4 3- (m) loR
(f) 2 J 3
3. Draw the graphs of the hulctlans defined by

(8) Y = 3X (b) Y - log3x

on the sme s e t of axes. How am these graphs related? F m m


the graphs read the appmxhate valuea of 3 1.7 ,
,
4. maw tne y a p h of the M o t i o n defined by y - l o g m x.
Wrlte the equation wMch defines the Inverse of thia f'unctlon.
Draw the gsaph of the inverse function,
5 Follow the Instructlorn e v e n in Bcerc5sd 4 above f o r the
functions defined by:

b. ~uppoee c ~ , C2 and C
3
are the gsapha of three h c t i a n s ,
fl, fg, and f and suppose further that ( a ) C1 and Cp are
3
syametric v l t h respect to the y-axis, (b) C2 and C are
3
symoet~tcwith respect to y I x. If fl is defined by
y = )
Ix ,
(a > 0, a 1). Wrlte the equation which defines

5-
We are n o w In a position ta prove a very important r e l a t i o n
whlch 1s baaed on (9-8g) and t w o PomuZaa f m m the preceding
section, (9-7d) and (9-7c).
Aaoordlng t o (9-7d) L

But loexxt - t for t any ma1 number by (9-8%),


and
Ogx
I - logox
In dealing with posltive I n t e a l exponents In our previous
studies in algebra, the number as was Snterpreted t o mean the
p r o d u c t o b t a i n e d when a is used as a ractor s tlmas, It was
then a slrnple matter t o v e r i f y t h a t , if s and t are p o s i t i v e
integers, and a and b are reel numbers,

Later we &Pined a' and a-', where s is a p o s i t l v e


fnceger, so t h a t (9-813 remained v a l l d , and we wore l e d t o con-
clude that
9-81 a0 - 1 , and
a -'- as
- are appropriate d e f i n i t i o n s provided

a { 0. In fact, it I s readily shown that (9-Bi), ( 9 - 8 j ) , and


(9-8k) are y a l l d when s and t are any integers if d e f i n i t i o n s
( g - 8 j ) and ( y-am) a r e accepted.
Xou we have assigned a meaning to as f o r any real exponent
s, provided a > 0 and a P 1. Do the relations (9-81) through
( 9 - h ) remain v a l i d when s and t are any r e a l numbers and a
and b any p o s l t i v e numbers n o t equal to one? The answer I s yes,
and we shall prove it d i r e c t l y . But l e t us f i r s t give a name t o
t h e relations (5-81)through (9-831) -- call them the --
exponents. 1(oreover, to dispose of the c a s e 2 - laws of
1 which is not
covered i n our d e f i n l t t o n , l e t ua agree t h a t '1 shall equal 1
f o r a l l r e a l a , It is then easlly seen that f o r a
b > 0 t h e laws of exponents are v a l l d ,
- 1 and
Theorem a Let a and b be any positlve numbers, then
f o r a l l real numbers s and t the laws of exponents (9-81)
through ( 9-8m) are satisfied.
Proof of 9-81: asat = a s + - t
t
(1) lag, aSat = log, as + log, a (9-73)
s
(2) log, a = s, log, at = t (9-8f)

3 loga aSat = 9 + t From (1) and (2)


log, aS + t , s + t ( 9-Sf)
(4) log, a + = log, a at s
(31
s t ( 9-7k)
(5) as + t = a a
Proof of 9-8j: (aslt = a st.
(1) log,(as)t = t laga as ( 9-8h

(2) log, as = s ( 9-8f)

(4) ~ u t ,log, aSt = st ( 9-cf 1


s t st
(5) 9 ' . l o g a b ) = log, a ( 3 ) and ( 4 )
(6) (as)t = a
st
( 9-7kl
8 s
Proof of 9-8k: (ab)' = a b
(1) log,,(ab)' = s (9-8f)

3 ) logab as = s lagab a and


S
logab b = a logab b
(9-7k) and (9-7h)
( 4 ) and ( 5 )

(1) and ( 6 )
s s
(8) (ab)" = a b (9-7k)
Proof of 9 - 8 1 : a0 = 1
(1) log, 1 = 0

(2) log, a0 = 0

In Section 7 we developed a I t change of basett formula f o r t h e


logarithm function. Equation (9-7d) can be written in the form
logb x = log, x logb a

which enables us to express t h e logarithm of x to the base b


as a multiple of the logarithm of x to the base a, We now de-
velop a similar change of base equation f o r t h e exponential func-
tion,
For example, we might ask: "mat power of three is equal to
t h e third power of nine?" To answer this we must solve the equa-
tion 3X = 9. In thls caae it is readily seen that the value of
9
x Is 6 . Ordinarily t h e s o l u t i o n of the equation ax = b I s
more difficult.
We have learned that if t w o numbers are equal, then t h e i r
logarithms t o any base are equal ( 9 - ? k t ) . Therefore, log, ax =

loga b" and t h i s equation is equivalent t o x = s log a b accard-


ing t o (9-7h) and (9-811). Accordingly,

9-8n b s = as logab . (a > 0, b > 0, s any r e a l number).


A s p e c i a l case of this formula which is frequently used in
mathematics is o b t a i n e d by l e t t i n g a = e, t h e base of n a t u r a l
logarithms:
9-80 b s = e s logeb or bS =
s l n b

At this p o i n t it is a p p r o p r i a t e f o r us to c o n s i d e r t h e r e l a -
tion between r a d i c a l expressions such as
Js; , qn
and expressions involving positive r a t i o n a l exponents such as

5
1
', 2 '-
1
and a
1
-1
'.
Consider first and a wherz q is a natural number.
-1
According to D e f i n i t i o n 9-8a, a is defined a s the unique
positive solution of log x = That is,
a 4'

In Section 3 , qfiIs d e f i n e d as the positive number whose


gth power is a. That Is
If two positive numbers are equal t h e n t h e i r logarithms t o base a
are e q u a l (9-7k). Hence,

(iii) log, (qo)q


loeaa , or =

(9-8h) and (9-7h;

-*
*
9
(vi) loga qfi
= loga a (1) and (4
-1
(vii) qf=la (9-7k)
We have t h u s established
-I
qfi = a q where a >o and q is a natural
number.
-1
NOW thatwe have established the equallty of 'fl and a q,
it is readily seen that
E
= (qfi)P
= x where p and q are posl-
t i v e integers.
The proof requires our new equality and t h e "power of a power" law,
(9-8j). We have
-1 E
(1) qp( x p p = = x q and
-1 E
(ii (qfi>" ((x q)P = ,q.
Equations (i) and (ii) together a r e equivelent to (9-8q).
We c l o s e this section with the statement of a theorem which
summarizes t h e r e l a t i o n betwcen logarithmz and exponents:
Theorem 9-W. as = N if and only If s = log, N, provided
a is positive # 1 and a ie real.
Proof:

(2) log, as = log, N ( 9-7k)

(4) :. s = log, N

The proof of the second part of thls theorem, if 8 = log, N,

then as = N, is left t o the student.


The following is a summary of the properties of the exponen-
tial function:
kfinition 8 If a > 0 , a # 1, and s is a real num-
9
ber, a I s that real number x which I s the unique p o s l t l v e
solution of the equation logax = 8.
logau
a = u for a l l u > 0, a ) 0, a j 1.

9 4 logaas = s for all real s, a ) 0, a { 1.

Equations (9-8s) and (9-8g) together are equlmlent to


thls statement: if a > 0 and a # 1, then ax and
logax are inverse functions.
9-?h logaxt = t logax for a >0 and a # l and
x > 0 , t real.
9-81 a B x at = as + t , a ),0, e and t real,
9-8~ (as)t = ast , a > O, a and t real.
9-8k (ab)' = asbs, a > 0, b > 0 , s real.
9-81 aO=l, a>0.
s 13~,b
g-z5 ks r; 2 -
- c s lr. b , b > 0, s real.

p ~ s i t i v ei n t e g e r s .
S
Tkeorer. 9-eb 2 = 3 If and only If s = log I;.
a
Tne f o l l o w i r ~ gexamples show sore applications o f t h e
laws 3f exponents:
aarrple 9-66, show tnat (E)' = f-
bS
,.5
where & r k are

positive r e a l numbers, and Ls real.


.First S o l u t i o n :

as s
~lso, logc - = l o g f i a - 1 0 ~ =~ s ~1 ' 3 ~ -
~ 52 1 3C i~
bL L,

- s(log,a
., - 1 3 ~ ~ b )

S e c c n d Soiution: We l e a r n e d in C h a p t e r 1, S e c t i o n 6, t h ~ t
Example - 9 - e e . -&pressions i n v o l v i n g r a d i c a l s may be express-
ed In e q u i v a l e n t foms i n v o l v i n g p o s i t i v e rational exponents.
V e r i f y c h e following:

L ~ a n p l e 9 - b f . kn expression Involving r a t i o n a l exponents


nay be converted into an e q u i v a l e n t r a d i c a l form. V e r i f y t h e
following:

&ample 9-8g. Expressions involving negative integral ex-


ponents may be changed to equivalent expressions in which a l l ex-
p o n e n t s a r e positive. V e r i f y the f o l l o w i n g :
7
Example 9-8h. Some expressions i n v o l v l n g r a d i c a l s and
r a t i o n a l e x p o n e n t s are easily evaluated. Verify the f o l l o w i n g :

-
-

(b) 273 = ( 3 n 7 p = j2= 9 .

&ample 9-81, Express 7h'13 as a power of 1).

Solution: We a p p l y (9-8n), b s = a s logab , Let b = 7,


4.13 logl37
S =4.13 and a = 13. We have 7"'13 = 1) = 13X
loglO7
where x = 4.13 l o g 7 log 7 =
13 r 13 10gl013

1. 0.8451
and 10g137 * 1.1139 '

:. 4-13 l o g
13
7 5 4.1 x 0.84 1 = X

and log x = log 4.13 + log 0.8451 - l o g 1.114 .


.. x = 3.134

and 7 4.13 2 13 3.134


Example 9-8j. Equations in which a t l e a s t one exponent is a
function o f t h e unknown a r e c a l l e d exponential e q u a t i o n s .
Solution:
1 1
2 T X =3x
(a) g2 = (3 .'. )X 2 - = 3X. If we find t h e
logarithm of each member, we oStain

x2 - x - 6 = O .
.*. x = 3 or x = - 2. Each r e s u l t checks.

(b) Find the common logarithm of each member:

Exercises 9-8b.
J3valuate the f o l l o w i n g .

(b) 11°

(h) (0.0001)
?
E,

[sec . 9-81
2. Write each of the following as an equivalent expression in
which a l l exponents are positive.

(b) (a-2b)-3

(c) (X-I + fl) (X-l - fl) (g) -+g


x - Y
(d) a2b-I

(i) (Xm2 + y)-2


Wrlte each of the following as an equivalent expression
in radical form.
1 1
(f) 57
2
*( j) &press with a single radical s i g n .

L J

4. Use common logarithms t o compute %he value of each o f t h e


following expressions.

(a) (24.80)-~

J. I

( 0 ) (0,8412)*~ (f) 16 ( 0 . 0 0 7 ~ 3 4 ) (~8 2 6 7 1 ) ~

5. Show t h a t xS = yS lf and only If x = y provided x and y


are positive # 1 and s is real.
6. Solve the following equations f o r x.
x2-12
(a) 2X + = 32 (d) 252X = 5
2
(b) 92X = 273X - (e) 8 = hX 25X

( 4 2x2 + 4x = 18 (f) 2 X h 5 = l o X

7. Solve the following equations f o r x.

(a) loa - = 43 ( e ) (1.031~= 2.500

(b) e3x = 16 (f) e2x - X


2e + 1 = 0

(g) log3 (x + 1) + 1 0 g 3 ( x + 3 ) = 1

(i) 1 0 4 = X = 0
-1
8. Prove that xS and xS are inverse functions. (x> O,s#O,
1
s real). Graph y = x3 and y = n7 on the same s e t of axes.

9. Prove as = N if s =logN.
a (a>O, a # 1 , Nreal, N > 0 . )

10. Draw the graphs of (1) y = 2X and (2) y = QX on t h e


same s e t o f axes. It w i l l be found t h a t each abscissa on t h e
graph of (1) i s t w i c e the corresponding abscissa on the
graph o f 2 Can you generalize this statement f o r the

graphs of (1) y = ax and ( 2 y = bX (a>0, b > O , x real).


1 If O < a < b compare a
X
and bX when

(a) x > O

(b) x = O

12. Solve each of the f o l l o w i n g equations as fndlcaled,


( a ) Solve y = c xn fern.

(b) Solve u = a e -bV f o r v. (use n a t u r a l logarithms.)

(c) Solve s = a (*> f o r n.

( d ) Solve x = log y for y.


5
(e) ~ o l v e R= a rn - 1 f o r n.
(f) S o l v e for x: loglO(x - 4) c loglO(x + 3) = logl$O.

(g) S o l v e the following equations f o r x and y.

l o g 10x + log y = 2
10
x +y = 25.
3 . Solve f o r x.

(a) ex + e-X = 2
X -X
(b) e - e = 2
14. According to the law of radioactive decay, the mass m re-
mainlng t years from now is given by the formufa:
-c t
m = m e
o , where e is the base of t h e n a t u r a l log-

arithm, mo is the present mass and c is a constant de-


pending on the p a r t i c u l a r radioactive s u b s t a n c e involved.
The h a l f - l i f e of a radioactfve substance 1s t h e time elapsed
when m = $no Find the half-life of a radioactive substance
f o r which c = 2.
15. If an mount of money P is Invested at an i n t e r e s t rate of
r (expressed as a decimal) p e r year, the amount A accumu-
lated a t the end of n years, when interest is compounded
annually, i s given by the formula: A = ~ ( +1 r)n. This
statement is known as the compound interest law,
( a ) Find the amount A to which an investment of $1,000
will accumulate in 20 years if i n t e r s t is compounded
annually at 6s.
(b) How Long w i l l it take f o r an Investment t o double itself
1
If i n t e r e s t is compounded annually at 4 7 $?

(c) If one dollar grows to J d o l l a r s in 30 years when


interest is compounded annually, f i n d the approximate
rate of interest.
9-9 Qcellaneous Exercises
1. Flnd t h e value of:
logp55 - logl 8
(a)
T6

(b) log 1
5
+ logg 4 . 5-

2. Find the value of x:

(a) x = l o g 81
3

(b) log2x = -5

(c) hgx8 = f (h) l0g~0.04 = -2

(d) 1030.2 = x (1) 10% a=


1

(e) logx49 = 4 (J) logoello = X

3. Complete the following statements:

(a) l a gaaf =

(e) The inverse of the function defined by the equation


y = ax is defined by t h e equation y = ?
4. Solve f o r x:
(a) l o g x = 3
5

2
(d) loglox - loglox = 2

5. Write with p o s i t i v e exponents and simplify:

-1
(h) x + y-'

6, Solve f o r x. (N and a are p o s i t i v e real numbers).


(a) ax = N ,
(b) xa = N , x > 0,
(c) N = logax . a # 1, x >o
7. Find the value of x by means of the laws of exponents:

(d) -
4* =

8. Find the numerical value of x f o r each o f t h e following.


Base necessary computations on Table 9-5d.

(a) 1dL = 41.63 ( 9 ) l o g y = 2.4

X
(@> I3 = 35 (k) ~ O ~ ~ , ~ O . O ~ = X

(4 ( 0 . 5 ) ~= 70
9. Solve for x, Assume that a l l other l e t t e r s represent
p o s i t i v e real numbers. Express a l l logarithms to base LO.

(a) ax = b (e) m = arx

(b) xa = b (f) A = ~ ( +1 x)'

(c) b = logax (g) A = P(1 + r)X

(d) m = axn (h) ex L n b z c

*lo. Compare the graphs of y = loglO 2x and y = Are these


inverse functions?
*11, How to construct a logarithmic scale: On a sheet of paper
draw a l l n e 10 inck.zs long, Mark o f f in t e n t h s and hund-
redths. Fold t h e sheet of paper on t h e line so you can use
it as a ruler. On another sheet of paper draw a line equal
to 10 inches. Align the ruler with t h i s l i n e . Because the
log of 1 is 0, p l a c e 1 on your new s c a l e opposite 0
on you ruler, Because 0.3010 Is the log of 2, p l a c e 2
opposite 0.30 on y o u r ruler. Proceed s h l l a r l y until you
have placed 10 opposite 1 on your ruler.

LOG SCALE

RULER

(a) Using two of these l o g s c a l e s , can you make a s l i d e


rule? Can you explain how a s l l d e rule multiplies and
divides?
( b ) Construct a coordinate system using logarithmic s c a l e s
on the coordinate axes instead of the normal llnear
.
s c a l e s . On this coordinate system p l o t y = x 2 Can
you explain the result?

(c) Construct a coordinate system using a logarithmic scale


on t h e axes of ordinates and the normal linear scale on
the axls of abscissas. On this system plot y = 9 .
C a n you explain t h e result?
Chapter 10
INTRODUCTION TO TRIGONOMETRY

10-1. Arcs and P a t h s ,


Let P and Q be any two
d i s t i n c t points on a circle with P
radius r. These two p o i n t s
separate the circumference of a
c i r c l e i n t o two arcs, the sum of
whose lengths is 2rr. The
length of any arc of a c i r c l e is
Ff gure 10-la.
equal t o or less than 2m. A r c s on a c i r c l e .
Let P be a p o i n t on a
given c i r c l e as in Figure 10-lb.
Let a point R s t a r t a t P and
move, without r e v e r s i n g i ts
direction, a distance d along
the c i r c l e to a final p o s i t f o n
F
i 3
p
Q ( o b s e r v e t h a t d may be great-
er than the circumference o f the
c i r c l e ) . This motion will be
called a path, and it will be 'u,.\ .
Figure IO-lb
denoted by the symbol ( ~ , + d ) A p a t h on a c i r c l e .
if the motion is in the counter-clockwise direction, and by the
symbol ( P , - d ) if t h e motion is in the clockwise direction around
the c i r c l e . The symbol (P,o) corresponds to the path in which
G does not move from P. The p o i n t P I s the i n i t i a l p 0 4 of
of the path, and t h e final position of Q is t h e terminal p o i n t
of the path.
Observe that every path is described by a symbol ( P , c ) ,
where c is some positive or negative real number. Conversely,
if c is any real number, t h e r e is a unique path on the given
c i r c l e corresponding t o t h e symbol ( P , c ) . .
Two paths ( P ~ , c ~ and ) (P2,c2) are equal if and o n l y if

LG
P1 = P2 and c I = c 2 . Two p a t h s are e q u i v a l e n t if and only if
c1 = c 2 , Observe that t w o p a t h s are equivalent if they a r e e q u a l ,
b u t t h a t two e q u i v a l e n t paths need n o t be e q u a l ,

p
If ( P , c ) is any
t haet hr e onFs aagiven
uniquec i path
rcle, J{ ,
( P ~ , C ) on this c i r c l e
which has its i n i t i a l
p o i n t a t a given p o i n t
\ El P,
Po and which is equiva-
l e n t to (P,c).
We shall now define, C
t h e addition of p a t h s . Figure 10-lc.
The unique equivalent p a t h
If (Pl'cl) and (p*,c2) with inftlal p o i n t at G.
a r e any twc paths on t h e
same c i r c l e , then
( P ~ ' c ~+) (p2,c2) = (p19c1 + c2).
Since (pp,c2) + ( P ~ , c ~= )(p2,cg + cl), we see that (P1,cl)
+ (p2,c2) and (p2,c2) + ( P ~ , c ~ a)r e not equal unless P1 = Pg.

Nevertheless, ( P p , c p ) + ( P ~ , c ~ IS
) equivalent to (pl,cl)
+ (p2,c2), since cp + cl = cl + c p by the commutative p r o p e r t y
of the addition of r e a l numbers.

3igure 10-16. Graph of ( P ~c , + ( P ~c2)


, .
[ s a c . 10-11
Exercises 10-1

Let PI and P2 be
points on a c i r c l e of
radius 5 as shown in
Figure 10-le. Draw
diagrams to show t h e
following paths:

(4 (P~.")
(b) (P~,~OT)
(4 ( ~ ~ 9 - r ) Figure 10-le.

(dl (p2,-1W Figure f o r Exercises 10-1.

(h) (P*, - 3 0 ~ )

2. Draw diagrams t o I l l u s t r a t e the following additions of paths:

3. Which ones o f the sums in Exercise 2 above are equivalent?


10-2. S i m e d Angles . e,
The rays 3 and. A & form
an angle in the elementary sense
( s e e Figure 10-2a). It will now
b e shown t h a t paths on a circle
can be used t o extend the
elementary n o t i o n of angle.
Figure 10-2a.
Consider a c i r c l e w i t h An angle in the elementary sense.
radius 1 whose c e n t e r is the
vertex of the angle formed by rays
+ +
AP and AQ. F i g u r e 10-2b shows
a p a t h ( P , Q ) on this c i r c l e
t h a t can be associated with t h i s
angle. It is immediately c l e a r ,
however, that o t h e r paths could
be associated with t h e elementary
angle. To overcome this d i f f i c u l t y
we introduced the n o t i o n of signed
angle. There is a one-to-one Figure 10-2b.
Angles and paths.
correspondence between the s e t of
signed angles and t h e s e t o f paths
on the unit c i r c l e .
Let a p e t h (P,Q) on the
u n i t c i r c l e be given ( s e e Figure
10-2c). The ray AP is t h e
Initial s i d e of t h e corresponding
signed angle. If Q is t h e
t e r m i n a l p o i n t of the path (P,Q),
t h e ray A& is the terminal side
of t h e signed angle. The p a t h ~ i g u r e10-2c,
Generation of angles.
(P,Q) s p e c i f i e s how the signed
angle is generated, in the follow-
+
ing sense* The ray AR I s placed in the initial p o s i t i o n AP
-
and then rotated about A so that R traces the path (P,Q) .
The terminal p o s l t i o n of AR is then The signed angle is
+
the t r i p l e , A&, (P,Q)) ; it I s completely determined by the
path (P,Q) and t h e vertex A in the sense that the signed a n g l e
can be constructed when A and ( P , Q ) a r e given. It is thus
appropriate to denote the signed angle CAP, AQ, (P,Q)] by
(A,P,Q).
A signed angle has a d i r e c t i o n Q
a s s o c l a t e d with it. If 8 > 0,
the angle ( A , P , Q ) is generated by
r o t a t i n g AR in the counter-clock-
wise direction, and we say t h a t the
angle is p o s i t i v e ; if Q < 0, t h e
angle ( A , P , Q ) is generated by
r o t a t i n g AR in the clockwise Figure 10-2d.
Signed angle,
d i r e c t i o n , and we say t h a t the
angle is negative.
Example 10-2a. Construct t h e angles (A,P,$), (A,P,-T),
5 and A - 1 0 where P is a given f i x e d p o i n t on a
unit c i r c l e w i t h c e n t e r A .
Solution: The angles are shown in Figure 10-2e.

Figure 10-2e. Constmction of the f o u r angles indicated.


We say t h a t two angles ( A ~ , P ~ , Q ~and
) P ~ are
( A ~ , ,Q2)

-
e q u a l if and only
angles a r e equal,
if A1 = A2, PI = P2, and Q1 = Q2. If t w o
they c l e a r l y have the same v e r t i c e s , t h e same
initial s i d e s , and the same terminal sides. It I s n o t t r u e , how-
ever, that two angles with the same vertices and initial and
terminal sides are e q u a l . If ( A ~ pl, , Q ~ ) and ( A ~ P2, , Q2) have
t h e same initial and terminal s i d e s , then Al = A 2 and

where n is 0 or a positive or negative i n t e g e r , Furthermore,


angles w i t h the same initial and terminal sides are c a l l e d
co-terminal angles. Two co-terminal
angles are shown i n Figure 10-2f.
Two angles A , P , and
( A ~ , P ~ , Q are
~ ) equivalent if and
only i f OX = Qp. If the signed
angles ( A1, PI. Q ~ )and ( A2, p2. Q*)
are equivalent, t h e n the geometric
Figure 10-2f.
angles PIAIQ1 and P2A2% are Two co-termfnal angles.
-
congruent in the sense of geometry.
-
Example 10-2b. The two angles (A~,P~,$) and (A~,P~,$)

shown in Figure 10-2g are equivalent, but the two angles


A , , and A P - ) show. in Figure 10-2h are n o t
equivalent.
An angle is said t o be in standard position in a coordinate
system if and only if i t s v e r t e x 9s a t the origin and its i n i t i a l
s i d e extends a l o n g t h e p o s i t i v e x-axis. Every angle is equivalent
to one and only one angle In s t a n d a r d position. It will b e
convenient to d e n o t e an angle in standard p o # i t l o n by (o,x,Q).

Figure 10-2g. Equivalent angles.

Figure 10-2h. The angles (A~,P~,$)


and (A~,P~,;) are not equivalent.

Example 10-2c. Construct the anglea in standard p o s i t i o n


1la 4a
denoted by the symbols (o,x,$), ( o , x , ~ ) , and ( o , X , - ~ ) .
Construct two other angles which are co-terminal with each of
these angles.

[ s e c . 10-21
mlution: The s o l u t i ~ n eare d ~ o w nAn F l g u m 10-21. Recall
that the length of the c11cczDnPerence of the unit ctrcle ia 2rr.

I
llw *
P i g u m 10-21. The angles ( o , x , ~ ) , (o,x,+~
0 x ) and two angles which are co-terminal w i t h each.
The a d a t i o n of paths s w a t s hbw anglee are to be added.
The ~ollowtngs b t e m n b define tlie,addition of two angles an8
the m l t i g l i c a t i o n ~of an angle by a real number c:

The properties tf theee operatima follow from the pmpertiea of


the corresponding operations on the real numbers.
ft is now clear that an angle (o,x,Q) In standard poaition
I s completely determined by the aingle real number 0 . Hence-
forth, we shall apeak of the angle 8 and man themby the
angle (o,x,B). The aum of the angles Q1 and Q2 is Q1 + Q2;
t h e addition of angles has all of the properties of the addition
of ma1 numbers. Furthermore, c timea the angle 9 is the
angle c0; the m l t l p l l o a t i o n of angles by a real number ha8 a l l
of the propertie8 of #e multigllcation of real numbera.
[asc. 10-21
Exercises 10-2

1. Given a unit c i r c l e with center A and a point P on it.


Cons truc t the angles .
(a) (A,P,.rr) (d) (A,P,~)

(b) (A,P,-?) (e) (A,P,-~)


(c) (A.P,J$) (f) (~,P,-l.5).

2. Construct the following angles 1n standard position.


(4 (o.x,$) (4 (o,x,-~)
(b) ( o , x , ~ ) (e) (%x,-~I
(4 ( o , x , - ~ ) (f) (o,x,~)
3. Find two p o s f t i v e angles and two negative angles which a r e
co-terminal w i t h each of the angles in Exercise 2.
4. Construct t h e following angles in standard p o s i t i o n and find
one negative angle which is co-terminal with each one of
them,

(a) (J) g
(b) g ( 1 3
(4 f LP) 9
(dl 5 (m)
3?T
7
(4 5 (4 q
(fl 7a
(0)

lla
(PI 3-
10-3. Radian Measure.
We have defined the signed angle ( A , P, 0 ) in terms of the
path ( P , B ) on the unit c i r c l e whose center is A . The real
number 8 1s called the radian measure of the angle ( A , P , B ) .
It follows from the deffnition o f e q u i v a l e n t angles given in
S e c t i o n 10-2 t h a t any two equivalent angles (A1,P1,Q1) and
(A
2
,P2 ,82 ) have t h e same radian measure 8, where 8 = Q1 =
O2
11
The statement "the angle 8" uaually means the s i g n e d angle
in standard p o s i t i o n whose radian measure is 8 " . Of course there
are i n f i n i t e l y many o t h e r angles t h a t have the same radian
measure 8 .
The radian measure of angles I

is e s p e c i a l l y u s e f u l because there Q'


exists a simple r e l a t i o n between
the length o f an arc of a c i r c l e
and t h e radian measure of the angle
subtended at the center of the
circle. Figure 10-3a shows an
arc P t Q ' of length s on a
c i r c l e of radius r, and the
corresponding a r c PQ of' length
8 o n a c i r c l e of radius 1. By .Figure 10-3a.
a theorem on similar sectors of A r c 3 on a circle of radius r.
c ire les , we have

- a r c P 4 Q t or
a r c I PQ - Q = -3
r r
But Q is the radian measure of the angle fomed by the rays
and 3. Thus, the formula

gives the radian measure of t h e angle in terms of r a d i u s of t h e


clrcle and the l e n g t h of the i n t e r c e p t e d a r c . Formula 10-3a can
be stated also in the form
Formula 10-3b gives the length of an arc in terms of the radius of
a c i r c l e and t h e radian measure of the subtended angle.
Example 10-3a. Find the radian measure of the angle subtend-
ed a t the center of a circle of radius r by one-fourth of the
circumference.
Solution: The length of the circumference o f a c i r c l e of
radius r is B i r r , ahd one-fourth of the clrcwnf erence is F.
Ey Equation 10-3a, 8 Fr
a-= -
T
2'
-
Example 10-3b. An arc on a c i r c l e of radiua 10 subtends an
angle of 2.5 radians at the center. Find the length of the arc.
Solution: By Equatlon 10-3b, s = 10 x 2.5 = 25. The reader
should draw a figure.

Exercises 10-3

I. Compute t h e radian measures of the angles determined by the


following values o f s and r.
(a) s=17, ~ = 5 (e) s = 2, r = 5
(b) s = 10, r = 5 (f) s=3a, r = 5
(c) s = 8, r = 10 (g) s = 6 ~ , r = 10
(d) s=4a, r = 4 (h) s = a, r = 1

2. Compute the l e n g t h s of the arcs determined by the following


values of r and 8.
(a> r = 5, Q = 0.2 (e) r = 10, Q = 2.7
T
(b) r = 5, Q = T (f) r=10, Q = 7~g
(4 r = 5 , Q = 2 (g) r=10, Q=3.2
K
Q = E 2a
Id) r = 5, (h) r=10, Q = T
3. On a c i r c l e of radius 24 inches, find the length of an arc
subtended by a c e n t r a l angle of:
(a) 2 radians . (c) 4 radians .
(b) radians .
4. FZnd the radius of a c i r c l e f o r which an a r c of 15 inches
long subtends an angle of:
(a) 1 radian . (c) 3 radians .
(b) 2 radians .

0 Other Angle Measures.


The radian measure of angles was t r e a t e d in the l a s t S e c t i o n .
The size of an angle (A,P,Q) l a determined by the length of the
p a t h (P,Q). In the radlan system of measure, the unit of length
used i n measuring the l e n g t h of the path is the length of the
radius of the c i r c l e (see Equation 10-3a).
The circwnf erence of the
0
circle containa 2rr of these
units.
Another aystem of measure
c a n be obtained by using the
length of the circumference as
the u n i t length for paths . The
angle subtended by an arc one
circumference in length is called
one revolution. Since one circum-
ference subtands an angle of 27r Figure lo-4a.
An angle 9 of one radian.
radians or 1 revolution, we
have 1 r e v o l u t i o n = 2~ radians.
A third system of measure results from u a i n g 1 of the
circumference of the c i r c l e as the unit of length. The angle
subtended by
1 of the circumference is called one degree.
S i n c e one circumference subtends an angle of 360 degrees or 1
revolution or 27r radians, we have t h e following basic s t a t e m e n t
of equivalents:
10-4a. 1 revolution = 2a radians = 360 degrees.
The degree is f u r t h e r subdivided i n t o 60 equal parts called
minutes ( a b b r e v i a t e d mln. and denoted by , as in loi) ; the
minute finally is divided into 60 equal p a r t s called seconds
(abbreviated s e c . and denoted by ", as in 20"). Thus,
10-4b lo = 601, 1' = 60".
It Is customary to measure angles in degrees, mlnutes, and
seconds in surveying and in the solution of triangles. The radian,
however, is the simplest unit for measuring angles in those
problems which involve the differential and integral calculus.
-
Example 10-4a. Ffnd the measure of each of the following
angles in t h e o t h e r two systern : radians , 150' .
Solution: From Equation lo-4a,
1 rev.
a radlana = 180°, ?r =

or IT
5 radians = 30°, 9 ,=
1 rev.

Similarly,
1 r e v . = 28 radians, 1 rev. = 360'
3 rev. = 3 8 radians, = 540';

and 10 = m1r e v . , lo = a radians


150' = 5 rev. , 0
150 =
5~ radians .
Exercises 1 0 - 4

1. Express the following in degrees.


( a ) 3 revolutions (e) .I25 r e v o l u t i o n
(b ) - $ revolution (t) .833 revolution

(c) 2 revolution (g) -1.5 revolutions


5 revolution
(d)- 5 (h)
1
-$ revolutions

2. Express the following In revolutions.


(a) 135O (e) 67'30t
(b) -60' (f) 930'
(c) 210° (g) -485O
(d) -150' (h) 360'
3. Express the angle as a multiple of T radians.
(a) 30' (g) -112O40'
(b) -25' (h) -315O
(c) -160' (1) -180'
(d) 135' (J) 300'
(e) 36' (k) -90'
(f) 75'30' 880'
4, Express the following In degrees.
-48
(4 g (9) 3
(b) $ (h) 6
(4 L g

(f) 7T
15
5. 2
In a triangle, one angle is 36' and another is T~ radians.
Find the t h i r d angle in radians .
6. Through how many radians does t h e mlnute hand of a clock
revolve in 40 mlnutea?

10-5. Definitions -- of the Trigonometric Functions.


We s h a l l define t h e t r i g o n o m e t r i c functions i n this section.
Some functions, such as the logarithmic and exponential f u n c t i o n s ,
have names; the trigonometric functions a l s o have names, The
situation is sometimes confusing because s e v e r a l different b u t
c l o s e l y r e l a t e d functions have been given t h e same name. F i r s t
we s h a l l define the t r i g o n o m e t r i c functions of angles in standard
position. Next, we s h a l l define the trigonometric functions of
arbitrary angles, and finally we shall d e f i n e c e r t a i n additional
trigonometric functions which are closely r e l a t e d t o the trigono-
metric functions of angles.
Definition lo-5a. L e t (o,x, 8 ) be any angle in standard
p o s i t i o n , and let ( X ~ , Y ~ )be the intersection of its terminal
s i d e w i t h the standard unit circle. Then
sine of (o,x,Q) = Yo sin 8 = yo

cosine or (O,X,Q) - xo cos Q = x0

tangent of -
(o,x,Q) = Yo
Xo
tan 8 = -
Yo
Xo
provided xo # 0

cotangent o f (o,x,~)= -
X~
cot 8 = -
X" provided yo # 0
Yo Yo
secant of (O,X,Q) = -1 sec 0 = - provided xo # 0
*o Xo
c o s e c a n t of (o,x,Q) = -1 csc 8 = - provided yo f 0.
Yo Yo
where the statements on the r i g h t are abbreviations f o r the s t a t e -
ments on the left.
These d e f i n i t i o n s do not enable us to calculate these s i x
functions except fn a few special cases since it is usually n o t
possible t o f i n d the coordinates of a point on the terminal s i d e
7f the angle (o,x,B). In c e r t a i n important s p e c i a l cases, how-
ever, the c a l c u l a t i o n is p o s s i b l e as shorn in the following
examples.
Example 10-5a. Flnd a l l six trigonometric runctlons of .
'
0
3
Solution: Figure 10-5a shows
t h e angle ( o , x ,3
0
'
) .
The terminal
side of this angle i n t e r s e c t s the
standard unit c i r c l e in the point

1
sin30°= 7 c o t 30' = fi
6 fi Figure 10-5a.
cos 30' = 300 =
2
7 The angle (0,~,30') .
t a n 30' = & =1 7 csc 30' = 2 .
Example 10-5b. Flnd a l l s- trigonometric functions of 120'.
S o l u t i o n : Figure 10-5b shows
the angle (0,~,120'). The termlnal \ IY
s i d e of t h i s angle intersects t h e
standard unit circle In the p o l n t
- Then

sin laoO = 6 c o t 120° = - 6


-5

cos 120° = - H1 aec 120' = -2


Figure .
tan 120' = - cao 180' = -.2 J3 The angle (0,X, 120') .
-
Example 1 0 - s ~ , Find a l l s i x trlgonometrlc functions of
,Y
270'.

Solution: Figure 10-5c shows


t h e angle (O,X, 270'). The terminal
s i d e of t h i s angle i n t e r s e c t s the
standard unit c i r c l e in ( 0 , -1) .
Tan 270' and s e c 270' a r e not
defined s i n c e x = 0 . The other
values are
\lo; I 1
sin 270° = -1 c o t 270' = 0
F i g u r e 10-5c.
c o s 270' = 0 c s c 270' = -1 . The angle (0,x,270°).
Definition 10-5b. Let (A,P,Q) be any angle, and l e t
(o,x,Q) be t h e u n l q u e angle in standard p o s i t i o n to which it is
equivalent. Then
sin ( A , P , Q ) = sin (o,x,Q) c o t (A,P,Q) = cot (o,x,Q)
cos ( A , P , Q ) = cos (O,X,Q) sec ( A , P , Q ) = sec (o,x,Q)
t a n (A,P,Q) = tan (o,x,Q) csc (A,P,Q) = csc (o,x,Q).
If we p a i r w l t h each signed angle ( A , P , Q ) the r e a l number
s i n (A,P,B), we define a function whose domain is t h e s e t of all
signed a n g l e s . It follows f r o m Definition 10-5a t h a t its range
is x : -1 x 1 This function is denoted by s i n 8, and 8
is most commonly measured in degrees. P a i r i n g cos ( A , P , B ) w i t h
(A,P,Q) defines a function whose domain is the s e t of a l l signed
angles and whose range is Ex: -1 i x 513; it is denoted by
zos 8 . The functions t a n 8 , c o t 8, sec 0, and c s c Q are
d e f i n e d in a similar manner. The functions s i n 8 , ---, csc 0
are called t h e s i x trigonometric functions.

-
Theorem 10-5a. L e t 8 be any angle in standard position
whose terminal s i d e does n o t lie along one of t h e axes, and l e t
~ ( x , y ) be any point on i t s terminal s i d e . Then
sin 0 =
Y cot 0 = -Y
X

sec Q = 4 '+ y2X

tan Q - Y.
X
csc 0 =
Y
Proof: Let r be the
distance from 0 to P. Then

r = J m e equation
of the line through 0 and P
is
y = -Y o X , X
Xo I
where (xo,yo) Is the polnt at
which t h i s line i n t e r s e c t s t h e
standard unit c i r c l e .
The point P(x,y) is one
of the i n t e r s e c t i o n s of this line Figure 10-5d.
The figure f o r
2
with the c i r c l e x + y = r
2 2
. Theorem 10-5a.
The two intersections are found
by s o l v i n g the following system:

The two solutiona are (rxo,ryo) and (-rxo, - r y g ) (remember that


xO2 + = 1). Since a ray lies entirely In one quadrant, the
a l m s of the coordinates of are the same as the signs of the
P
coardlnates of (xo,yo). It follows that the coordinates of P
are (rxO,ryO) ~hus
Than, by Definition 10-5a,
sin Q = y
+ Y
and the o t h e r statements In the conclusion of the theorem are
obtained in the same way.
Remark: In the course of the proof of the l a s t theorem, we
proved the following important f a c t : If (x0,yo) i3 O n e Point
on the t e r m i n a l s l d e o f an angle, then all polnts on t h i s terminal
side have coordinates o f the form

We shall now define a second s e t of trigonometric f u n c t i o n s .


This second s e t is h i g h l y important in more advanced mathematics
and also in this course. This second s e t of functions is so
c l o s e l y r e l a t e d t o the f i r s t s e t t h a t the two are often confused.
Let ( C , X , Q ) be an angle in standard p o s i t i o n , and let 8
be its radian measure. If we p a i r w i t h the r e a l number 8, the
real number s i n (c,x,Q) as defined in D e f i n i t i o n l0-5a, we
d e f i n e a f u n c t i o n whose domain I s the s e t of a l l real numbers and
whose range is x : -1 x 1 . This function is obviously quite
d i s t i n c t from the f u n c t i o n defined in Definition IO-5b. The
domain of the former f u n c t i o n is the s e t of all signed angles, but
the domain of the p r e s e n t function Is the s e t of all r e a l numbers.
It would be appropriate to denote the former function by
sin (A,P,Q) and the l a t t e r f m c t l o n by sin 8. Unfortunately,
b o t h are denoted by s i n 8 , but it will usually be c l e a r from the
context which is intended. It will usually be t r u e t h a t s i n 60"
means the sine of the angle whose measure 18 60°, whereas,
sin ;means the s i g n of t h e number 9.
In t h e same way, we d e f l n e t h e functions t a n 8,
cos 8,
c o t 0, s e e 8, and c s c 0, whose domains a r e t h e set of all
real numbers. If 8 is the measure of (0,x,8 ) in degrees,
and if we associate with the real number 8 , the real number
s ~ ~ ( o , x , Q ) ,we have a n o t h e r function whose domain is the set o f
a l l real numbers. This function is c l o s e l y r e l a t e d t o t h e one
already d e f i n e d , and it will not be consldered f u r t h e r i n this
course.
The definitions o f the six B

trlgonome t r i c functions can be


s t a t e d in a s p e c i a l way that is
highly useful for an a c u t e angle
in a right triangle. Let A,B,C
be a right triangle as shown in
Figure lo-5e, and let U denote A b
the angle at t h e vertex A . Figure 10-5e.
Functions of an a c u t e angle.

-
Theorem 10-5b. If Q: is the angle at the v e r t e x A of t h e
r i g h t t r i a n g l e shown in F i g u r e 10-5e, then

sina = -ac = opposite side


hypotenuse
cos a = =

t a n aE = -ab -- opposite side


adjacent side

.
cot a =

,,, =
-=
g = adhYPo.t;enus;
-a =
j a c e n t si e
csc a =
-o-p-%
posite sl e
-
Proof: In order to f i n d the
trigonometric functions of a ,
we must first take an equivalent
angle in standard p o s i t i o n .
Figure 10-5f shows such an angle.
The point ~ ( b , a ) is one p o i n t
on the terminal s i d e of t h i s angle.
The statements in the conclusion I
Figure 10-Sf.
of the theorem now follow from An angle In standard
Definition 10-5b and Theorem LO-% posl t i o n equivalent t o &A.

(remember t h a t c =Ja2 + b 2) .

Exercises 10-2

1. Sketch the angle 0 in the standard posltfon and find sin 8 ,


cos Q and t a n Q when the following p o i n t s are on t h e
terminal s i d e of the angle 8.

(a) P(-4,3) (e> P(-2,4)


(b) ~(5,-12) (f P( -7, -24)
(4 P(-l,-l) (g] pI3,-5)
(d) ~(2~3) (h) ~(4,l)
2, In each of the following sketch the angle B and find the
other five f u n c t i o n s of 0 .
(a) tan Q = T 3 , O in quadrant I

(b) COB Q = 7, Q in quadrant TV

(c) sin Q = - B in quadrant IV


(d) t a n Q = -3, Q in quadrant I1
4
(e) cos 0 = --,
7
8 i n quadrant I11
3
(f) s i n 0 = 8, Q in quadrant I1
3. Let P be t h e s e t { a , b, c ] and let Q be the s e t
[sin c , cos a: , tan a , t a n p , c o s f l , a i n B ] . It can be proved
t h a t if any 2 members of s e t P are g i v e n , then a l l the
o t h e r mernbers o f P and Q can be expressed in,terms of
these given members .
(4 If a = 3
b = 4
tan@ = ?
cos a = ?
c = ?

PI ~f b = 12 (dl If a = 12
C = 13 b = 7
a = ? c = ?
sinp = ? sinu = ?
t a n 6: = ? cosp = ?
tan a = ?

4. Let P and Q be the s e t s given in Problem 4. It can be


proved that if a member of s e t P is given and if a member
of s e t Q is given, then a l l other members of P and Q can
be expressed in terms of the given members.

(a) If a = 12 (b) b = 15 (C c = 20
cos a =
b = ?
4 sina =
c = ?
f tanp = 2
a = ?
c = ? a = ? b = ?
tan@ = ? cosH = ? cos a = ?
slna = ? tana: = '? sina = ?

(dl If c = S ( eI a = 2 (fI b = 10
sin a = 3 tan cf = 1.8 cos/ = .8
b = ? b = ? a = ?
a = ? c = ? c = .?
cos/ = ? sin/ = ? sin a = ?
tan a = ? cos a = ? tan/ = ?
10-6. -- ----
Some Basic Properties of the S l n e and Cosine.
In o r d e r to simplify t h e s t a t e m e n t s of some of the r e s u l t s in
t h i s section, it will be convenient t o i n t r o d u c e the notion o f
primary angle.
-
Definition 10-6a. An angle ( o , x , Q ) in standard p o s i t i o n 1s
c a l l e d a primary angle if and only if 0 ( 8 < 360' ( o r the
equivalent condition in o t h e r units of measure).
Theorem 10-6a, Let ( A , P, 8 ) be any angle. Then

( s i n QI* + ( c a s Q)* = 1.

-
Proof: Let (0,X,Q) be the unique equivalent angle in
standard p o s i t i o n . L e t ( x ~ , ~be~ the
) p o i n t where the terminal
s f d e of (o,x,Q) intersects the standard unit c i r c l e . Then

= c o s 8, yg=sin
X~

and t h e proof is carnplete.


Theorem 10-6b. - (converse o f Theorem 10-6a.) Let xo and y 0
be any two numbers such t h a t xo2 + yo2 = 1. Then t h e r e is one
and only one primary angle ( o , x , Q ) such that cos 0 = x
0'
sin 8 = yo.

-
Proof: The p o i n t p(xO,yo) is on the standard u n i t clrcle.
L e t (o,x,Q) be the primary angle whose terminal side passes
through the p o i n t P(xo,yo). Then cos 0 = xo and sin Q = yo.
If ( o , x , Q ') is any o t h e r primary angle, then its terminal s i d e
does n o t pass through ~ ( x ~ , y ~ Thus,
). it is not t r u e that
cos 8 ' = xo and s i n 0' = yo. The proof of t h e theorem is
complete.
Theorem 10-6a emphasizes t h e following c o r o l l a r y , which has
already been observed From t h e def i n f t i o n s in S e c t i o n 10-5.
-
Corollary 10-6a. For a l l angles (A,P,Q)

-1 -< s i n 8 -< 1
C o r o l l a ~10-6b. If yo is any number s u c h that -1 < yo < 1,
there are e x a c t l y two primary angles (o,x,Q) such t h a t s l n 8
= yo. These angles have d
- a,nd - 1 f o r their

r e s p e c t i v e cosines . IY
-
Proof: The line y = yo

i n t e r s e c t s the standard unit


c i r c l e in the two distinct points

There are two primary angles


Q1 and Q2 whose terminal Figure 10-6a.
Two angles for
sides pass through PI and P2. which s i n 8 = y 0 '
Then
J-
oos Q1 = and c o s Q2 = - 43.
Corollary -
10-6c. I f xo is any number such t h a t -1 < xo < 1,
there arc exactly two primary angles ( 3 , ~ , 0 ) such that
these angles are J- and
-
P F O D ~ : The line x = xo i n t e r s e c t s the c i r c l e x2 + y2 = 1
in t h e t w o d i s t l n c t points

There are two primary


angles (o,x,Q~)and
(0,x,Q~) whose termlnal
s i d e s pass through PI
and P2. Then

sin €)I =.Jq


vigure 10-6b.
Two angles f o r
which cos 8 = xo.

-
Corollary 10-6d. There is exactly one p r i m a r y angle whose
sine is 1, and it is 90'; t h e r e f s exactly one primary angle
whose cosine is 1, and it is o O , There is exactly one primary
angle whose s i n e is -1, and it is 270'; there is exactly one
primary angle whose coslne is -1, and it is 180'.
Let Q and 8 + n.360°, where n is an integer, be two
angles in standard position. These two angles have t h e same
terminal s i d e (they are called co-terminal angles), and hence,
the s i x trigonometric functions o f 0 + n .360° are equal respec-
tively to t h e s i x t r i g o n o m e t r i c functions of 8. Hence, if
sin 8 yo, cos B = x
=
0 '
then
sin (Q + 11.360~)
= yo, cos ( Q + n-360') = xo
for n=0, +- +
1 - 2 ... .
Exercises 10-6

1. Sketch a l l t h e angles between 0' and 360' In standard


p o s i t i o n which s a t l s f ' y t h e following conditions and give the
values of t h e other functions f o r t h e s e angles.
24 7
(a] s i n Q = 25
- (e) tan B =

(b) COS Q =--


4 (f)
5
(c) tan Q = -2
-
(a) s i ~ o = ~ -
JT
(h) sin =$
2. In what quadrant will the terminal s i d e of Q l i e if:
(a) s i n 8 and cos Q a r e both p o s i t i v e ?
(b) tan 0 is p o s i t i v e and cos Q is negative?
(c) sin 8 is positive and tan 0 is negative?
(d) cos Q and tan B are b o t h negative?
(e) sin 8, cos Q and tan 8 are a l l negative?
3. Flnd t h e v a l u e of cos2 Q - sin2 Q when tan 8 = - V
3
and cos 0 is negative.
4. Find the v a l u e of tanpQ when cos Q = -7 and tan 0
1-tan 0
I s positive.

5. Prove t h e r e l a t i o n t a n Q = m provided 6 (2k+1)90~


where k is an i n t e g e r .
6. llse t h e r e l a t i o n s i n2 B + cos2 0 = 1 to prove:

(a) 1 + tan 2 8 = s e c 2 0 [0 (2k + 1)90"1 .


(b) l + c o t2 e = c s c2 6 [ ~ # k * 1 8 0 ~ ] .
7. Prove that the range of the tangent -function is the s e t of
all r e a l numbers.
10-7. Tri~onometricFunctions or
Special Angles.
The values of t h e trigonometric functions can b e o b t a i n e d by
simple geometrical cons1derat:ons f o r c e r t a i n s p e c i a l angles.
These a r e the angles f o r which the coordinates of t h e p o i n t
(x0 ,yo), where the termlnal s i d e i n t e r s e c t s the u n i t c i r c l e , c a n
be computed. We l i s t these angles 8 in a t a b l e whlch shows t h e
degree measure of 8 , the r a d i a n measure o f 0, the coordinates
(xO,yO), and t h e values of the s i x trigonometric functions.
Table 10-7a. Trigonometric functions of s p e c i a l angles,

EXo, Y d COS 9 SIN 8 Tan 0 Sec e Csc 8 Cot 8

----- Un- Un-


0 0 (I, 0 ) I 0 0 I
daf ined defined

30 -rr 45 I 4z I I
2 fi
6 , - n3 2

(4
2 ' 42 , d
2 1
45 rr4 2 42 I
3 a 1

Tr
- I & -1 I
60 3 2
-
, 2 &2 J s
90 -rr ( 0 ,I ) 0 I
un - un- I 0
2 -
defined defined

12 0 y - 1 2
I
2 2
I

135
3TT
4 (-2 e 2
z 2 -I
2
3 -
2 -I

5-2 I
- I 2
150
6 2 z z
2 -fi
-
-
(-1 , 0I -I I 0 0 -1
-
Un - Un-
defined defined

2 I0 7TT (-6
2 - 21 -26 --I I --
2
6 , 2 f i f l
7
22s($ -,$I 4
-- I 2
------ , 2 2

240
4l-r
3
I -2 --
.Js
un - Un-
270 (0 --I) 0 -1 defined defined
-1 0

300 5y - 2 1
-2I - -2 -Js
-
2 - -2 - -I
-1

330 -
It is not necessary to rnernorlze the r e s u l t s in Table 10-7a,
but it is important to learn t h e methods by which these r e s u l t s
are o b t a i n e d . I v
Consider f i r a t the a n g l e s
oO, go0, 180°, and 270'.
Figure 10-7a shows a p o i n t
(xO,yo) on the terminal side
of each of these angles. The
e n t r i e s i n Table 10-7a f o r these \ I /
angles a r e obtained by applying
the d e f f n i t i o n s in S e c t l o n 10-5.
Observe t h a t certain of the
trigonometric functions of these F i g u r e 10-7a.
The special angles oO,
angles are undefined,
90°, 180°, and 270'.
The angle 225O, shown in IY
F i g u r e 10-7b, will be used to
illustrate t h e method o f f i n d i n g
the trigonometric functions o f
the s p e c i a l angles 45', 135*, x
225', and 315'. The t r i a n g l e
OPD is an i s o s c e l e s right tri-
angle. Since t h e length 10P I
of i t s hypotenuse is 1, we
rind 1 0 ~ 1 = JDPI = fl
?2' and Figure 10-7b.
the coordinates of P are The special angle 225'.
( - -) . An application of the def lnitions leads to the r e s u l t s
given in the t a b l e . The trigonometric functions of 45', 135',
and 315' can be obtained in a s lmilar manner.
The angle 120°, shown in Y
Figure 30-7c, will be used to
illustrate the method of finding
the t r i g o n o m e t r i c functions of the
s p e c i a l angles 60°, 120°, 240°, I'
! X
and 300'. The t r i a n g l e OPD is D 0 I

a ri@t triangle whose acute angles \ I


are 30° and 6 0'
. Since IOP~= 1,
we f i n d IoD( TI^^/
1
= g . men
lDPl =
A
7, and the coordinates
or P are ( - 9).~n Figure 1 0 - 7 c .
application o f the deffnitions The s p e c i a l angle 120'.
leads to t h e results given in t h e
a b l e . The trigonometric functfons
cf 60°, 240a, and 300' can be
o b t a i n e d in 2 ~ P r n i l a rmanner.
The angle 330°, shown in
Figure 10-7d, will be used to
illustrate the method of finding I
t h e trlgonometric functions of
3oC, 150°, 210°, and 330'.
The t r i a n g l e OPD is agaln a tri-
angle whose acute angles a r e 30'
and 6 0 ~ . Then 1 OD 1 = 9 and
X

IDP( = and t h e coordinates


or F are ( , - in
application o f the definitions
l e a d s t o t h e r e s u l t given In
I
1

Table lo-7a. The trigonometric Figure 10-7d.


functions of 30°, 150°, and The s p e c i a l angle 330'.
210" can b e o b t a i n e d in a simllar
maniier.
Exercises 10-7 -
1. Evaluate the following:
(a) s i n 150' t a n 210' - cos 135' s i n 240'.

(b) cos 90' - s i n 300' + tan 225' - cos 150'.


(c) s i n 270' + tan 180' cos 90'.

2. Flnd all the functions of the following angles without using


a table.

(a) 230' (c) 315O


(b) -135~ (d) -225'.
3: Showthat c o s 2 Q + s i n2 0 = 1 , f o r :
(a) Q = 45O (dl o "= ~
radians

(b) 0 = 150' (e) Q =?a


T radlans

(c) 0 = 330' (f) 2R radians.


Q =J

4. Showthat:
( a ) a l n (60' + 60') f s i n 60' + sin 60'
(b) cos (90' + 60') # cos 9
0
' + cos 60°

(c) s i n (180' + 6
0
'
) # s i n 180' + s i n 60'
(dl cos (150' - 60') # cos 150' - cos 60'
(el sin (300' - 120') # s i n 300' - s i n 120'.
5. Vei-iw the following:
(a) 1 - cos2 60' = sin2 60'
, (b) s i n 60' cos 3
0' + cos 60' sin 30' = 1
(c) coa 60' cos 30' - sin 60' sin '03 = o
6. Which o f t h e following statements are correct? J u s t i f y your
answer.
(a) s i n ~ = 3

(b) s' -- tan Q

(c) s i n 3 0 ° + sin 6 0 ° = s i n 9
0'
(d)
2
cas2 45' + sin 45' = s l n 9 0'

(f) srn 45' cos 45' =


1 sln 9
3 0'

(€5) s i n 3 00 =
1 sin 9
0
'

(3) cos 3
0' + 2 cos 60' = cos 150'

10-8. -
Tables of Trigonometric Functions.
In Section 10-7, we explained how to ffnd s i n 8 , --- , csc Q
f o r c e r t a i n special values of 8 . There is no elementary method
f o r computing t h e s i x trigonometric functions of an arbitrary
angle 8 . In a t y p i c a l case, the s i x trigonometric functions of
8 are irrational numbers which would be represented by non-
terminating decimals. These values can be calculated to any
desired degree of accuracy by methods developed fn calculus.
Tables of the trigonometric functions are available. Table 10-8a
gives sines, cosines, and tangents f o r the angles lo, 2',

We s h a l l now descrlbe certain characteristics of trigonometric


tables.
(1) Since the values of the trigonometric functions are
u s u a l l y irrational numbers, and since the tables give these values
only to three (or f o u r , or five) decimal places, the valuea in the
tables are usually n o t exact. Table 10-8a is correct t o three
[sec. 10-81
decimal places, but tables correct to five, seven, or more decimal
places are available for calculations which require greater
accuracy.
(2) Table lo-& gives sines, cosines, and tangents of
the angles oO, lo, . .. , go0, but it does not g i v e these
functions for other angles such as 37.8'. It will be shown t h a t
the approximate values of t h e functions of these angles can be
obtained by i n t e r p o l a t i o n .
(3) Table 10-8a does n o t c o n t a i n any angles 8 such t h a t
8 < '0 or 8 > 90'. We s h a l l show that t h e approximate values
of t h e sines, c o s i n e s , and tangents of a l l angles can be
obtained from Table lo-8a.
First, we s h a l l give some examples which involve i n t e r p o l a t i o n .
Linear i n t e r p o l a t i o n has been explained already in Chapter 9 on
l o g a r i t h m and exponents, and the theory will not be repeated here.
Example, 10-8a. Find cos 37.8'.

Solution: From Table 30-8a we f i n d


cos 3 7 O 2 .799
cos 38' 2 -788
2I
=
.a OP x = -8.8

cos 37.8' 2 .790.


It is Important to observe that cos 0 decreases as 8
increases.
580
Table la-&.
De - Tan- De- Tan-
grees Radians Sine Cosine g e ? t grees Radians Slne Cosine gent
0 ,000 0.000 0.000
1 .017 .c18 .018 46
2 .035 .035 .035 &7
3 .052 .052 .052 45
4 .070 .070 .070 49
5 -087 .o87 .088 50
6 -105 .lo5 ,105 51
7 .I22 -122
8 -140 .139
9 -157 .156
10 175 .I74
11 ,192 .lgl
12 ,209 .208
13 ,227 -225
14 ,244 .242
15 .262 .259

,070 14.30
.a52 19 ,Oa
,035 28.64
.018 57.29
,000 unde-
fined
Example 10-8b. If s i n 8 = ,600, what is Q?
Solution: From Table 13-$a we find
sin 36'
s i n 37
0
-
2 .588
'b
,602

Thus, if s i n 0 = -600, then 0 2 36.9'. Prom Corcllary 10-6b


wc know that t h e r e is another primary angle Q such t h a t
s i n 8 = - 6 0 0 . A l i t t l e l a t e r we shall show how t o f i n d t h i s
second primary angle. All o t h e r solutions of the equatlon
sin 8 = ,600 can be obtained from the two primary solutions by
t h e method explained at the end of S e c t i o n 10-6.
We s h a l l now explain how the trigonometric functions o f any
angle can be obtained from a table which gives t h e trigonometric
functions of angles from 0' t o 90'. We observe f i r s t that the
functions of any angle a r e equal t o t h e functions of a co-terminal
angle which lies between 0' and 360'. For example, sin 473'
= sin 113'. The problem is thus reduced to f i n d i n g the functions
of a l l angles between 0 O and 360'. If 0 is one of the s p e c l a l
angles oO, go0, 180°, and 270°, i t ' s f u n c t i o n s can be o b t a i n -
ed from Table lo-7a. F o r each angle 8, where '0 2 8 < 360'
and 0 is not one of the s p e c i a l angles, an angle OR c a l l e d t h e
reference angle o f 0 is defined by Table 10-8b.
Table 10-8b, The reference angle of 8

1 I The r e f e r e n c e anglel
Figure 10-8a. Reference angles f o r the angle 0.

F i g u r e lo-8a shows the reference angle QR f o r angles Q in


quadrants XI, 111, and IV. The c i r c l e s in t h i s figure are the
standard unit circles. Let ~ ( x ~ , ybe~ )the point where the
terminal side of the reference angle BR intersects the unit
circle, and let PI be the corresponding p o i n t on the terminal
s i d e of 8 . The trlangle OP'D' is congruent to the triangle
OPD in every case. Thus, the coordinates of P 1 are

-
Theorem lo-8a. Let 0 be any angle such that '0 ( 0 < 360' -
and such t h a t 8 La not an Integral multiple of go0, and l e t
QR be the reference angle of 8 . Then
sin 4 = +-
+
sin OR Cot 8 - +--I-
c o t BR

coa 0 = - cos OR set Q = - sec QR

tan O = t a n OR cso e = ? csc oR


-
Proof: Examine Figure 10-8a. For every angle 0 o r the
kind s p e c i f i e d In the theorem, sin Q = y 0 or s i n B = -y 0 '
But sin QB - y o . Thus, e i t h e r s l n 8 = s i n 8 or s l n 8
R
= - s i n QR.
The o t h e r statements in the conclusion of the theorem
can be established in the same way.
Table 10-8c shows the signs of the six trigonometric functions
f o r angles in the f o u r quadrants. The r e s u l t s given in this table
follow from the definitions in S e c t i o n 10-5.

Table 10-8c. S i g n s of the Trigonometric Functions


I
Trigonometric
Functions

sin
COB
tan
cot
sec

1 CSC f + I -
I
Theorem 10-8a and Table 10-8c enable us to f i n d the six
trigonometric functions of any angle from tables f o r a l l angles
from '0 to 90'. The method wlll be explained by means of
examples .
-
Example 10-80. Find a l n 603' by using Table 10-8a.
Solution: The angles 603'
and 243' are co-terminal; hence,
sin 603' = s i n 243'. By Table
10-8b, the reference angle of
243' is 63'. T h u s , by Theorem
10-8a, sin 243' = 1 s i n 63'.
From Table 10-8a, s i n 63' 2 -891.
Since 243' is an angle in the
t h i r d quadrant, s i n 243' Is
negative by Table 10-8c. Thus,
s i n 603' = sin 2430 = - a i n 63'
2 - .891. The e n t i r e solution,
except f o r f i n d i n g sin 63' in
Figure 10-8b.
the table of s i n e s , should be
Graph of 603'
geometrically obvious from and its reference angle.
Figure 10-8b.
F,xample 10-8d. Find t a n 328',
Solution: The reference angle is 32O, and t h e tangent is
negative i n t h e f o u r t h quadrant. Thus, t a n 328' = - t a n 32'
* - ,625. The reader should draw a f i g u r e .
w

Example 10-8e. Find 5~ ,


coa T

Solution: S i n c e is the radian measure of the angle, t h e


reference angle is radiana. A l s o , 5~ is an angle in t h e
second quadrant, where the coaine Is negative. T h u s , cos 5a
= - cos 2 - .866. The reader should draw a figure.
Example 10-8f. Find s i n 1046'.
Solution: Since 1046' = 2(36a0) + 326', the angles 1046'
and 326' are co-terminal. Thus, s i n 1046' = s i n 326'. The
reference angle of 326' 1s 34*, and s i n 326' is negative
since the angle is i n the f o u r t h quadrant. Thus, s i n 1046'
= s i n 326' = - s i n 3 4 ' 2 - .559 .

[ s e c . 30-81
Example 10-8g. Find cos(-150~).
S o l u t i o n : The angles -150' and 210' are co-terminal;
hence, cos (-150') = coa 210.
0
The reference angle f o r 220' is
30°, and t h e cosine is negative in the third quadrant. Thus,
cos(-150~) = cos 210' = - cos 3 00 & - .866.y .
Witho u r t a b l e s a v a i l a b l e we are now equipped to discuss some
examples o f a simple and important application of t h e trigonometric
functions -
the indirect measurement of distances by triangulation.
Example 10-8h. A t a point 439 feet from the base o f a
building the angle between t h e horizontal and the l i n e t o t h e top
of t h e b u i l d i n g (angle of elevation) is .
'
1
3 What is the h e i g h t
of t h e building?
Solution: In the r i g h t
.angle ABC we have F = 90'
a = 31' and b = 439 feet.
In t h i s drawing we seek the
h e i g h t a of the building.
According to the formula f o r the
tangent of an a c u t e angle o f a
right triangle we have
tan 31o side o osite
= ~ESZZTZ=T~~
0 -
our Table 10-8a gives
t a n 3 1 * .601 . be 439 fi.
Combining these two equations
we have Figure 10-8c.

Therefore a 2 439( .601) 2 264,


s o thzt building I s approximately
264 f e e t h i g h .

Esec. 10-81
-
Example 10-81. To measure t h e wldth of a river a stake was
d r i v e n i n t o the ground on the s o u t h bank directly s o u t h of a tree
on t h e o p p o s i t e bank. From a p o i n t 100 ft. due west o r the
stake,ths t r e e was sighted and the angle between t h e l i n e of sight
and t h e east-west lLne measured, What is the width of t h e r i v e r
if t h i s angle was 60°?
Solution: The p o i n t from
which the tree was sighted was
taken due west of the stake so
t h a t the angle RST (Figure
10-8d) would be a right angle.
From the formula for the tangent
of an acute angle in a right
t r i a n g l e ( S e c t i o n 10-5) and
Table 10-8a we have Ti;6 = tan 60'

= where r La t h e required
width of t h e r l v e r .
y = 1 0 0 n 2 173 . Figure 10-8d.
The r i v e r is apprcximately 173
.
f e e t wfde
Example 10-8j. A t t h e instant when the moon is exactly a t
h a l f phase t h e angle between the line f r o m the earth t o the moon
and the line from the earth to the sun is between 89' and 9 .
'
0
Show that t h e distance from t h e earth to t h e sun is at l e a s t 50
times the distance from t h e e a r t h t o t h e moon.
S o l u t i o n : From Figure 10-8e
we s e e t h a t if the moon is exactly
a t half-phase the angle SME is a
right angle. Since angle SEM =p
and $9' < / < go0, we have
0' < a < lo. Then the d i s t a n c e
m of the earth t o t h e sun and
t h e d i s t z n c e s from the e a r t h S M
to t h e moon are related t h u s
Figure 10-8e.
s i n a = ms-
and from Table 10-8a
sina: < s i n 10 * .018
s o that

Thus the distance from earth to the


sun is at least 50 times the d i s -
tance from e a r t h to moon.
The e s s e n t i a l s t e p i n these examples is the discovery and
construction of a right t r i a n g l e one of t h e s i d e s of which is t h e
l e n g t h to be measured. In S e c t i o m l O and 13. we will learn some
f u r t h e r theorems about the trigonometric functions which w i l l
permit us t o use more general triangles in a similar way. Before
t h i s we m u s t discuss t h e trigonometric functions in more d e t a i l .
I. What is the reference angle of each of the following?

(a) 150° (f) -9a0


(b) 260' ( g ) -235'

(c) 7a radians (h) radians

(d) 308' (i) 9 radians


(d radians

2. Express the following i n terms of the same functfon of the


reference angle.
(a) s i n 165' (k) 31n(-195~)

(b) t a n 190' (1) sin 305'


(c) cos 212O (m) t a n ( -378')

(f) tan(-lr) (PI tan(-%)

(g) sin 340' (q) s i n 335'


(h) s i n 98' (r) C O B ( - 3 ~ )
(I) t a n 462' (3) sin 600'

( j) eos (-160')
338
(t) tan
3. A wire 35 f e e t long is stretched from level ground t o the
t o p of a pole 25 f e e t high. Find the angle between the
pole and t h e w i r e ,

4. A k i t e string 200 yards long makes an angle of 70' with


the ground. How high is the k i t e ?
5. From the t o p of a rock which rises vertically 326 f e e t o u t
o f the water, the angle between t h e line of s i g h t of a boat
and t h e horizontal (angle of d e p r e s s i o n ) Ls 2h0. F i n d the
d i s t a n c e of the b o a t from the base of the rock.
6. The edge oS the Great Pyramid is 609 f e e t and makes an
angle o f 5 2' with t h e h o r i z o n t a l p l a n e . What is t h e h e i g h t
o f the pyramid?

7. A gun G shoots a t T at a range of 5400 y a r d s , and t h e


s h o t hits a t S s o that angle TGS = 3'. Assume that angle
GTS = 90'. How f a r f r o m T is S?
8. Flnd t h e radius of a regular decagon, each s i d e o f which is
8 inches.
9. From a mountain t o p 4000
f e e t above a Port t h e angle
of depression o f the f o r t
I s 1 7'. F i n d the airline
distance from t h e mountain F
t o p t o the f o r t .

10. At a point185 f e e t f r o m t h e base of a t r e e , the angle of


elevation of the t o p is 55'. Haw tall is the tree?
11. From a n observation p o i n t t h e angles o f depression of two
b o a t s in line with this p o i n t a r e 18' and 28'. F i n d the
d i s t a n c e between the two boats if t h e p o i n t o f observation
Is 4000 feet h i g h .
12. A b u i l d i n g stands on a horizontal plane. The angle of
elevation at a c e r t a i n p o i n t on the plane is 30' and at a
p o i n t 100 f e e t nearer the b u l l d i n g it is 4
5'. How high is
the b u i l d i n g ?
13. Find the angles of intersection of the diagonals of a r e c t -
angle 8.3 f e e t wide and 13.6 feet long.

Csec. 10-81
14. The area of an e q u i l a t e r a l A is 300 square inches. What
is the area o f the i n s c r i b e d clrcle?
15. A c i r c l e is divided i n t o 7 equal parts. Find the length o f
a l l p o s s i b l e chords whose end-points are these d i v i s i o n p o i n t s
if the radius of the c i r c l e is 7 inches,
16. The minute hand of a clock is 9 inches long. A t 7 minutes
a f t e r 3 t h e l i n e j o i n i n g the ends of the hands is per-
p e n d i c u l a r to t h e hour hand. How long is t h e hour hand?
17. If t h e hands of a c l o c k are 7.4 inches and 4.8 inches,
a t what time between 2 ; 0 0 and 2: 10 is the lfne joining

p
',,
t h e ends of the hands perpendicular to t h e hour hand?
18. Given A ABC w i t h A , 6 and
7' know.. L e t h be the
altitude t o a.
a P 7
Prove: h = cot,& 4- cot f 0

19. A chord 6 inches long subtends a certain angle at the c e n t e r


of a c i r c l e whose radius is 5 inches. Find the length of
the c h o r d which subtends an angle twice as l a r g e .
20.
base
-
The area of trapezoid ABCD is 4800 square feet. Lower
i s 150 f e e t long, s i d e AD is 47 f e e t long
and angle A I s 57', Find t h e other base.

10-9. Graphs of the Trigonometric Functions.


We have found It h e l p f u l in the past to draw t h e graphs of the
f u n c t i o n s under study. Recall t h a t t h e graph o f y = f ( x )
c o n s l s t s o f the s e t of p o i n t s (x,y) in t h e coordinate plane such
t h a t y = f( x ). But it is c l e a r l y impossible t o draw t h e graphs
of those trigonometric f'unctions whose domains are the s e t of all
signed angles, because we have no scheme f o r exhibiting graphically
the s e t of all pairs ( ( A , P , B ) , s l n ( ~ ,o)) .
~ , Notice that t h e f i r s t
e l e m e n t i n t h i s p a i r is a signed angle (A,P,B) , which is a
geometric o b j e c t - n o t a real number.
We now r e c a l l t h a t a second set of trigonometric f u n c t i o n s
was defined in S e c t i o n 10-5. The domains of these functions a r e
the s e t of all real numbers, and it is thus possible to draw their
graphs In the usual way. For example, the graph of y = sin x
consists of all points (x,y) , where y = sln(O,~,x) and x is
considered to be the radian measure of t h e angle (o,x,x).
S i m i l a r statements h o l d f o r the graphs of the o t h e r five t r i g o n o -
metric functions of real numbers.
It is important to observe that the f o l l o w i n g statements are
true every_ E.
(i) sin(x + 21r) = s i n x
(11) cos(x + m) = cos X

(iii) s e c ( x + 2w) = sec x


+
(iv) c s c ( x 2?r) = csc x
(v) tan(x + T ) = tan x
( v i ) cot(x + a) = cot x
Statements (I) to (iv) follow from the fact t h a t , if x is
the radian measure of an angle, x + 2a I s t h e radian measure of
a coterminal a n g l e . Statements (v) and (vi) follow from the facts
t h a t t h e angles having radian measures of x and x + a
respectively, have the same r e f e r e n c e angle and thelr tangents
(or cotangents) have the same algebraic sign.
If for a function f ( x ) there exlsts a number p such t h a t
( v i i ) f (x + p) = f (x)
--
for all the function f Is said to be periodic. If p is
the smallest positive number f o r which (vil) is true, t h e function
is s a l d to be p e r i o d i c with period e. Since 2 r is t h e smallest
p o s l t i v e number f o r which statements ( i ) to (iv) are t r u e for a l l
x, we conclude on the basis of o u r definition that the functions
s i n , cos , sec and c s c a r e all perloaic w i t h period 27~-
Similarly, t h e t a n and c o t functions are periodic with period
'IT.
Notice that t h e curve in Figure 10-gb is congruent to the
graph o f y = s i n x in Figure 10-ga and is o b t a i n a b l e by shffting
that curve units to the left.
F i g u r e 10-9c shows the graph of y = tan x.

Notice that it is composed of congruent p i e c e s which have t h e


v e r t i c a l lines
+
x = -
T
~9 X = '
- 311
7 as asymptotes.
Exercises 10-9

1. Draw the graphs o f each of the following s e t s of equations


using a s i n g l e s e t of axes.

(a) y = sin x (g) y = sin x


y = s i n 2x y = 2 sin x
(b) y = cosx y = 3 sin x
y = 2 cos x y =
1 sin x

(c) y = tan x (h) y = sln x


1
y = tan ~x y = sin 2x
y = s i n 3x
(d) y = sin 1 p 1
y = sin p
y = cos
+x
( ) y = sin x
(e) y = sec x
y = csc x

(f) y = sin x (J) Y = sinb +$I


y = cos x
y = cos ( x - 5)

---
10-10. The Law of Cosines.
One of the most famours of all mathematical theorems is the
Theorem of Pythagoras, which s t a t e s t h a t in a r i g h t triangle ABC,
2
c 2 = a 2 + b .

Figure 10-10a.
It is plausible that if 7' is leas than a right angle, then c 2
18 1.58 than a2 + b2; and if $ is greater than a r i g h t angle,
then c2 is greater than a2 + b 2 . Our next theorem covers a l l
three p v s s i b i l i t i e a in a sfngle formula. 1% refers to any triangle
ABC and uses the n o t a t i o n of Figure 10-lob.

Figure 10-lob .
Theorem 10-lOa. h he Law of Cosines.) In triangle ABC

c2 = a* + b2 - 2ab cos f
b2 = a2 + c2 - 2ac c o s ~
2
a2 = b + c2 - 2bc cos a

-
Proof: We introduce a co- B(x,y)
ordinate syatem In such a way
that f I s in standard position.
In t h i s coorcdLnate system, C
has coordinates (0,0), A has
c o o r d i n a t e s (b,0), and B has
coordinates which we denote by
(x,y) . ( s e e F i g u r e 10-10c).
UsZng the distance fomula we have
c2 = (x - b) 2 + y2 = x2 + p2 + b2 - 2xb and
2
a 2 = x 2 + y .
It follows that
X
We also know f r o m Theorem 10-5a that cos f =
' which

is a. Therefore, x = a cos f. Substituting a cos f f o r x


gives 1 x 1 c2 = a2 + b 2 - 2ab cos f .
The o t h e r t w o r e l a t i o n s In the theorem can be proved s i m i l a r l y .
Example 10-10a: In triangle ABC, a = 10, b = 7 , and
f = 32'. Find c.

Solution: By the law of cosines


c 2 = 100 + 49 - 140 cos 32'.
Uslng Table 10-8a cos 32' 2 ,848 and

therefore, c' 2 149 - 1 4 0 ( ,848)


30
Hence, c 2 5.48
Example 10-lab! In triangle ABC, a = 10, b = 7 , and
c = 12, Find a .
Solution: By law of cosines
a" b2 + c2 - 2be cos 6.
Hence,

Thus, a 2 56' to the nearest degree.


Suppose triangle ABC is a right triangle w l t h right angle
at C, i . e . , f = 90'. In t h i a case, c is the hypotenuse of t h e
right t r i a n g l e , and s i n c e coa 9 ' = 0, the law of cosines becomes
0
c2 = a2 + b2. But t h i a is J u s t t h e Pythagorean Theorem. There-
fore the law of coslnes can be viewed as the generalization of
the Pythagorean Theorem t o arbitrary triangles. However, we do
n o t have a new w o o f of the Pythagorean Theorem here, because our
proof of the law of cosines depends on the distance formula which
was established on t h e basis of the Pythagorean Theorem !
It 3s worth n o t i n g , though, t h a t t h e law of cosines can be
used t o prove t h e converse of the Pythagorean Theorem. If, In
'
t r i a n g l e ABC we know that c 2 = a + b2, then we must show t h a t
2
1' = 90'.By t h e law of cosines c = a2 + b2 - 2ab cas ? and,
combining thls w i t h c 2 = a2 + b 2 , we o b t a i n cos 7 = 0. We h o w
that 0 < f < 180°, and t h e only angle in t h i s range whose cosine
is z e r o is 90'. Therefore, f' = 90° as was to be proved.

Exercises 10-10 -
1. Use t h e law o f cosines to a o l v e t h e following:
(a)a = 60°, b = 10.0, c = 3 .o, find a.
(b) a = 2 1 , b = 8, c = 10, find/.

2. Find the largest angle of a t r i a n g l e having s i d e s 6, 8,


and 12.
3. In the following problems find the length of t h e side n o t
given.
( a ) b = 8, c = 12, Q = 25O
0
(b) a = 2.5, b = 13, f' = 140

(c) a = 60, c = 30, 4 = 40°


4. Find all three angles of t h e t r i a n g l e i n each of the following:

(a) a = 15, b = 16, c = 17


(b) a = 24, b = 22, c = 25

5. Two sides and the included angle of a parallelogram are 12


inches, 20 inches and 100' respectively. Find the
length of the longer diagonal.

[ a e c . 10-10)
10-11. The Law of Sines.
The following theorem expresses t h e area of a triangle in
terms of i t s s i d e s and angles,
Theorem 1 0 - l l a . In triangle ABC
area of t r i a n g l e ABC = $ab sin 7"
= +c sin a

Proof: Introduce a coordinate system so t h a t f is in


standard position, (see Figure 10-lla) .

, . '
Figure 10-lla.
Then by Theorem 10-5a
sin f = Y = X,
a
J-
but y I s also equal to h, the altitude of the triangle, so
h = a sinf. Since the base of the t r i a n g l e is b , i t s area is
+b sinf.
The other formulas follow similarly,
Theorem 1 0 - l l b . ( ~ a wof Sines), In triangle ABC,
a in u sln f
a

-
Proof: According to Theorem 10-lla we have
1b
p 1
sin/ = $c s l n a = p1 c sin@.

If re divide each member of these equations by we


obtain

Example 10-lla. If, in triangle ABC, a = lo,/ = 42


0
,
f r 51°, find b.
Solution: Since a +/ +f = 180' we have a = 87'.
By the law of sines
-s=i n a sinP
a 7'
a sin/ 10 sin 42' 2 6.69 2 b m 7.
b f
8 na ~ s i =n 87' 773
Example 10-llb. Find the area of triangle ABC if a = LO,
b = 7, f = 68'.

area of triangle ABC -


Solution: According to the formula In Theorem 10-lla, the
b s i n f = 35 sin 68' Z 3 5 ( . 9 2 7 ) 2 32.4 .
=ample 10-llc. A r e there any triangles ABC such t h a t
b = 5 , c = 10, m d r ' = 22'1

[aec* UNlVERSlTY OF CHIUGQ


LABORATORY SCHOOL
HIGH SCHOOL UBRARY.
Solution: Before a t t e m p t i n g t o s o l v e Example 1 0 - l l c l e t us
t r y t o construct a triangle ABC geometrically, given b , c,
a n d y . Lay off side AC o f length b, and c o n s t r u c t angle f
at Z . Now with A as c e n t e r strike an a r c of radius c . There
are a number of p o s s i b i l i t i e s depending on b, c, and 7^ which
a r e i l l u s t r a t e d in Figure L O - l l b .

(i1 (ii) ( iii)

Figure 10-1lb .

[ a e c . 10-111
In case (i) there is no trf angle;
in case (11) there is one triangle;
i n case (iii) there are two triangles;
in case (iv) there is one triangle;
in case ( v ) there is no triangle;
in case (vl) there is one t r i a n g l e .
Thus to solve Example 10-llc, we a t t e m p t to find/ keeping
in mlnd that t h e r e may be z-ero, one, or two s o l u t i o n s . If such a
triangle exists, then by t h e law of sines

or sin/ =
1
7 sin 22
0 .u
.187 .
Recall t h a t sin/ fs posltive in the second quadrant and if

sin/ -
4 = 180' - Q where 0 < Q < go0, then sin/ = s i n 0 . Thus from
'U
-187 we conclude t h a t @ 2 11' or B 2 16g0 to t h e
nearest degree. A r e b o t h of these values o f # p o s s i b l e ? If
N = 16g0, then f l + p = 191' which is impossible. Why?
Therefore, there is one triangle w i t h t h e given data. We are in
case (iv) .
Example 1 0 - l l d . Are there any'triangles ABC with b = 10,
c = 15, and f l = 105'?

Solution: W e attempt to find @ . If there is such a tri-


angle, we have, from the law of sines,

But s i n f = s i n 105' = sin(180~ - 75') = s i n 750 * .966. Hence,


s i n g 2 $( .966) 2 ,644 and this implies/ 2 40' or/ 2 140'.
C l e a r l y P can n o t be 140' and there is one t r i a n g l e w i t h the
given data. This is an example of case ( v f ) ,
Example 10-lle. Are t h e r e any t r i a n g l e s ABC such that
b = 50, c = 10, and = 2
'
?
Solution: We attempt to f i n d / . If there is such a tri-
angle we have, from the law of sines,
sin/ s i n 22'
- g T = l o ,

But we know that the sine never exceeds one, and t h e r e f o r e o u r


assumption t h a t a triangle with the given data exists leads to a
contradiction. Thus there a r e no such t r i a n g l e s . This is an
illustration of case (I).

Exercises 10-11 -
1. Use t h e law of e i n e s to a o l v e t h e following:
( a ) p = 68', 7"' = 30°, c = 32.0, find a

(b) a = 45O, f = 60°, b = 20.0, find c

(c) a = 26O, y = 4 3o , c = 21.3, find b


(d) y = 126', d = 33O, b = 3.71, find a
(e) p = 113.2', a = 46', c=17.5, find b
(f) a = 68.5', a. = 103.z0, c = 51.3, find a
2. Solve completely the followlng triangles :
0
(a) a = 27*, f = 4 2 , b = 24

(b) f = 29.s0, fl = 48.5


0
, c = 8.4
(c) a = 132', &
, = 24O, a = 135
(d) a=5.8, = 5
0') / = 73'

(e) O[ = lop0, 4 = 41°, c = 52.8


(f) a = 48.5', f = 67.8', b = 28.7
3, In each of the following, without s o l v i n g , determine t h e
number of solutions.

4. In the following, determine number of solutions and solve


completely.
(a)u = 6g0, a = 5.2, b - 6.2

(b)p = 13.3', b = 80, a=193


(c) a = 142', a = 8.4, b = 3.7
(d) = 59.6'; a = 39, c = 37
( e )p = 5.8'. c = 98.3, a = 23.2
5. Find the area of the triangle in each of the following.
(a) b = 12, c=14, = 42'
0
(b) a = 8.6, b = 7.9, Jf = 67
(c) a = 14.1, c = 27.4, p = 112'
(a) c = 5.5, b = 8.0, a = 103.5'
6. One diagonal of a parallelogram is 24.8 and it makes an
angle of 42.3' and 27.6' w i t h the sides. Find the sides,
7. Twopoints o n a s i d e o f a m a d a r e 30 feet
A and B
apart. A p o l n t C across the road is located so t h a t angle
CAB I s 7
0' and angle Al3C Is 8 0'. How wide is the
road?
d
8. TWO observers, one at
A and the o t h e r at
B, were 1760 yards
apart s h ofthey
the f l awhen an observed
enemy A
1760 yards B
gun at C. If angle A
was 38' and angle B
was 61°, how f a r was
each observer from t h e
e n e w gun?
9. From the top of a c l i f f , the angles of depression of two
successive mileposts on a horizontal road m i n g due north
are 74' and 2 s 0 , r e s p e c t i v e l y . Find t h e elevation of
the c l i f f above the road.
10. A tower at the top of an embankment c a s t s a shadow 125 f e e t
long, s t r a i g h t down one s i d e , when the angle of elevation of
the sun is 48'. If the s i d e o f the embankment is inclined
3 3 O from the horizontal, f i n d t h e h e i g h t of the tower.
11. A triangular lot has frontages 90 f e e t and 130 f e e t on
two streets whlch intersect a t an angle of 82'. Find the
area of the l o t .
12. The lengths of two s i d e s of a t r i a n g u l a r lot are 240 feet
and 300 feet and t h e angle opposite the longer s i d e Is
equal to 75'. Find t h e third s i d e and t h e area.
10-12. -
The Addition Formulas.
Angle measures and trlgonometrlc functions have a common
feature, namely, they b o t h are schemes f o r a t t a c h i n g numbers t o
angles. One important d i f f e r e n c e between them has to do w i t h
addition of angles. If a and are any angles, the measure
of t h e i r sum +,g is the same as the sum of the measures of a
and o f @ . The corresponding statement is n o t t r u e for trigono-
metric functions , For instanc e s i n ( 30' + 60') -
1 and

s i n 30' + s i n 60' = 21 + 7, which does n o t equal 1. In this


section we derive the correct expression for sln(a +,&) and
related expressions, We need a preliminary theorem.
Note: In what f o l l o w s t h e expression (eos or ) and (sin a)'
2
a r e w r i t t e n as cos2 a and s i n ol ( i n s t e a d o f as cos a
2
*
and
s i n a 2 which could mean c o s ( a ) * and s i n ( t ) .)
Theorem 10-12a. Let C be a c i r c l e of radius 1, l e t f
b e any angle whose vertex is t h e center of C, and l e t P and Q
be the respective intersection of the initial and terminal sides
of f with C. Then

-
Proof: Introduce a coordinate system in which the initial
side of f is the p o s i t i v e x-axis (see Figure 10-12a),

si nfl
Figure 10-12a.
Length of the chord PQ.

[aec. 10-123
Then the coordinates of P are ( 10 and those of Q are
( c o s f ,sin?"). The dlstance formula gives
I P Q2~ = (cos - 112 + (sin F )*
IPQ~ 2 = 1 - 2 cosy + coa2/+ sin2/
o r since cos
2
7'" + s i n2 P = 1,
2
~PQI = 2 - 2 cost".

Theorem 10-1213. For a l l angles a andp


-
cos ( p c~ ) = c o s p cos a + sin/ s i n a.
-
Proof: We first u s e Theorem 10-1Pa to evaluate 1 P Q I ~ ,
o b t a i n i n g ( s e e Figure 10-12b) lp012 = 2 - 2 c o s ( p - a: ) . We
then re-evaluate ~ P Q [ ~ using , the distance formula. We have

Figure 10-12b. The difference of/Q and ol .


2 = -
IPQJ (COB@ C O S ~ ) ~ ( +s i n p - sins) 2
= cm.5 - 2 cos (I toss + cos& + sin2/
-2 sin cC sin^ + sin2 a
2 2 2 2
~ s i n @ + cos a + s i n a
= c o s +

- 2 cos ol c o s p - 2 sin a sin/ .


Since toss + sin2/ = cos
2
a + sin2 a = 1, we have
*
By equating t h i s expression for ~ F Q I w i t h the one g i v e n at t h e
beginning o f t h i s proof, we have

Example 10-12a. Find cos 1


5
'
.
Solution: cos 15' = cos(45' - 30')
= cos 45' cos 30' -k s i n 45' s i n 30O

We next d e r i v e similar formulas f o r s i n ( / - a ) , cos [a +/ ),


s - ~ n ( +p
a ). F i r s t we need some preliminary theorems.
Theorem 10-12c. For a l l angles a
ir
c o s ( a - T) = s i n g ,

Proof: By Theorem 10-12b


cos(a - g) = cos a
T
cos
-rr
+ T
s i n a sln g.

Since cos P = 0 and sin a = 1, it follows t h a t


7T =
cos ( a- F) s i n a.

Since t h i s relation holds f o r any angle a , we can use it f o r


7r
a- T itself. It then reads

The l e f t hand s i d e of this equation is c o s ( a T), which equals -


COB a cos a + s i n a sln r. S i n c e cos a = -1 and s i n N = 0 , we
conclude that
Theorem 10-12d. For a l l angles a andp
sin(P-cL ) = s i n @ cosQ - cos@ s i n a .

Proof: By Theorem 10-12c, we have

sin(p-a) = cos ((@-a) - 5)


= cos $) - a).
((p
Using Theorem 10-12b, we can write
COS((P- $1 -(L ) = COS(&- $1 ~ C O C[
S + s i n ( @ - $1 slna .
We substitute sin,& for c o s ( p - q and -cos@ for sin(p- $)
2
in t h i s l a s t r e l a t i o n to obtain
s m ( p -a:) = s i n p cosa - cosp sing .
Example 10-22b. Find sln 15'.
Solution: sin 15' = sin(45' - 3
0
'
)
= s i n 45' cos 3
0' - cos 45' s i n 30'

Theorem 10-12e. F o r all angles a


COS(- a) = cos a
s i n ( - a) = -sin a .
Proof: c o s ( - a ) = cos(0 -a)
= cos '0 cos a + s i n 0' sina .
Since cos 0' = 1, sin 0 = 0, we conclude t h a t
C O S ( - a=) cos a .
Since s i n ( - a ) = s i n ( 0 -a )
= (sin 0 )(cos a) - (cos 0 ) ( s i n a )
we have sin(- a ) = O * c u s a - lmsin a
Theorem 10-l2f. F o r a l l angles U andP
cos(p+a) = cosp cosa - sin/3sin a
s i n ( p + a ) = s i n @ cos a c c o s / sin a .
* proof: c o s (P +a) = cos(p- (-a 1)
= c o s p cos(-d ) + sin,& s i n ( - a )
= cos/ cosa - sin/ sin a .

= s i n & cos a + cosfl sin Q .


Example 10-1Pc. Find cos 75' and sin 7
5
'
.
Solution: c o s 75' +~30')
= cos(~-15

= cos 45' cos 30' - s l n 4 5 O s i n 30'


&,A & 1
= 2 - 7 - 7 2

sin 75' = sin(45' + )'03


= sin 45' cos 30' + sin 30' cos 45'

Notice t h a t s i n 75' = c o s 15' and cos 75' = sin 1


5
'
. This
i l l u s t r a t e s Theorem 10-l2c.
Theorem 10-l2g. For all angles ff

sin 2 d = 2 s i n a c o s a
cos 2 a = cos 2 a - s i n2 a .
-
Proof: s i n 2 a: = s i n ( a +a)
= s i n a cos a + sina cosa
= 2 s i n a ; cos a .
= cos a cos a - slna sina:
= cos 2 a - s i n2 a .
Swnmary Formulas

cos(d + p )= cos d c o s a - sina sinF


s i n ( a + p )= s l n a c o s d + cos a: sin,@
c o s ( a ~-@) = cos a c o a 4 -t sin a s l n H
s i n ( a -@) = sin COB@ - cos a s i n
COB( a - F)
K
= sina

sin 2 a = 2 sins c o s a

cos 2 a: = cos
2a - s i n2 a

Exercises 10-12

1. Let be an angle in the t h i r d quadrant whose cosine I s


--5 and @ be an angle in second quadrant whose tangent is

-d. Find

(a) sinb w) (d) cask -S 1


(b) cos(~+/3) (e) tanCa+&)
(c) s i n k -/1
2. Use the addition formulas t o compute the exact value of the
following:

(a) sin 7 5 O (dl sin 15'


(b) cos 7 5 O (e) cos 105'

(c 1 t a n 75' (f) s i n 195'


3. U s e the addition formulas t o find t h e exact v a l u e of t h e
following:
(a) COS(T - ); ( ) COS(T + 5)

4. Show that cos (a. - 5) = sln u for

(a)a = 45' (a) a =


5
3iT
(b)a = 210' (4 u =

4
5. Show that sin(a-$) = -cost[ for

6. Prove cos 2 a = 2 cos


2a - 1, and deduce f r o m t h i s equation
t h e half angle formula cos

7, Prove cos 2a: = 1 - 2 s i n z a , and deduce from this equation


the h a l f angle formula sin
= 2 $
- .

[ sec . 10-121
8. Compute t h e e x a c t value of s i n 2 a , cos 2 a and tan 2a
f o r t h e following:
(a) cos If = E in quadrant I
5'
(b) tana= in quadrant 111
7'
2
(c) sins= 3, Q in quadrant I1

(d) cos o: = T3 , a I n quadrant IV

9. Compute the exact value of sin 2, cos T, d and tan Ot


for
the following:
I
(a) cos d = 2, a in quadrant IV
(b) sina =-
5' a in quadrant 111

(c) cos a = -m,


5 OC in quadrant II
12
(d) sina = TJ, a in quadrant I

10. Use t h e formulas from Problems 6 and 7 t o compute the e x a c t


value of
(a) cos 1
5' (c) sin 11.25'
(b) cos 22.5' (d) s l n 7.5'

-
10-13. Identities and Equations.
Equations such as
sin 2~ = 2 sina cosd

are known as identities. They y i e l d t r u e statements no matter


what angle o r real number I s substituted f o r # . I n a slightly
g e n e r a l i z e d sense, the f o l l o w i n g equation is an i d e n t i t y .
sir, Q
tan Q = -
cos 8
Q # (2k + 1)$
s l n c e , by Theorem lO-5aY
L
ns i=nT 0= % ' tra n e .
-
This identity has one pecu1iar:ty which should be observed care-
fully; tan Q is n o t defined for 8 = (2k + 1); and sin 8

n o t defined for 8 = (2k + 1)s since cos(2k + I)$ = 0. Thus,


t h e two s i d e s o f equation 10-13a a r e equal f o r every v a l u e o f 8

f o r which t h e two sides a r e d e f l n e d , and tan 8 = is a l s o


called an identity.
The e q u a t i o n
s i n 2a = 2 s l n u
y i e l d s a true statement if a is replaced by Zna, n an i n t e g e r ,
b u t it y i e l d s a f a l s e statement f o r every o t h e r v a l u e of a . A n
e q u a t i o n o f this t y p e is called a conditional equation. We have
mathematical responsibilities toward each of these types of
equations. We shall b e asked to prove i d e n t i t l e s , that is, p r o v e
t h a t the solution s e t consists of a l l values of t h e variable.
More precisely, to prove an i d e n t i t y means to prove that the
solution set consists of all values of t h e v a r i a b l e f o r which t h e
-
two sides of t h e equation are defined. To s o l v e a conditional
equation means to find the solution s e t .
There a r e no standardized methods f o r proving identities or
s o l v i n g equations. To prove an identity o r to s o l v e an e q u a t i o n
o f t e n requires ingenuity and perseverance, and many methods must
be devised t o handle a l l the problems t h a t a r i s e . The procedures
a r e b e s t explained by a variety o f examples.
Example 10-13a. Prove the Identity
tan 2 a = 2 tan a
1 - tan%
Solution: Observe that n e i t h e r side of t h i s equation Is
defined f o r
a -IT
= T B
+ kg- , k aninteger,

f o r , on t h e l e f t , 2 a is an angle co-terminal with and t h e


tangent is undefined; on the right, tan 2 a = 1 and the denomin-
ator vanishes. We are thus asked to prove

tan 2 a =
2 tan a u #f + k.5.
I - tan2,
By the proof at the beginning of t h i s Section,
tan 2 a = sin 2 4 or # f + k.5
- 2 sin a cosa (by the formulas from
cos2a - sin2a Section 10-12)
sin a
= 2
sin
y d
( d i v i d e the numerator
and denominator by
cas2 a 1
2 tan a
- ,a # $ c k.5.
1 - tanCa
Example 10-3b. Prove the identity
tan(8 + a) = tan Q , Q # (2k + 1 1 5

[sec. 10-131
Solution: By Equation 10-13a,
t a n ( ~ + a =) * cos 8 + a , Q # (2k + 1 1 5

(by the formulas in


Section 10-12)

= tan 8 .
Example 10-13c. Prove the following Identity
sin a[
t a n CC
1 + cosa
=

Solution: The key to the solution 1s the observation t h a t

=
2 sin 9 cas 3 (by the Identities in
1 + cos2q - s1n2 5 ~ e c t i m10-12)
2 sln
E
cos a
-
2 cos2 g
u
sin -
- 2
3
4-

c.

-
= tan pQC

Example 10-13d. Prove the following i d e n t i t y :

sin a + sin,& = 2 sin a +& COB


a -@
2-
Solution: The simplest proof of this identity employs a
device. Observe that

Then by t h e addition formulas in Section 10-12

sin a = [
sln \-" +"c,%I
si?g = sin[(+ - ?+I]

Example 10-13e. F i n d all solutions of t h e following equation:


s i n x = 2 cos x.
Solution: Observe f i r s t t h a t x = (2k + 1)$ I s not a

solution of the given equation. Then cos x f Q f o r a value of


x which I s a solution of the equatton, and t h e given equation is
equivalent t o the equation
-sin
= x
COS X 2 , C O S X # O ,

or tan x = 2.

Interpolation in Table 10-8a, shows t h a t x is 1.107 radians


approximately. From Example 10-13b above, it follows that
P + 1.107 radians, or 4.249 radians, is also a s o l u t i o n . Finally,
since the trigonometric runctions are periodic with period 2a,
all solutions of the given equation are
x 2 1.107 2 2ka radians
x -
'k
4.249 + 2ka radians
where k is an i n t e g e r .
Example 10-13f. Find a l l solutfons of the following equation:

Solution: It should be observed f i r s t t h a t t h e g i v e n e q u a t i o n


is a quadratic e q u a t i o n in s i n 8 . It would be possible t o s o l v e
for s i n 8 by u s i n g the formula f o r t h e r o o t s of a q u a d r a t i c
equation, but it is s i m p l e r in the p r e s e n t c a s e t o solve by
f a c t o r i n g . The given equation is equivalent to
(2 sin Q - l)(sin 8 - 1) = 0 ,
and all solutions can be found by solving the two simpler e q u a t i o n s
2 sin Q - 1 = 0, sin 8 - 1 = 0.
The solutions of the given equation a r e thus,
T
8 = 6 + 2 k ~

8 =
5~ + 2k7r
7

where k Is an integer.
Example 10-13g. Solve t h e equation
tan x = 2x.
Solution: By scanning t h e entries in Table 10-8a, we s e e
that f o r small values of x ,
tan x < 2x,
whereas f o r large values of x,
tan x > 2x.
The change in the d i r e c t i o n o f the inequality occurs between
x = 1.152 and x = 1.169, t h a t is,
2.246 = tan 1.152 ( 2(l.l52) = 2.304,
2.356 = tan 1.169 > 2(1.169) = 2.338.
Since 2x and tan x a r e continuous, i t follows that there is a
s o l u t i o n o f the equation between x = 1.152 and x = 1.169
radians. Methods a r e g i v e n i n more advanced courses for approxl-
mating this solution to as many decimal places as may be desired.
There a r e g r a p h i c a l methods which are u s e f u l in finding the
approximate values o f the s o l u t i o n s of trigonometric equations.
The graphical solution i n t h e present case shows that t h e given
e q u a t i o n has an i n f i n i t e number of solutions. Figure 10-13a shows
the graphs of y = 2x and y = tan x . If (xO,yO) IS a po:nt of
I n t e r s e c t i o n o f the graphs o f these two equations, then

yo = tan xO,

and 2x0 = t a n x0 ' Thus xo is a s o l u t i o n of t h e given equation.

Figure 10-13a. Graphical solution of tan x = 2x.

f sec . 10-131
It is c l e a r from the figure that the line y = 2x Intersects the
graph of y = tan x in infinitely many p o i n t s . For large values
of x the intersections are almost on t h e llnes x = (W + I)$,
and x = ( W + 1)E is approximately a solution if k Is an
integer whose absolute value is large.

Exercises 10-13a

Prove the following identities:


1. tan 8 cos 8 = sin Q

3. coa 8 - 1 - sin 0
1 + sin Q - cos 0

4. ain 2 8
tan 9 = 1 + cos 2 a

6. 2 csc 2 0 = sec 8 csc 8

7. tan Q sin 2 0 = 2 s i n2 Q

8. 1 - 4 O=cas40
2 sin2Q+sin

2 coa 2 8 - sin2 @ +1 =
9. cos 0

12. sin4 Q - sin2 o c o s L ) e - 2 cos 4 Q = s i n2 Q - 2 cos2 Q

14. aec' 0 - csc2 B = (tan Q + cot ta tan 0 - cot 8)

15. tan x - tan y = sec x sec y ain(x - y)


[sec. 10-131
sin 4 8 = 4 sin 8 cos 0 cos 2 8

sin(a + @ ) sin(or -@) = singor - sln2p


cos(a cs) cos(or -a)= oos 2 a - sin2&
sin(a: +# ) + s i n ( a -#) = 2 s i n a cos B
sinla +&) - sin(a -#) = 2 cos a sin&
COS(U +#) + cos(a -/) = 2 cos a c o s ~

cos (a +a) - cos (a -6 ) = -2 sin a sinfl

3
3 sin 0 - sin .3 8 = 4 s i n 8
P r w e that none of the following is an identity by counter
example. See Sec tlon 10-7, P roblem 6 .
(a) cos (a -a)= cos a - COSF

(b) COS(E +#) = cos a + COSF


(c) sin(a -6)
= slna - sin/
(d) s i n ( u +p)= s l n a + Binp

(e) cos 2a = 2 cosa

(f) sin 20: = 2 sins

s i n ( a + P ) = tma
cos a c o s p
+ tan/

sin28 - 1 - cos 2 8
1 + cos 2 8 - sin 2 0
csc Q 1 - -cot 0 -
cot 8 csc 8 -F 1

~f A + B +c = 180°, prove
(a) sin A = sin(B + C)

(b) cos A = - c o s ( ~ + C)
Solve the following equations f o r 0 5 8 2 27~

3 t a n2 8 - 1 = 0
2
sin Q - coa28+ 1 = o
2 cos2 0 - cos 8 = 0
sec2 Q - 4 sec Q + 4 = o
3 sec Q + 2 = coa 4
4 sin3 0 - sin 0 = 0

2 s i n 0 cos Q + sin 8 = 0
fi csc2 Q + 2 csc 8 = 0

cos 2 Q = 0

4 tan2 Q - 3 sec 2 8 0
cos 2 8 - sin 8 = 0
2 cos2 0 + 2 cos 2 Q = 1

sec2 Q - 2 tan 0 = o
sin 2 0 - cos2 Q + 3 s i n2 8 = 0
cos 2 Q - cos 8 = 0

cos 2 Q cos 8 + sin 2 8 sin Q = 1


cos2 Q - sin2 8 = sin Q
25. cot 8 + 2 s i n 8 = csc Q
26. cos 8 + sin 8 = 0

27. 3 sin Q + 4 cos 8 = 0


28. Prove t h a t if k is any real number then the e q u a t i o n
s i n x = k cos x has a s o l u t i o n .
29. tanO=8

30. ~ s I n 0 = 2 0

10-14. Miscellaneous Exercises.


1. Convert each o f t h e following to radians:
Ia) o0 (h) -100'

(b) go0 (1) -lOOoO


(c) 60' (j) 7.F
12O

( d l loo0 (k -go
2
10oO
(el 390' dl 7
(f ) 100oO (m) T8(TO
r

(g) lo
2. Convert each of the following to degrees:
(a) 0 radians (h) 2 radians

(b) T radians (i) -10 radians


(c) radians ( j) 8 radlans
(d) % radians (k) y
18*radians

( e ) 108 radians v) radians

(r) 1 radian (m) 90 radians

(g) -1 radian
3. Angles a r e sometimes measured in revolutions, where 1
r e v o l u t i o n is 2~ radians, and also in m F l s where 3200
mils is IT radians. F o r each of these units, find t h e
radius o f a c i r c l e for which a u n i t angle corresponds t o a
unit dfstance on the circumference?
4. Using t h e definitions in Problem 3, convert:
( a ) 10,000 mils to revolutlons
(b) 108 degrees to m i l s
(c 10,000 m i l s to degrees
(d) 108 degrees to revolutions
(e) 10,000 degrees t o mlls

(f> .8 revolutlons to degrees


(g) 80 degrees to r e v o l u t i o n s

(h) .8 radians to degrees


(i) 80 m i l s to radians
(j) 800 m i l s t o revolutions

5. Find s i n 8 , cos 9 and tan 8 if the termlnal side of 8,


in its standard p o s i t i o n , goes through the given polnt,

(a) (-3J4) (dl (-39-2)


(b) (-290) (4 (39-5)
(c) (295)
6. Sketch in s t a n d a r d position all the angles between 0' and
360' which s a t i s f y the following conditions, and give
values of t h e other f u n c t i o n s of these angles.
4
( a ) sin 8 =
5 (c) tan Q = -
(b) cos 8 3
=
- 7

Esec. 10-141
7. Express t h e following as functions of p o s i t i v e a c u t e angles.
( a ) c o s 170' (f) c o s 305'

(b) s i n 160' (g) cos(-looO)


4
(c) ~0~(-130~) (h) s i n q i i

(d) s i n 6110~ (i) cos - * .rr

(e) tan(-45') (j) tan 3


7 r
1 1
8. ~f s i n a = 5 and s i n @ = g, ( a a n d d each a r e a c u t e
angles) find
(a) s i n ( a -I-@ ) (dl cos(a -A

(b) sin( a -& ) (e) sin 2 q


(c) COS( a + d ) (f) cos 2~

9. Find the v a l u e o f t h e following;


(a) s f n 90' -k c o s 120' + tan 225' + c o s 160'

(b) s i n 30 cos 150 - sin 60 c o s h 5

(c) s i n 330' t a n 135' - sin 225' c o s 300' tan 180'


10. S o l v e t h e f o l l o w i n g t r i a n g l e s f o r the indicated parts. Given:
(a) a = 3, b = 2, f = 60°, find c .
(b) a=5, b = 6, c = 7, find P .
(c) c = 16, p = 8 40, f= 54O, find a.
(d) c = 5& a = 450 , a = 6, find b.
(e) a = 20, b = 21, f= 43'351, find c.

(f) b=5, a = 750 , @ = 30°, find c and a.


(g) a = 60' a = 8 6 , c = 15, find P.
(h) b=15, c = 2, a = 36O, find Area.

(i) a = 12, b = 35, c = 37, find f .


(j) a = 21, b = 17, c = 10, f i n d Area.
1 1 Prove t h a t tan(-8) = - tan Q

-
PROVE
7T
12. c ~ s - ( 0)
~ = sin Q

14, c o s 0 cos 2 8 - s i n 8 sin 2 8 = cos 3 8

15. cns 2 0 cos 0 -t sin 2 Q s i n 8 = cos Q


16. 2 cos2$- cos 0 = 1

cos 2x
21. cosx+sinx=
cos x sin x-
F i n d all primary angles which are solutions of t h e f o l l o w i n g
equations.
22. sin x - tan x = 0
2
23. 1 -sin x = c o s x
24. cos x =
1 - cos x
T
25. sin 2 8 - sin Q = O
26. cos 2 8 = 2 - 2 cos2 8
V

28. 2 c o s 2 2 @ - E S I ~ ~ ~ Q - J .

29. 2 cos2 Q - sin 8 - 1 = 0


30. 1 - cos Q = ,in g
sir, Q

31. c o t 2 8 t csc 0 = 1
32. Let a and b be any non-zero r e a l numbers and let €I be
any a n g l e , prove t h a t there l a an angle such that

a cos + b s i n B = J-cos(~ -a)-


2~
33. In a t r i a n g l e , one angle is 36' ahd another is 5- radians.
F i n d t h e t h i r d angle in radians.

34. Through how many radians does the mlnute hand o f a c l o c k


r e v o l v e in 4 0 minutes.
35.
b -
Find the t h r e e angles of a triangle
300, and c = 400.
ABC, given a = 200,

36. Find t h e remaining p a r t s of t h e triangle ABC, 'given


b -128, c = 145 a n d p ' = 21'.
37. A man standing 152 feet from t h e foot of a f l a g p o l e , whlch is
on h i s eye l e v e l , observes t h a t the angle of elevation of the
top o f the Flagpole is 48'. Find the h e i g h t o f the p o l e .
38. TWO p o i n t s A and are on t h e bank of a river are 40
B
f e e t a p a r t . A p o i n t C a c r o s s t h e river is located so
that angle CAB is 70' and angle ACB is 70'. How wrde
is t h e r i v e r ?
39. The adjacent s i d e s of a parallelogram are 20 and 15 inches,
respectively, w h i l e the s h o r t e r diagonal is 17 inches. What
is t h e l e n g t h of the l o n g e r d i a g o n a l .
40. A f l a g s t a f f known to be 20 f e e t hi& stands on t o p o f a
bul l d i n g . An observer across t h e s t r e e t o b s e r v e s t h a t t h e
angle of elevation of the bottom of the f l a g s t a f f is 69'
and t h a t the angle of e l e v a t i o n of the t o p of the f l a g is
76'. F i n d t h e h e i g h t o f the bullding.
41. is a t o w e r which stands on a v e r t i c a l cliff z.
At a
p o l n t P 310 f e e t from t h e f o o t o f the c l i f f , t h e angle of
e l e v a t i o n o f B is 21' and the angle of e l e v a t i o n o f A
is 3 5'. Find the h e i g h t o f t h e tower.
42. Use t h e f i g u r e below t o find C
t h e following:

a. R,
b. r,
c. L BAO.
Chapter 11
THll SYSTEM OF VECTORS

11-1. Directed L i n e Segments.


It is assumed in t h i s chapter that you are familiar w i t h
w
plane geometry. We review some of the symbols of geometry. AB
means the l i n e which c o n t a i n s t h e distinct polnts A and B .
+
AB means the ray whose vertex is A and which a l s o
contains the point B . (ABI means the d i s t a n c e from A to 3
(and from B to A ) , It is a positive r e a l number if A and B
are distinct. It is zero if A and B are the same,
We need one further idea which I s n o t ordinarily covered in
geometry--that of parallel rays. Rays are said to be parallel if
they lie on l i n e s which are either p a r a l l e l or coincident, and if
they are slrnilarly sensed. Figure 11-la shows t y p i c a l instances
of rays which are parallel and of rays which are not parallel,
and l a supposed to take the place of a formal definition.

4 -+ 4 -4
rays AB and CD are II rays AB and CD are n o t 1)

Fig. 11-la
Definition 1 3 - l a : A line segment is said to be a directed
l i n e segment if one of i t s endpoints is designated as i t s l n i t l a l
point and the other2 endpoint Is designated as i t s terminal point.
We use the symbol AB to denote the d i r e c t e d line segment whose
initial p o i n t is A and whose terminal point is B
2 4
.
We say t h a t
directed line segments AB and CD are equivalent if it Is true
that t h e l r l e n g t h s are t h e same
C
and also t h a t the rays
2
a
and
are p a r a l l e l . We write AB CD to denote the f a c t that
and CD are equivalent.
Note: We c o n s i d e r t h a t a single p o l n t can be both i n i t i a l
and terminal polnt o f the same directed line segment and we
consider that all such d i r e c t e d line segments are equivalent to
one another,

Fig. 11-lb

Figure 11-lb shows some pairs of equivalent directed l i n e segments.


It uses the convention that the endpoint of a segment which has an
arrow I s the terminal point of the segment. Notice that if A, B,
2 4
C, D are not c o l l i n e a r , then AB ;CD if and only if ABDC is a
3
parallelogram. We need the fact t h a t if AB 1s any directed line
segment and if C I s any point, then there I s one and only one
point D such t h a t AB
2 2
CD .
We do n o t grove this f a c t , but
assume that it is known from the study of geometry.

[ s e c . 11-11
-
Definition 11-lb: Let A8 and CD be any two d i r e c t e d l i n e
-
2 2
segments. Then by t h e i r sum AB + CD we mean the directed line
.
4 3 .
L

segment AX, where X is the unique p o i n t auch t h a t BX & CD


We call t h e operation whlch assigns their sum t o each pair of
directed line segments the addltion operation for directed line
segments. Figure 11-lc shows some sums of directed l i n e sewents.

Fig. 11-lc

Directed l i n e segments can be added and m u l t i p l i e d by real


numbers in a useful way. We give the formal definition of these
operations here. Their propertles'are studied and applied through-
o u t the reat or the chapter.
A
D e f i n i t i o n : L e t A E be any directed line segment and l e t r
be any real number. Then the product
2
re I s the dlrected l l n e
segment AX , where X l a determined aa Pollowa:

I s e c . 11-11
2

(1) If r ) 0 , then X I s on the ray AB and 1 ~ x 1= ~ ~ A B I.


2
( 2 ) If r ( 0 , then X is on the ray oppasite to AB and
\ A X \ = - rlAB\.
(3) If r = 0 , then X = A .
(4) I f B = A , then X = A .
Figure 1 1 - l d shows some typical products.

It Is u s e f u l to know t h a t If equivalent directed line seg-


ments a r e added t o equivalent d i r e c t e d line segments the sums are
equivalent, and that if e q u i v a l e n t d i r e c t e d l i n e segnents are
multiplied by the same number the products are equivalent. We
now state theae f a c t s formally as theorems and I l l u s t r a t e them.

-
Theorem 11-la : and if then
-5 2

AX A CY.

Figure 11-le shows a typical instance of t h f s theorem. It is


equivalent to the f a c t t h a t if ABDC is a parallelogram and If
XYDB is a parallelogram, then AXYC is a parallelogram. T h i s
is a s p e c i a l case of a famous theorem of geometry known as
Desargues Theorem.

2 d
Theorem 11-lb: If A13 k CD and If r is any real number,
then
A
rAB 1 rCD
-5

. Y

- - #
- 2
/

I
I
,--
C ,--
v-
X

d 2
AX = rAB;
A a
CY = rCD;
2

AX - 2

CY .
Fig. 11-1f
Figure Il-lf illustrates a case in which A, 3, C, D are n o t
c o l l i n e a r . It also illustrates the geometric version of the
3

statement, t h a t if ABDC is a parallelogram and if AX A CY


then AXYC is a parallelogram.

Exercises 11-1
1. A and B are d i s t i n c t points. List a l l the d i r e c t e d llne
segments they determine.
2. A, B and C are d i s t i n c t p o i n t s . List a l l the directed l i n e
segments they determine.
3. A , B, C and D are vertices of a parallelogram. List a l l
t h e directed llne segments they determine, and i n d i c a t e whlch

4. In triangle ABC
3 2

(a) AB + BC = ?
(b) BA
A
+ ?
2
= BC .
(c) ?
2
+ Bh
C
= BC .
(d) ? + RB. A '
L
= AA
(el ( G + Z+
)6= A A
?
(f) + (AC +CB) = ?
a
.
2
(g) ? + AC = CB
5. At B and X are c o l l i n e a r p o i n t s . Find r such that

and s such t h a t
-
(a) X Is the midpoint of segment AB
(b) B I s the midpoint of segment - AX
- ..
(c) A is the midpoint of segment BX .
(dJ X is two-thirds of the way from A to B .
( e ) B i a two-thirds of the -way from A to X .
(f) A is two-thirds of the way from B to X .
6. In triangle ABC , X is the midpoint of and Y is the
-
midpoint of segment BX . 0
A A 2
( a ) 3X = BA + ?AC .
(b)
2
= ?BY .
( c ) BX = BC + ?
2
.
( d ) BX = BC + 1 ?
2 2
.
(e)
2
BY
2
= ?BX .
(f) s =?(Z+iiii).
(9) G ?s+ s .
= A X C

11-2, Applications -
to Geometry.
It i a possible to use d i r e c t e d line aegments to prove
theorems of geometry. These proofs are based on algebraic
properties of directed line segments. They are q u i t e d i f f e r e n t
from proofs usually g i v e n In geometry which appeal to such matters
as congruent triangles and the l i k e .
We s t a t e and illustrate the necessary algebraic properties of
directed line segments here. We prove these statements in
Section 11-3.
I. Commutative -
Law:
2
AB + CD = CD + AB ,
Figure 11-2a shows an i n s t a n c e of the c o m u t a t f v e law f o r
2 2

a d d i t i o n in which t h e dlrected line segments AB and CD have


a common initial point.

Fig* 11-a

11. Associative &:


&

AB +
A 2
(CD + EF)
2
= (AB +
Z
A
CD) +
A
4
EIF .
2
Figure 11-2b shows sums AB + (CD + EF) in which B and C
are the same and D and E a r e t h e same.

111. Existence of Z e r o Elements.


Every d i r e c t e d l i n e segment of the type AA 1s a zero
2 - 2
element because PQ + AA = PQ ,
IV. Existence -
of Additive Inverses.
2

BA 18 the a d d i t i v e inverse of
A
AB , because
2
AB +
A
BA = AA .
We use a minus sign to denote the a d d i t i v e inverse of a
directed line segment
- AB , and write - AB for BA
2 2 2
.
We write
&
- A
Pe AB for PQ + BA
A
.
This operation of subtraction is illustrated in Figure 11-2c.

F i g . 11-2c

V. T h e Associative m.
. { 8 f , = crs,t m

Figure 11-2d shows an instance of the associative law in


which r = - T1 , s = 4 .
VI. The Distributive L a w s :
2 2 A A
r(A3 + CD) = rAB + rCD,

Flgure 11-2e i l l u s t r a t e s t h e d l a t r i b u t i v e laws f o r


r = 4 , s=-2.

The combined effect of a l l these laws can be summed up


b r i e f l y as f o l l o w a :
Directed line segments obey the
familiar rules of algebra with
respect t o addition, subtraction,
and multiplication by real numbers.
We now show how t h f s algebra of directed line segments can be
a p p l i e d to proving theorems of geometry.
Example 11-2a: Show that the midpoints o f the sides of any
quadrilateral are vertices of a parallelogram,

-
Proof: Let ABCD be t h e quadrilateral ( s e e Figure 11-2f)

Fig. 11-2f

and let X, Y, 2, T be t h e midpoint8 of its sides as IndLcated.


A
It is s u f f i c i e n t to show that XY A TZ s i n c e t h i s implies both
++
r
that XY 1 1 TZ and that I X Y ~ = ITZ~ .
W e have XY=$AB++

and

+ 3C , we
4 2
Since = + AB a130 have

T h i s shows that
A A

XY L TZ .
Example 11-2b: Prove that the diagonal8 of a parallelogram
b i s e c t each other.
Solution: Let ABCD be the parallelogram ( s e e Figure I l - 2 g ) .

Pig. 11-28
1 - + BC)-.
Then the midpoint af
midpoint of 5
I s the endpoint of

-
3
?(AB
-
1 - + AD) which equals
is the endpoint of AB + ?(BA
The

2
AB 9-
1- + gT 1-
-
1-
or -;AB + $D .
We show t h a t t h i s is the same as
1-
$B + 1-
. Since ABCD is a parallelogram d
AD BC , so the
1- 1-
last sum Is certainly equivalent to +B +9 . We conclude that
these directed line segments are t h e same by noticing t h a t in
addition to being equivalent they also have t h e same I n i t i a l p o i n t .

Example 11-2c: Prove t h a t the medians of a t r i a n g l e meet in


a p o i n t which trisects each of them.

Solution: Let ABC be the triangle. (see Figure 11-2h. )

Fig. 11-a
Let X, Y,
be the midpoints of its sides. Then, the p o i n t two-
Z

thirds the way from A to X i s tho endpoint of +p 1- ). $(z


The point two-thirds the way from 3 to Y ie the endpoint of

The p o i n t two-thirds the way from C to Z is the endpoint of

2
same. We use the f a c t that BC
-
BA + AC
3
-
We show that these three directed l i n e segments are one and t h e
.
Then the first I s equal to

which is equal to 2-
3(" - *
1-
+
1-
YC)

me second is equal t o AB
3
- +B
2- + +? which also equals

The t h i r d is equal to - -JAC


2-
+$ which also equals

-
Example 11-2d: Prove that the line which joins one vertex of
a parallelogram to the midpoint of an opposite side is trisected
by a diagonal. Prove also that it trisects this d i a g o n a l ,

Solution: Let ABCD be the parallelogram (see Figure 11-21).


Let A be the given v e r t e x and let X be the midpoint of 5.

F i g . 11-21
We are to show that the point two-thirds of the way from A to
I s the same as the p o i n t two-thirds of the way from D to 3 .
The f i r a t p o i n t is the endpoint of

The second point is the endpoint of

Thia latter equals

A A
Since AD is equivalent t o BC we see that these two
directed line segments are equivalent; that they are in f a c t the
same follows from the additional fact that they have the same
i n i t i a l point.

Exercises 11-2
4

If ABCD is a parallelogram,
A
express DB ,
DA .
A

( a ) in terms of A
DC and A
(b) in t e r n of DC and CB .
A
( c ) In terms of AB and BC
A
.
2
( d ) in terms of AB and AD
2
.
(e)
A
in terms of BA and BC
A
. A 0

2. If A and 3 are d i s t i n c t points, i d e n t i f y the s e t of a l l


terminal points of the d i r e c t e d line segments of t h e form
2
t A 3 for which
(a) t > o . (4 t -
> l .
(b) - -
o;t<1. (d) -
- l < t < l .
d
3. If A, 8, Care non-collinear points, find the s e t of a l l
terminal p o i n t s of directed line segments of the form

for which
(a) r = 0 ,s arbit~ary.
(b) s = 0 , r arbitrary.
(c) 0 -< r -( 1 , a arbitrary.
(d) 0
-( s -< 1 , r arbitrary.
(4 - -
O < r < l , 0 < 3 < 1 -
(f) r = 1 ,
a arb?trary.
(g) s = 1 ,r arbitrary.

*(k) 6r + 7s = 8 .
+(1) ar +
bs + c = 0, where , are real numbers and
where n o t both - a and b are z e r o .
4. Show by an example that subtraction of d i r e c t e d l i n e segments
( a ) is not c o m t a t l v e ,
(b) is n o t associative.
4. In the following figure

ABCD, EOOB, and HDFO are each parallelograms. Prove that


- - h

their respective diagonals AC, Ea, HF, extended if necessary,


meet in a single point X .
6. ABCD is a parallelogram and P, Q, R, S are the midpoints of
the s l d e s .
A

For each of t h e following directed l i n e segments, f i n d an


.
2 d
equivalent directed l i n e segment of the form r W + s OP
d 4
(a) OB (4 DB
(b) z
L
(f)
2

(4 OD (g) C*
(d) s (h) BD
7. Show t h a t the f o u r diagonals of a parallelepiped bisect one
another.

11-3. Vectors and Scalars; Components.


D i r e c t e d line segments acquire new propertfes when algebraic
operations are defined f o r them, so I t is proper to give them a
new name. Real numbers a l s o acquire new properties when they
m u l t i p l y d i r e c t e d l l n e segments, so it is proper to rename them
also. From now on we s h a l l call a dlrected llne segment a vector.
We s h a l l call a real number a scalar if and when it multiplies a
vector. This I s a refinement which is n o t absolutely necessary
f o r logical thinking, but It h e l p s .
We are going to discuaa equivalence of vectors, a d d i t i o n of
vectors and multiplication of vectora by scalars i n terms of
coordinates. The following theorem is t h e basic tool in t h i s
discussion.
-
Theorem 11-3a: Let A, B, C, D have respective coordlnatea
(a1.a2), (bl.b2), (c1,c2), (dl'd*) Then

if and only if
bl - al = dl - c1 and b2 - a 2 = dg - . c2

-
Proof: Figure 11-3a illustrates Theorem 11-3a.

We give only a few Indications of i t s proof.


If bl - a1 = dlf - c1 and if b2 - a2 = d g - c 2
then

and

provided that
b1 - a1 4 0 and bg.- a2 4 0 .
We conclude t h a t IABJ = ICD) and that AB I ) CD
--
This
makes plausible the fact t h a t if the given equations hold then
+ 4
.
AB 4 CD . It doesntt completely prove t h i s (we need AB I I cD)
I A

and it doesntt contribute at a l l to t h e proof of the converse.


4 C
Corollary: If OP 1s the v e c t o r equfvalent to AB , where
0 ISthe o r i g i n , then P has coordinates (bl al - , b2 - a*) .
-
D e f i n i t i o n 11-3a: If A is the point (al,a2) and 3 Is
the point (bl,bg), we c a l l the number bl -
al the x-copponent
a
-
of AB , the number b p a 2 the y-component of AB
2
.
In most discussions of vectors the i n i t i a l and terminal points
of the vectors are n o t 8 s important a s t h e i r x and y-components,
W e e h a l l therefore often specify a vector by giving its x and y
component. We use square brackets [ , I to do this; [ p , q ] means
any v e c t o r whose x-component i a p and whose y-component is q .
We shall sometimes denote v e c t o r s by single l e t t e r s , with an arrow
A
above, l i k e A , when the apeclflc endpoints are n o t important.
& C
We also write A = B to assert that t w o vectors are equivalent.
The equal sign should properly connect not the vectors themselves
but their components. Thus Theorem 11-3a can be restated as
follows :
2 2

ff X 1s (xl,x2] and Y ia [yI,y2J , then


A C
X = Y
if and only if
x1 = y1 and x2 = Y 2

We use the symbol 1x1 to denote t h e length of X . We have


We t u r n now t o the a d d i t i o n and multlplicatlon operations for
vectors, show how they can be effected i n terms of componenta and
prove the basic a l g e b ~ a l claws stated for them in Section 11-2.

Theorem 11-3b: If X is [x1,x2] and if Y 1s [y1,y2]


2 2
then X + Y is [(xl + Y ~ ) . ( x 2 +~ ~ 1 1 .

-
Proof: By d e f i n i t i o n of a d d i t i o n fop vectors ( s e e
z ( x,+ Y,, x*+ Y,)
Figure 11-3b)

F i g . 11-3b
A -L A
OZ is OX f OY if and only if XZ ;OY
C C
.
According to
Theorem 11-3a, t h i s w i l l be so if and only if t h e point 2A is2
(xl + y1 , x2 + yg) . It f o l l o w s that the components of X + Y
are xl + yl and x2 + y2 .
Corollary: Addltfon of vectors i a commutative.
2 - a -
X + Y = Y + X ,

Corollary: Addition of vectors l a associative.


- a - 2 - A
( X + Y ) + Z = X + (Y+Z) .
Corollary: There is a zero vector [0,0],

2
C o r o l l a r y : Every vector X has an additive Inverse -X .
A

If X is [xl,x2] , then
3

- 2

X is E-xl, -x2] .

-
Theorem 11-3c : If X
2
La [x1,x2] , then
a
rX is [rxl, mc2] .
Roof: Let Y be the point (rxl,rxg) (see Figure 11-3c).

Fig. 11-3c

Then

Also 0, X, Y are colllnear, s l n c e they are on the line whose


equation I s x2x -
xly = 0 . We must show that the ray 1s 3
+
parallel to the ray OY to complete our proof. We omit t h i s
p a r t of the proof.

Corollary: Multiplication by scalars is associative.


Corollary: Multiplication by scalars obeys the d i s t r f b u t i v e
laws.

A &
Corollary: If X Is [xl,xg] and if Y 1s [yl,yp] then

-
Definition 11-3b: Mon zero vectors X and Y are said to
A
be parallel if and only if t h e d i r e c t e d l i n e seepnants OX and
equivalent to them are c o l l i n e a r .

Fig. 11-3c

A 2

Theorem 11-3d: Non zero vectors X and Y are parallel if


and only if

Por some non-zero real number r .


a
-
2
Proof: Let X be [x1,x2] and Y be [y1,y2], let X be
the point (x1,x2) and Y be the p o i n t (yI,y2) .
4 &
Then OX A X , OY A Y
- 4

. Then
A
X II
2

Y if and only if 0, X and


Y are c o l l i n e a r . Rut

if and only if

which holds If and only if 0, X, Y are colllnear,

A A
x n o t parallel to Y

-
Theorem 11-3e: Let and ? be any p a i r of non-zero, non-
parallel vectors. Then for each vector
2
there are numbers r
and s such t h a t

-
Proof: Let
a d -
X, Y, 2 be [x1,x21 , [ Y ~ , Y ~,I [ Z ~ ' Z ~ I. We

are to show that the equations f o r r ,s


.
have a unique s o l u t i o n ( r , s ) Since X
A

is not parallel to
2

Y ,
it followe from Theorem 11-38 that

Our conclusion now f o l l o w s from the result of Chapter 7, Section 3


on the existence and uniqueness of solution of equations.
a - A 2
Corollary: If r X + BY u 0 (where 0 is a zero v e c t o r )
then r = s = 0 .
-
Definition 11-3c: Any two non-zero, non-parallel vectors in
-
the plane are said to be a base f o r a l l the vectors of the plane.
Fig: 11-3e

Esec. l l - 3 j
A A
Figure 11-3e shows two base vectors X and Y and vectors
and
A A 2
expressed in t h e form rX + sY .
[1,0] and [0#1] form a base which I s frequently used.
2
The vector [1,0] is denoted by i and the vector [0,1] l a
denoted by 7.
- - A A
'Iheorem 11-31 : X = ai + bj if and only if X is [a, b ]
and (a,b) is the point P for which

-
A
Proof: If X is [a,b] , then, Bince

I t follows that

- 2 -
If X L a l + bj , then

2
Figure 11-3f shows an example of a vector X expressed as a
.
A A
aum 31 + 2j
1, If A, B and C are respectively 1 , 4 , 6 , find
X so t h a t
A 2

(a) AB 2 CX ,
A 2
(b] AX & CB ,
(c)
d
XA CB
c
2
.
.
A 2
(d) Xh L BC
2 Same as Problem 1, if A, B, C ape respectively (-1,2), (4,-31,
( - 6 , -1).
3. F i n d the components of

(b) f3p-21 + 1-4,11


(4 415361
(a) -4[5,6] .
(el -1[5161
If) - 15,6J -
(g) 3 1 4 , l I + 21-1,31 .
(h) 3[11,11 - 2[-1,3] .
4. Deternine x and y so t h a t
(a) x13,-11 + y13,11 = [5,63 .
(b) x[3,2] + ~ 1 2 , 3 1 1 1 9 2 1 .
( c ) x13,21 + ~ I - 2 , 3 1 = [ 5 # 6 1
( d ) x [ 3 , 2 ] + y[6,4] = [-3,-21 ( ~ n f l n l t e l ymany s o l u t i o n s ) .
5 Determine a and b so that
a
.
a
( a ) [3,1] = ai t bj
2.
(b) [1,-3) = ai + b j ,
>
( c ] 5 = a[-3,1] + bl1,-33 .
6
(d) 5- a[-3,l] + b(1,-31
&ternnine a and b s o that 31
-I
.
- a-
23 = a(31 + 4 j ) + b(41 + 33)
a 2
.
11-4. -
Inner Product.
Our algebra of vectors does n o t y e t Include multiplication of
one vector by another. We now define such a product,
We first say what we mean by the angle between two vectors
2 2
X and Y which do not necessarily have a common I n i t i a l point.

Definition 11-4a: Let X and be any non-zero vectors


2
and let OX and be vecfors whose i n i t i a l point i a the origin
0 and which are equivalent respectively to X and Y
L 3

A
.
Then by
the angle between X and 7 we mean the angle between OX and
.
A

OY

D e f i n i t i o n 11-4b: Let ? and ? be any vectora. Then t h e


2
inner product of X and Y is the real number
2 A
1x1 IYI cos e
2 A
where \ X 1 is t h e l e n g t h o f y , 1 is t h e length of Y and 9 is
A
the angle between X and Y
2
. a 2
(1f X OF Y l a a zero vector
then 8 is not defined. We i n t e r p r e t the definitions to mean
that the inner product is zero, in this c a s e . )
The i n n e r product has Mportant properties. Before we
investigate these properties of the Inner product we relate the
inner product to a familiar mathematical relation--the law of
cosines.
A A
If our given vectors X and Y are not parallel they
determine a - t r i a n g l e OXY , where 0 is the o rC
3
i g i n and where X
and Y are endpoints of the vectora OX and OY respectively
equivalent to X and Y .2
We can f i n d at least one earlier
appearance of the inner product by applying t h e law of cosines to
t h e triangle. It asserts (Figure 11-ha)
F i g . 11-4a

so that

Thus t h e expression we have called the "inner productn i s


suggested by t h e law of c o s i n e s .
3 2

W e sometimes denote t h i s product by the symbols X Y


A
(read '% dot Y") and sometimes call it the "dot product. "
Usually, in algebra, a multiplication operation f o r a s e t of
objects assigns a member of t h i s set to each pair of Its members.
The inner product I s n o t an operation of t h i s type. It does n o t
assign a vector to a pair of v e c t o r s but rather it assigna a
real number to each p a i r of vectors.
Exam le -
2 11-4a: Evaluate
2
X . C
Y if 1x1 = 2 ,
2 A
IYI = 3 and
(a) 0 , 0 , ( b ) B = 45' , (c)e = g o 0 , ( d ) e = 180' .
Solution :
A
(a) ~ . ~ = 2 . 3 c o s o ~ = 2 . 3 . 1 = 6
(b)
A

X .Y
'
L

= 2 3 cos 45' = 2 - 3 .q=


C
3 0
(c)
A
X
>
a
Y = 2 . 3cos90°=2 - 3 O = O
. .
A

(d) X Y = 2 3 cos 180' = 2 3 (-1) = -6

The i n n e r product has many applications. One of these is a test


for perpendlcularfty.

-
Theorem 11-4a: If X and Y are non-zero vectors, then
they are perpendicular if and o n l y if
A '
L

X ' Y - 0 .

-
Proof: According to the definition of I n n e r product

This product of real numbers is zero if and o n l y if one of its


2 2
f a c t o r s isAzero. Since
a
X and Y are non-zero vectors, t h e
numbers (XI and IY\are not zero. Therefore the product is
z e r o if and only if cos 8 = 0 , which is the case If and only If
4
X and Y are perpendicular.
The following theorem suppllea a u s e f u l formula for the
l n n e r product of v e c t o r s .

-
Theorem 11-4b:
2

If X = [xl,x2]
a
,Y = [y1,y2]
then
-
Proof: According to the law of cosines ( s e e Figure l l e 4 b )

- XIY1 'x g 2 .
A 2
Since, by d e f i n i t i o n , the left member of t h i s equation l a X Y ,
our theorem 1s proved.

A 2

2 -Exam le 11-4b:
P- If X is [ 3 , 4 ] and Y is [5,2] , find
X . Y .

Solution:
A
X .Y = 3 . 5
C
-I-4 .2
J 2
Exam le 11-bc:
d- If X is [ 3 , 7 ] and Y Is [-7,3j , show
t h a t X and Y are perpendicular.

Solution:
2
X
2
Y = 3(-7) + 7 3 = 0 . 2
The conclusion follows from Theorem 11-4a, and the f a c t that X
A
and Y are non-zero.
A u s e f u l fact about
inner products is t h a t they have some of
the algebraic properties of products of numbers. The following
theorem gives one such cormnon property.

- a -
Theorem l l - 4 c : If X ,Y ,2 are any vectors, then
A - 2 A d 2 2
X (y+z)=x Y + X 4 Z
(tS;) * Tt(X5
2 2

Y)
Corollary:
2
X .
2
(aY +
A
bZ) = a ( x
2
. Y) + b(X .
a A 4
Z) .
In certain applications of vectors to physics the n o t i o n of a
component of a vector in the d i r e c t i o n of a n o t h e r v e c t o r I s
Important. We now define t h i s concept.
-.L A
Definition 11-4c: L e t X be any Anon-zero v e c t o r and l e t 2Y
be any vector. Then the component of Y in the direction of X
is the number given by each of the following equal expressions:
2

-X=
13
2
Y rjtj 2

Ix I
13 COB B
=
-
~ Y Icos 0.

A
NOTE: The component of in t h e direction of ? can be
Y
described geometrically ( s e e Ffgure 11-4c ) .

F i g . 11-4c

In both parts of the figure PA is the f o o t of the perpen- 3

dicular from the I n i t i a l point of Y to the line of X , anda Q


I s t h e foot of the perpendicular from the terminal point of Y t o
2
t h i s l i n e . In the fipst part the component of Y in the direction
A
of X t u r n s out to be the distance from P to Q . In the
second part t h i s component t u r n s out to be the negative of the
diatance from P t o Q .
The inner product is used f r e q u e n t l y in applications of
vectors to physics. For the moment we consider i n n e r products
from a purely mathematical standpoint,

2
Example 11-4d: L e t X be any v e c t o r p a r a l l e l to the
2
positive x-axis, let Y be any vector parallel to the p o s i t i v e
A
y-axis and let Z be the v e c t o r [ p , q ] . Show t h a t p and q are
A
t h e components of Z in the direction of ? and Y respectively.

Solution: According to Theorem 10-3

cos 0 -

so p = cos 0 *-pJ

Since 12 1 - d p v, we conclude that


The angle between
2
't and the y-axis is f
2 - 8 . Conse-
2 a
quently the component of' Z In the direction of Y is

Slnee eos ($ - e ) 5 s i n e and s i n c e


4

we conclude that t h i s component is, in f a c t ,

Much of o u r discussion of
V e c t o r s in Three Dimensions:
vectors in the plane can be carried over to three dlmensiona
with only minor modifications.

[ s e c . 11-41
The portlons about d i r e c t e d line segments require no modifi-
cation. When we come to coordinates and components, the con-
clualons are as follows:
1. The components of a vector in three dfmensional space
are an ordered triple [ a , b , c J of real numbers.
2 Two vectors [a,b,c j and [p, q, r] are equal if and
o n l y l f a = p , b = q and c = r .
3. The a d d i t i o n of vectors [ a , b , c ] and [p,q,r] I s
given by the rule

4. Scalar multiplication of vectors is given by t h e rule

The u n i t base vectors associated with the coordinate


axes are a
i= [l,O,O]
2

J = tO,l,OI

Figure 11-4d shows these baa8 vectors.

Fig. 11-4d
3 2 2 A
The vector V = 41 + 8j + 8k I s Illustrated in
Figure 11-4e.

F i g , 11-4e
2 2
6. The inner product of V and W is stfll g l v e n by
-I-

v = w = lvllwl cos e ,
a a
In component form if V Is [vl,v2,v3] and W is

also
3

I V I =Jvl
2
+ vp
2
+ v3
2 .

Exercises 11-4
A 2
Find X ' Y if
& - A &
(a) X = I, Y = j .
- a > -
(b) X = i , Y = i .
A d 2 2
(c) X = j , Y = i .
- - - - A -
(e) X = I + J , Y = i - J -

(h) y, a l +
2
bj
-
,Y
-
= ci +
-5
dj .
a a -
3
(i) X = ai + bj ,Y
d
= 4X .

2. find the angle between ji and ? If ljil = 2 , = 3


2 C

and X Y is
(a) 0 , ( b ) l , (c)-2, (dl 3 , ( 4 - 4 , (f) 5 , ( d h y
(h) -6 . 2 A
3. If ?= 3? + 4j , determine a so that Y I s perpendicular

(b) a l - 47,
2
(d)
A
ai
2

- A
33 .
4. Find the angle between X and Y in each p a r t of Exercise 1
above.
2 2

5. If a 2 + b2 40 prove that a1 + bJ is perpendicular to


cy+ dr If and only If a l + b r I s p a r a l l e l to
A A
-dr+ c 7 .
6. Find the component of Y in the direction of X if
a A 2
. .
- A d 3 A A 2 2
(a) X = i , Y = 3 1 + 4 J (e) X = 3 1 + 4 j , Y = 3 1 + 4 j
a
(b) X = j
2
-

A
,Y
2
=
2
2

31
-
+
2

4j
2
. (f) 'ji=3r+
A
47, Y =
A a
51 +
2
2j
-
.
(c) X=31+4j, Y = I . (g) ~ = 3 1 + 4 j , Y = a i + b j .
-
11-5. Applications of Vectors in Physics. -
The n o t i o n of "force" is one of the Important concepts of
physics. This is the abstraction which physicists have invented
to describe ''pushes" and 'lpullsl'and to account f o r the effects
that pushes and p u l l s produce.
The student who knows something about vectors can r e a d i l y
learn about forces. The connecting links between the concepts of
II
force1' and "vector" can be stated as follows :
1. Every f o r c e can be represented as a vector. The
direction of the force is the same as the direction
of I t s representative vector. The magnitude of the
force determines the length of i t s representing
vector, once a "scale" has been selected.

Example 11-3: A red-headed man is standing on t o p of a h i l l ,


staring I n t o space. He weighs 200 pounds. Represent as a vector
each of the following:

(a) the downward pull of the earth's gravity on him,


(b) t h e upward push of the h i l l on him.

Scale: 1 inch = 200 lbs,. Scale: 1 inch = 200 lbs.

2. of forces which a c t on a single body


Any c o l l e c t i o n
is equivalent to a single force, c a l l e d their
resultant. If all the forces are represented a s
v e c t o r s on the same scale, then the vector whlch
represents the resultant of the forces is the sum
of these vectors.
Example 11-5b: Represent each of' t h e f o l l o w i n g forces a a a
vector, and f i n d the vector which represents their resultant: A
force PI of 300 pounds directed t o the right, a force F2 of
400 pounds directed at an angle of 45' with the x-axis and a force
of 500 pounds directed upward.

Solution: (graphical) Using the scale 1 inch = 400 pounds


2

X
a 2

represents F1 , Y represents P2 , Z repreaents F3 .


A
z A

.
2 - 2
X +Y c Z represents the r e s u l t a n t of F1 , Fa , F3
Its length 1s a llttle less than 5/2 inches; its direction is
about 54' .
3. If F and G are t w o forces which have the same
d i r e c t i o n , then they have a numerical ratio and
there is a number r auch t h a t r tlmesaforce F
l a equivalent t o force G. Moreover if F is the
vector which represents force F , then rF I s
2

'the vector which repreaents force U , where r is


the r a t i o of force Q to force F .

Csec. 11-51
Example 11-5c: W l y and Elsie are I d e n t i c a l twins. They are
a
a l t t l n g on a fence. If F represents the t o t a l force Emily and
A
Elsie exert on the fence and if G represents the force the
fence exerts on anfly alone, express
A 4
(a) F in terns of O
2
.
(b) F in terns of P .
Solution :
(a)
2
F
A

-Xf .
-
(b) G = - $r*
A body at rest i a said to be in equlllbrium. It is a f a c t of
phyaica that If a body I s at r e s t the resultant of all the forces
acting on t h e body haa magnitude zero. (~ote: The converse of
t h i s l a n o t true, since the resultant of a l l the forces which act
on a moving body can alao be z e r o . According to the laws of
physics, if the sum of all t h e forces which a c t on a body is zero,
then the body must be either at rest or it must move in a straight
1ine wP th c o n a t a n t speed. )
Exerciaes Il- 9
1, A weight is suspended by ropes as shown in the f i g u r e .

If the weight weighs 10 pounds, what is the force exerted


on the junction C by the rope CB ?
2. A weight of 1,000 pounds I s suspended from wires as ahown
in the f i g u r e ,

-horizontal

-
The d i s t a n c e AB I s 20 feet, AC La 10 feet, and CB i a
10 f e e t . What force does the wlre AC exert on the
junction C ? What force does w l r e BC exert on C ? If
all three wires are about equally strong, which wlre is
most likely to break? Which wire is least likely t o break?
3. A 5000 pound weight is suspended as shown in t h e figure.
Find the tension i n each of' t h e ropes CA, CB and CW.

4. A b a r r e l i s h e l d i n place on an i n c l i n e d plane EF by a
fdrce operating parallel t o the plane and another
o p e r a t i n g p e r p e n d i c u l a r t o I t . ( s e e diagram. )

If the weight of t h e barrel is 300 pounds, ( I = 300) OWI


and t h e plane makes a n angle of 23' with t h e h o r i z o n t a l
find and J ~ I (Flint:
. Introduce a coordinate system
with origin a t 0 and OW as negative y-axis.)
5. A weight is suspended by ropes as shown in the figure, If
the w e i g h t weighs 20 pounds, what is t h e f o r c e exerted on
the junction C by t h e rope CB? By the rope AC? If AC
and CB a r e e q u a l l y strong, which one is more likely t o break?

A 500 pound weight is suspended as shown in t h e figure. Find


each of the forces exerted on p o i n t C .

weight

A 2,000 pound weight is lifted a t c o n s t a n t speed, as shown in


the diagram. Find each of the forces exerted on point C .

\ hinged strut
[ s e c . 11-51
propeller t h r u s t

drag

force of g r a v i t y

The motion of airplanes provides another application of


vectors. Some technical terms involved are l i s t e d and i l l u s t r a t e d
in the figure.

-
Lift:
2
FL--a force perpendicular to the direction of motion.
This l a the "llftlng f o r c e " of the wlng.
2
Gravity: F --a f o r c e d i r e c t e d downward.
g
A
Propeller thrust: F --a forward force in the direction of
pt
mot ion.
2
Drag: Fd--a backward force parallel to the motion. This
f o r c e I s due to wind resistance.
A
Effective propeller thrust: Fept--the propeller t h r u s t minus
the drag.
The physical principle we shall use s t a t e s that a body in
motfon will continue to move In a s t r a i g h t l i n e w i t h constant
speed if and only if the resultant of a l l the forces acting on
the body is zero.
8. A n airplane weighing 6,000 pounds climbs s t e a d i l y upwards
a t an angle of .
'
0
3 Find the e f f e c t i v e propeller t h r u s t
and t h e lift.
9 . An a i r p l a n e weighing 10,000 pounds climbs at an angle of 15'
w i t h c o n s t a n t speed. Find t h e e f f e c t i v e p r o p e l l e r t h r u s t
and t h e l i f t .
10. A motorless glider descends a t an angle of 1 0' with c o n s t a n t
speed. If the g l i d e r and occupant together w e i g h 500 pounds,
find the drag and the lift.

--
The term ' k o r k " as t h e physicists use it also provides an
example of a concept which can be d i s c u s s e d in terms of vectors.
Consider for lnstance a t r a c t o r p u l l i n g a box-car along a track,

m air1-
8 ,
I

1 [MIIII
-
LI u ii Track
- 3
Liz- c:(Directlon of motion)
-- 2 --2-
<--,-
&

---+-,------
c-
- -
-
---<-
rac t o '~ -

F i g . 11-5a

The effect of t h e t r a c t o r t s force depends on the angle 8 It .


a l s o i n v o l v e s t h e force i t s e l f and the displacement produced. The
term2"work, " as2
2
used in p h y s l c s , is given in t h i s case by 2

F . S , where F is the force-vector of t h e tractor and where S


is t h e displacement of the box c a r .
More g e n e r a l l y , I f a f o r c e F a c t s on a body and produces a
displacement S while it a c t s , then the work done by the force is
A .
L 2
defined t o be F S , where F Zs the vector which represents
3
the force and where S is the vector which represents t h e d i s -
placement,
&ample 11-5e: If the tractor of Figure 11-+5a exerts a
f o r c e of 1,000 pounds at an angle of 30' to the track, how much
work does the tractor do in moving a string of cars 2,000 feet?

A
- +
Solution: Evaluate the expression IF'I.lSl cos Q where
IF! = 1,000 pounds, IS1 = 2,000 feet, cos 8 % .866. The value of
t h i s product is approximately 1,732,000 f o o t pounds.

Exercises 11-5b

1. a distance of d feet by a force of


A a l e d is p u l l e d p
pounds which makes an angle of e nlth t h e h o r i z o n t a l . Find
the work dons If

( a ) d = 10 f e e t , p = 10 pounds, 8 = 10' .
0
(b) d = 100 feet, p = 10 pounds, 8 = 20 *
( c ) d = 1,000 feet, p = 10 pounds, 8 = 30' ,

How far can the sled be dragged if the number of avallable


r o o t pounds of work i a 1,000 and if
0
(d) p = 10 pounds, 0 = 10
(e) p = 100 pounds, 0 = 2
0
' .
(f) p a 100 pounds, e = 00 .
(g) p = 100 pounds, e = 89' .
2. The drawing shows a smooth i n c l i n e d feet Long which
makea an angle 0 with the horizontal.
How much work is done in moving an object welghing p pounds
from R to S if
(a) d = LO f e e t , p = 10 pounds, 8 = 100 .
(b) d = 100 feet, p = 10 pounds, 9 = 20' .
(c) d = 100 feet, p = 10 pounds, 0 = 3
0
' .
How far can the weight be moved if the number of available
foot pounds is 1,000 and i f
(d) p = 10 pounds, 0 = 10' .
(e) p = 10 pounds, 9 = 20
0
.
(f) p = 100 pounds, 0 = lo .
(g) p = 100 pounds, e = 89
0
.

Velocity is another concept of physics that can be represented


by means of vectors. In ordinary language the words "speed" and
"velocity" are used as synonyms. In physics the word "speed"
refers to the actual time rate OF change of distance (the k i n d of
informatian s u p p l i e d by an automobile speedometer), and lfvelocity't
refers to the vector whose d i r e c t i o n is the direction of the
motion and whose length represents the speed on some given s c a l e .
When velocities are represented by vectors, t h e l e n g t h s of these
vectors give the corresponding speeds.
F l g u r e 11-5b s h o ~ sv e c t o r s which represent some of t h e
v e l o c i t i e s of a body moving around a circle with constant speed.

'g
Pig. 11-5b
It is easy to lmagine situations In which v e l o c i t i e s are
compounded o u t of other velocities. For i n s t a n c e , a man walking
a c r o s s the deck of a moving boat has a v e l o c i t y relative to the
water which I s compounded out of his v e l o c i t y relative to the boat
and o u t of the b o a t ' s velocity r e l a t i v e to the water, It is a
principle of physics that t h e v e c t o r w h i c h represents such a
compound v e l o c i t y i s t h e sum of the v e c t o r s which represent the
individual v e l o c i t i e s .

Example 11-sf: A s h i p s a i l s east at 20 miles per hour. A


man walks across I t s deck toward the s o u t h at 4 miles per hour.
What is t h e man's velocity relative to the water?
2

S o l u t i o n : In t h e figure, X
represents the s h i p ts v e l o c i t y A
A
relative t o the water, Y
represents the man 1s v e l o c i t y
r e l a t i v e ta o t h e s h i p . Conse-
A
quently, X + Y represents
the man 8s v e l o c i t y r e l a t i v e t o
the water. I t s length is

and its direction i a approximately 2 south of e a s t .

Exercises 11-5c B
A r i v e r 1 mile wide flows a t t h e rate
of 3 miles per h o u r . A man rows
across t h e river, starting at A and
aimlng his boat toward B the nearest
point on the opposite shore as shown in
C
t h e diagram. If it took 30 minutes f o r
him t o make the trip, how f a r d i d he row?
A r i v e r is $ mile wide and flows a t the r a t e of 4 miles
p e r hour. A man rows across t h e river in 25 minutes, l a n d i n g
1.3 milea farther downstream on t h e opposite s h o r e . HOW far
d i d h e row? In what direction d i d he head?
A river one mile wide flows at a rate of 4 miles per h o u r .
A man wishes t o row in a straight l i n e to a point on t h e
opposite shore two miles upstream, How fast m u s t he row
in order to make the trip I n one hour?
A body starts at (0,0)at the time t = 0 . It moves with
c o n s t a n t velocity, and seconds later it ts at the p o i n t
(40,30). Find i t s speed and i t s velocity, If one unit of
length of vector corresponds to 100 feet per second.
A body moves w i t h constant velocity which is represented by
A a
.
2
the v e c t o r V = 1 O i + 10j If the body is a t the p o i n t
( 0 , l ) a t time t = 2 , where will it be when t = 15 ? The
scale 1s: One u n l t of length of vector corresponds to 10
miles per hour; the time t is measured in hours.
Ship A starts from point ( 2 , 4 ) a t t i m e t = 0 . Its
v e l o c i t y is constant, and represented by the vector
.
2 -5

V, = 4 i - 33 S h i p B s t a r t s at t h e p o i n t (-1,-1) at time
t = 1 .It8 velocity is also constant, and is represented
2
by the vector Vb = 71 + j
A
.
W i l l the ships collide?
( ~ a s u m et h a t a consistent scale h a s been used in s e t t i n g ' u p
the v e c t o r representation.)
Ship A starts a t point (2,7) at time t = 0 .
Its
(constant) velocity is represented by t h e vector
2 A
- 2
V, = 31 2j. Shfp B starts at p o i n t (-1,-1) a t time t = 1.
I t s ( c o n s t a n t ) v e l o c i t y is represented by the vector
Vb = 5 1 . W i l l the ships collide?
A A

A river is 1/2 mile wide and flows at the rate o f 4 miles per
h o u r . A man can row a t the rate of 3 miles per hour. If he
starts from point A and rows to the o p p o s i t e shore, what is
the f a r t h e s t p o i n t upstream at which he can reach the
opposite shore? In what direction should he head?
1. Show each of the f o l l o w i n g g r a p h i c a l l y :
(a) 3?+ 87+ 55;. (f) 2r- 23
A
.
(b) 37+ 3T;. (g) G.

a
+k .
A
(e) 5?+
2 A
53 (J) a?+ 87+ 3 r .
2. Find A B , if:
2 2 2 2 2 - A -
(a) A = 3 1 + 2 J + b k ; B = 2 1 + j + 2 k .
+ 47-
2 h 2
. A 2

(b) A = 31 a ;B = 21 + 27+ &.


+ 3k ; 3 = 4 j .
4 A L a A
(c) A = 31
+ 4J ; B = 7k .
2 A - A 4

(d) A = @i
a
+ 2k ; B = 51 .
a A - 2
(e) A = 4j
2 2

3. Find t h e cosine of the angle between v e c t o r s A and B In


each part of Problem 2.
& 2
4. Find the cosine of the angle between the vectors A and B
2 2 - 3
if A = 3 1 + 2 j - k
and g= 4 r - 3 7 + 6.
*5. A l i g h t i n g fixture is suspended as shown:

View from t o p

Side View
(the angle is
shown in its
true shape;
isoan angle of
60 . )
The f i x t u r e weighs 15 pounds. Find the t e n s i o n in each of
the supporting cablea.
6. An a i r p l a n e is climbing a t an angle of 3
0
'
. Its climbing
speed is 100 m . p . h , Although a wind is blowing from w e s t
t o e a s t w i t h a veloclty of 30 m . p . h . , the p i l o t wishes to
climb while heading due n o r t h . What is the g r o u n d speed
of the alrplane?
7. Suppose that in Problem 6 the p i l o t climbs at an angle of
3a0, b u t does n o t i n s i s t on heading north. What is the
fastest ground speed t h a t he can achieve? Which way should
he head to achieve t h i s speed? What is t h e l e a s t ground speed
t h a t he can a c h i e v e ? Which way should he head to achieve
this?
8. Prove that

is the e q u a t i o n of a plane through the p o i n t & ( d , e, f ) r r i th


the normal vector

9. Find a vector normal to the plane

10. Find the distance from the point (0,0,0)to the plane

11. Find the d i s t a n c e from t h e plane

to the origin.

--
11-6. Vectors as a Formal Mathematical System.
In o u r discussion of forces and velocitfes by means of
vectors we made a few assumptions which we d i d n o t J u s t i f y . We
applied vector methods to the s o l u t l o n of force and velocity
problems in a f a s h i o n which turns out to be correct but which we
have n o t backed up w i t h a convincing argument. Our t h i n k i n g was
somethfng l i k e t h i s . "some of t h e r u l e s that forces obey are very
much like t h e rules that vectors obey. Therefore we can talk
about f o r c e s a s though t h e y were vectors. " This is n o t really a
sound argument, and if it were t r u s t e d in a l l cases it could lead
t o chaos. F o r instance, some of the rules that real numbers obey
a r e t h e rules that Integers obey, and it is n o t t h e case t h a t real
numbers can b e regarded as Integers.
Nevertheless, it really was c o r r e c t t o t r e a t f o r c e s as
v e c t o r s and we now explore a p o i n t of view which gfves convincing
evidence f o r this statement. The key fact in t h i s examination is
t h a t e v e r y mathematical system which obeys certain of the laws
which v e c t o r s obey must be essentially the same as the system of
vectors Itself.
We now formulate three goals:
1 , To list the r u l e a In question.
2. To g i v e a precise specification of what we mean by saying that
a mathematical system is ''essentially the same" as a system
o f vectors.
3 . 20 prove t h a t systems whlch obey the s t a t e d rules are
-,::entially t h e same as t h e system of vectors.

I. We s t a t e c e r t a i n rules which vectors have been shown to


obey. We have a set S , two o p e r a t i o n s @ ,a , for which,
f o r all a , 6 , r , in S and for a l l real numbers r , s

(I;) There is a zero element @ in S such that


a@$= a .
( 5 ) Each cr has an additive inverse -a f o r which
(6) r a
a: i s In S .
(7) r 0 (s 0 a:) = (rs) O a .
(8) (r + a ) 0 a = (r O a ) @ ( s a a ) .
(9) r O ( a : @ , d = (r a a ) @ ( r O 4 )
(10) 1 0 a = a .
(11) There are two members y and w of S such t h a t
each a has a unique representation

11. We have already shown that vectors s a t i s f y such rules,


where S is i n t e r p r e t e d as the s e t of v e c t o r s , @ is interpreted
a s ordinary + for v e c t o r s and where a is inte~preteda s
scalar multiplication. We take I t as g i v e n (by physicists
presumably) that forces also satisfy these r u l e a , where S l a the
s e t of forces, a @ /3 means the r e s u l t a n t of a and & and 0
means scalar multiplication, We are to show that forces are
essentially the same as vectors. What do we mean by "essentially
the same?" We mean that the system of forces is isomorphic to
the system of vectors. What do we mean by "isomorphic"? That
there is a one-to-one correspondence between the s e t of forces
and the s e t of v e c t o r s such t h a t , If force 6. corresponds t o
2 2
v e c t o r A and if force @ corresponds to vector B , then
3 A

a @ @ corresponds to vector A + B and f o r c e r Octcorreaponds


A
to v e c t o r FA ,
111. We now state and prove the promised theorem.

Theorem: Any system S which satisfies Rules 1-11 is


isomorphic to the system of vectors In a plane.

Proof: We first s e t up a one-to-one correspondence between


the members of S and the vectors. For each a: of S we Invoke
I t e m 11 to write
a = ( a a y ) @ ( b g ~ ).
The pair ( a , b ) which f i g u r e s in t h i s expression determines a
unique vector A , namely [a,b] , which we p a i r w i t h a
2

A
.
This
process assigns to each a of S a vector A as i t s image. We
muat show that if [a,b] is the image of a and if [ c , d ] is
the Image of@, then [a + c, b -t d ] I s the image of ar -F @
and that [ra,rb] is the image of r a .
a To prove t h e first,
write
a = ( a @ y ) @ ( b a w 1
/3= ( c ~ ~ l @ ( d ~ w ) -
m e r e f o r e ~ @ b =( ( a 0 y )@(bO w ) ) @ ( ( c O Y ) @ ( d o w ))
which equals using Rules 2 and 3,

( ( g oy ) @ ( ~ O y ) ) @ ( ( b Ow )@(dO w )) .
This in t u r n equals

by vlrtue of R u l e 8. We see then t h a t o u r one-to-one correspond-


ence assigns [a + c , b + d ] to d + /3 .
We now examlne rg a . We write
a =rg ( ( ~ @ Y ) @ ( ~ O W ) )
which by Rule 9 can be w r i t t e n as
r@ (aOY)@rQ ( b o w ) .
According t o Rule 7, t h i s last equals
( ( r a ) O)~@ ((rblO w ,
whence the image of r @ a is indeed [ra, rb] .
This completes o u r proof. Notice t h a t we did n o t use all t h e
rules given. They are i n f a c t redundant. If the last rule is
l e f t o u t , the remaining set of rules is not redundant, and is
the s e t of axioms which defines a vector space. The Rules 1-11
a r e axioms for a more special mathematical system--a
-
two-dimensional vector space.
W e have shown that every system which satisfies Rules 1-11
is isomorphic t o o u r system of v e c t o r s . We have n o t shown that
t h e system of forces satisfies these rules. We take t h e
p h y s i c l s t f s word f o r t h i s . We have n o t shown that to be
"isomorphic " r e a l l y means to be "essentially the same. " Let us
meditate a little on t h i s and then take the mathematician*s word
for it.

Exercises 11-6
1. Let S be t h e system of complex numbers. Does S satisfy
Rules 1-11 if @ is interpreted as ordinary a d d i t i o n of
complex numbers and a
as ordinary m u l t i p l i c a t i o n of a real
number by a complex number. ( ~ i n t : In checking R u l e 11
t r y 1 f o r Y and i f o r w ) .
2. L e t S be the s e t of a l l ordered pairs (a,b) of real
numbers, l e t @ be d e f i n e d by ( a , b ) @ ( c , d ) = ( a + c, b + d)
and l e t 0 be defined by

Which of the Rules 1-11 does t h i s system obey?


3. Let S be the s e t of all ordered pairs ( a , b ) of real
numbers, l e t @ be d e f l n e d by ( a , b ) @ ( c , d ) = (-,-),a + c b + d
and l e t 0 be defined by r a
( a , b ) = (ra,rb) . Which of
the Rules 1-11 does t h i s system obey?
Chapter 12
POLAR FORM OF COIdPLEX NUMBERS

12-1. Introduction.
In Chapter 5 we i n t r o d u c e d complex numbers z = x + iy,
x and y real numbers. We found (!Theorem 5-4) that each complex
number z is uniquely determined by i t s "real" and "lmginary''
parts, x and y , respectively; i . e , ,
z1 = x1 + iyl and z 2 = x 2 f iy2 are equal

If and only if xl = x 2 and y1 u y2 .


W e also discussed t h e a d d i t i o n and multiplication o f complex
numbers g i v e n by the formulas:

We found in Section 5-7 that t h e addition of complex numbers


may be described geometrically by means of a parallelogram. In
Section 12-2 we discuss a geometrical description of t h e product
of two complex numbers.
The remainder of thla section polnts o u t some similarities
between the work in Chapters 5 and 11. Exercises 12-1 provide a
review of some of the work in Chapter 5 .
-
Complex Numbers and Vectors. We call a t t e n t i o n to the
important case of Formula 1 2 - l b in which yl = 0:

In view of Chapter 11, t h i s s p e c i a l case appear8 in a new l i g h t .


Note the similarity between Theorem 5-4 and Formulas 12-la, 12-lc
and the statements in Chapter 11 concerning equality, sum, and
scalar m u l t i p l e of v e c t o r s in a plane.
J u s t as two complex numbers are equal if and only if t h e i r
real and imaginary parts are the same, two vectors in a plane are
equal if and only if t h e i r x and y components are the same.
This simflarlty I s more than a coincidence: o u r geometrical
representation of complex numbers I s exactly the ssme as our
p i c t u r e s of v e c t o r s in a plane.

Moreover we add complex numbers just as we add vectors and we use


the same p i c t u r e (a parallelogram) to represent sums in each case.
Multiplication of complex numbers by real numbers, as in
Formula 1 2 - l c , corresponds exactly to t h e multiplication of
vectors in a plane by scalars: we multiply each "componentt'by
.
the real mu 1t i p l ler
We t h u s recognize a kind of i d e n t i t y between t h e s e topics.
It is true that we have used a d i f f e r e n t s e t of x o r d s in what we
have said on t h e s e two s u b j e c t s , b u t o u r f q r m u l a s show t h a t even
with t h i s difference in t h e words we h a v e a c t u a l l y been saying
precisely the same things in t w o different contexts.
Two mathematical systems which a r e t h e same in t h i s sense a r e
o f t e n called abstractly identical or isomorphic. (The word
tl
isomorphic " h a s the Greek roots " i s o , " meaning "same, " and
t'morphos, meaning "shape " or "fo m . 'l See page 680. )
It must be emphasized that o u r isomorphism I s between frag-
ments of these two subjects. The theory of complex numbers and
the theory of vectors in a plane have t h e same form only when we
restrict our attention to the notlon of equality and the operations
of addition and multiplication by a real number ( s c a l a r ) , and t o
Ideas depending s o l e l y on these.

Isomorphism--like analogy--is n o t necessarily complete


identity. Our two systems--vectors in a plane and complex
numbers--differ remarkably, and in two very important respects.
They d i f f e r when it becomes a matter of discussing an operation of
" m u l t i p l i c a t i o n " between elements of the two systems: p r o d u c t of
two complex numbers, and product of t w o vectors. Perhaps t h e most
startling difference between the products i n o u r t w o systems is
the matter of closure. The product of t w o complex numbers 1s a
complex number, the product of two vectors in a plane is - not a
vector in the plane. In the case of the inner product, it is not
a vector of any kind, it is a scalar. Multiplication of complex
numbers is associative. The question of associativity f o r t h e
i n n e r product of vectors f a ludicrous; the very expressions

whose equality is presumably at issue do n o t make sense s i n c e the


factors in parentheses, being scalars, cannot be "dottedt1onto a
vector. Only a v e c t o r can be "dotted" onto a n o t h e r vector.
In C h a p t e r 13, we d i s c u s s e d t h e geometrfc interpretation of
the inner p r o d u c t of v e c t o r s . In Section 12-2 we p u r s u e t h e
analogous q u e s t i o n f o r the p r o d u c t of complex numbers. Such an
interpretation will serve to emphasize the d i f f e r e n c e s we have
been discussing.
The two systems also d i f f e r fundamentally if one attempts to
extend them from a plane t o a space of t h r e e o r more dimensions.
Vectors i n spaces having 3, 4, ...
, 6 N, , . . . dimensions have
very important applications I n physics, chemistry and engineering.
(No is Avogadrots number, 6.02: x ) On t h e o t h e r hand, ex-
tensions o f the system o f complex numbers a r e a b i t bizarre and,
in any case, are another matter entirely. They are beyond the
scope of this book.

Exercises 12-1. ( 8 e v i e d of Chapter 5. )


1. Hrite the f o l l o w i n g In standard form:

(a) 1 (d) 1 + l2

(b) i (4 m
2 3
73 -ni (f> fl
2. W ~ i t ethe conjugate f o r each of the following complex numbers:
(a) 2 - 3i

(b) +i
5
(c) -2 -+ 31
3. If z = a + bl , express z . -z in standard form.
4. &press the q u o t i e n t - in standard form.
Find the absolute value of:
(a) 4 + 31
(b) -2 - 51
(c) - 31
Solve t h e equation 22 2 + z + 1 = 0 .
P l o t each of the following complex numbera in an Argand
~ i a ~ r a: m
(a) 2 + 1. (dl 5
(b) -3 4- 2f (e) -5 - 31

Find the sum, difference, or product as indicated:


(a> ( 2 - 31) + (-4 + 1) (e) 5 + (2 - i)
(b) 3 + 1 ) + (1 - ) (f) 3(2 -- 31)
(c) (2+31) - ( 4 - 21) (g) ( 2 . t 31)(1 - i)
(dl 5 - (4 - 21) (h) (1 + a)(1- 2i)
IF a and b are r e a l numbera, under what c o n d i t i o n s will

Solve each of the following f o r the real numbers x and y :

(a) ( x + i y ) + ( 2 - 31) = 4 - 1
(b) 2(x + iy) - (3 - 21) = 1

id) (a+ i)(8 - LX) = 34


Find the value of by the following alternate methods:
I
(a) is a number x + yi such that

(x + yi)(a + bi) P 1 . From t h i s , obtain two equations


for x , y , and solve them.

Csec. 12-11
I
(b) m may be expressed as a number x + yi in
standard form by maklng use of the conjugate of
a + b i .

12-2. ---
Products and Polar Form.
In t h i s section we consider the problem of a geometrical
representation for the product of two complex numbers. We shall
find some of t h e ideas and methods In Chapter 10 very u s e f u l in
solving t h i s problem. Moreover the i n t r o d u c t i o n of trigonometrical
notions enables us t o write complex numbers in a form particularly
convenient f o r the study of powers and roots g i v e n in the
remaining s e c t i o n s of t h i s chapter.
We know from Chapter 5 that the absolute value of a product
of two complex numbers is the product of t h e i r absolute values:

In view of Formula 12-lb, this follous e a s i l y from the identity

This fact alone tells us something


rather interesting about the p r o d u c t .
Suppose zl represents a p o i n t on
the circle w i t h c e n t e r 0 and
radius rl . Then rl = (zl1 . If

z2 represents a point on t h e
c i r c l e with center 0 and radius r2,
then r p = 1z21 , and the product

z1z2 represents a point on the


circle w i t h center 0 and r a d i u s F i g . 12-2a
'1r2 , since 1z1z21 .
[ s e c . 12-23
Given that zlz2 represents some p o i n t on the c i r c l e , our
-
problem is now to locate which p o i n t on the circle.
We discussed questions of t h i s
kind In Chapter 20. In Figure 12-2b I
we reproduce the fundamental diagram
from Chapter 10, drawn i n an Argand
diagram, As in Chapter 10 we have

X = r cos 9 ,

Fig. 12-2b
. s o , if z = x + iy i s in s t a n d a d form we can write

12-a z = r(cos 9 + i sin e } ,

where T = \zl =AT.


Formula 12-2a, expressing z in terms of r and e enables
us t o s o l v e our problem. Indeed, suppose

z1 = rl(cos B c i sin B ,

If we form the product of these expressions, we obtaen


zlza = rlr2(cus e ,+ i sin 0 (cos 0 + Is i n 0 2)

= r1r 2( ( c o s 9 cos 0 - sin 0 s i n 9 2}

+ l(s1n 8 cos 9 + cos sin e .


Using the addition theorems for coaine and sine (chapter lo),
cos ( e + B 2) = cos 8 coa e - sin e sin e
sln(el + e 2) = s l n e l c o s e 2 + coa el sin e2 ,
we can simplify 12-2b.
Thu EI

Formula 12-2c gives the following geometrical description of


the product of a pair of complex numbers: t o multiply two complex
numbers one multiplies t h e i r absolute values and adds t h e i r angles.

Fig. 1 2 - 2 ~

Formula l 2 - a , expressing t h e complex number z in terms of


-
r and 8 , is called the polar form of z , We have seen that
P o m l a 12-2c gives us a way to deecribe a product in geometrical
terms. We s h a l l also see t h a t t h e algebraic consequences of
these f o r m l a s are extremely important.

Example 12-2a: Multiply 31 and 1 + i , plot t h e product


and the f a c t o r s , and check the result u s i n g p o l a r forms of the
numbers Involved.
Solution: 31(1 + I)
= 31 - + 3i
3 = -3
1311 = 3 , (1 + 11 = f l ,1-3 + 311 = 3 0 -
Polar forms: 31 s 3(cos $ + i sin 5)
I + I=*(COS $ + i s i n f)
-3 + 31 = 3 0 c o s ( $ + $) + i sin($ + $)
= 3*(cos $+ i sln $1 .
Example 12-2b: (1 + i)(l - I)
= 2 .
1 C 5 + 1 s i n g) ,
i =fl(cos

1 - i COB(- I) + i s i n ( - $11 ,
(1 + i)(l - i) = ( J F ) 2 [ ~ ~ -( $$1 + i sin($ - $11
= 2(c03 0 + i sin 0 )
= 2 .

Let us examine the relation between the standard form of z


and t h e p o l a r form of z , ( z # 0) . For the standard form we
nrlte
z = x + i y ( x and y real} ;
f o r t h e polar form we write
z = Izl(cos e + i sin e ) .
Since

the p o l a r form, expressed in terms of x and y , is simply


The polar form of z may be described by saying that it
resolves z into a product of two Sactors: the first f a c t o r
being ( 2 ) , a non-negative real number, the second factor
cos e + 1 sin e = 6+ i &-, which is complex, has absolute
value 1:
lcos e + i sin e I =J(cos e )2 + (sine l2 = 1 .

Example 12-2c: The polar fo m of 1 -t i is

1 1
+ 1 c o + i sin 3,
since
F= i, &=
I
cos K
sin y .
Example 12-2d: The polar form of 3 + 41 is

3 + 4 =
I?) e + i sin e ) ,
5(4 ~ ( C O S

where cos 8 = 53 , Sin 0


4
=3'

These examples i l l u s t r a t e the fact t h a t one does n o t have to


refer to a table of trigonometric functions i n order t o write the
polar form of a complex number when it is given in standard form.
Reference to a table is necessary o n l y f o r determining, or eat--
ting, the value of 8 . We shall see that f o r many c a l c u l a t i o n s
involving the p o l a r form it is n o t necessary to find e i t s e l f
--knowing o n l y cos 8 and sin 8 being aufficfent.
It is clear, on geometrical grounds, that t h e r e are many
values of 9 corresponding t o each gLven non-zero complex number z
However, these values of 0 are related t o each o t h e r in a very
simple way since each of them qeasures an angle from the positive

fsec. 12-21
x-axis to the ray from 0 through the p o f n t representing z. Such
angles differ by some number of complete r e v o l u t i o n s about 0 , so
the values of 8 measuring these angles d i f f e r by i n t e g r a l
.
m u l t i p l e s of 2 9 Each ray from 0 is terminal side of one
angle, having the p o s i t i v e x-axis as initial side, which is
less than one complete r e v o l u t i o n . If 8 is the radian measure of
such an angle then 0 - < 8 ( 2R ; we shall say that e is the
argument of each non-zero complex number z corresponding t o a
p o f n t on the ray. We write 0 = arg z .
Thus 9 = a r g z means
z = r ( c o s e + i s i n e ) , r > O , and
-
O < 0 <a.
S i n c e lcos 0 + i sin e I = & c o a e ) 2 + ( s i n e ) * = 1 , the
complex number cos 8 + Is i n 8 represents a point on the S n i t
c i r c l e 1 ' - - I . @ . , the c i r c l e with center 0 and radius 1 .
Consider two such complex numbers: cos $ -I- i sin $ and
cos 0 + 1 sin 8 . By the remarks I n t h e previous paragraph, we
have the f o l l o w i n g theorem.

Theorem 12-2a: cos jif + i s i n $ = cos e + 1 sine


if and only if
$=e + 2ks , for some integer k .
This theorem may be proved d i r e c t l y from the periodicity
p r o p e r t i e a of t h e c o s i n e and sine functions w i t h o u t appeal to
geometrical ideas. See Exercise 12-2.

Esec. 12-21
Exercises 1 2 2
1. Express each of t h e f o l l o w i n g complex numbers in p o l a r form
and determine arg z :
(a) z = 2+2i

(b) z = -3 + 31 (e) z = 4
(f) z = - 21
2. Fxpress each of the following complex numbers in standard
form a + bl :

(a) 3(cos $+ i sin s3 ) (d) ~ ( c o sT IT f sin ST)

(b) ~ ( c o s ? + i s i n T3iT
) (el c o sTr T + s~ i n 7f3
lla
c o s --b + i sin 7
'lr;- (f) 2(cos o+ i sin O)
(c)

3. Find the indfcated p r o d u c t s in polar form and express them


in standard form:
(a) ~ ( c o s + i sin g)
2~ + i sin 2Tr
- ~ ( c o s-5-
2a 28
(b) (cos ~ ii s -i n (cos !+ i sing)
(c) ( ~ ( C O S $+ i sin $)12
2
9. Prove t h a t [ r ( r o s 9 + i s i n 9 )) ;
; r2(cos 2 9 + i sin 2 0 ) ,
where r is a r e a l number.
5. Prove that, if zl = rl(cos e + i sin 0 , rl real,

and z 2 = r2(cos 0 + 1 s i n 0 *) # 0, r2 real,

then
z
1
---[cos(el
'1 -e2) + i sln(8 -Q23.] .
'2
6. Show t h a t
-z = - if 121 = 1 .
z
7. Show that two non-zero complex numbers lie on t h e same ray
from 0 if and only if t h e i r ratio is a p o s i t i v e real number.
8. Prove Theorem 1 Z 2 a w i t h o u t appealing to geometry; i e , show . .
that

(a) cos $ + Is i n $ = cos B + 1 sin 8


if and only if P( = e + , where
2 k ~ k is some integer .
[Hint: Prove f i r s t that (a) holds if and only if
0
cos e
+
+ f =1 ;; .
R e w r i t e (b) using Exercise 5; equate and imaginary
parts and show t h a t t h e two conditlons you get hold if
and only if cos ($ -
0 ) = 1 .
But the last condltian
h o l d s if and only if $ -
0 = 2 k ,
~ for some integer k .
l
MY?

12-3. I n t e ~ r a lPowers; Theorem of deMoivre.


We saw In S e c t i o n 12-2 t h a t if
z = r(cos B + 1 sin e )
and z t = r*(coa 0 t + 1. sin 0 I) ,
then zzt = rr1[c0s(8 + 8 ') + 1 sin( 9 .t 8 r)] .
We now turn to the caae where z and are e q u a l and
obtain f o r the square of a complex number

z 2 = r2 ( C O S 2 e + i sin P 0 ) .
W e can extend the idea to

. z3 = ,%
2
= [r (cos 2 e + i sin 2 8 )][rfcos e + i sln 8 )]
3
= r (cos 3 e + 1 sin 3 8 ) .
Continuing in t h i s way, we may derive, one a f t e r t h e other,
similar formulas f o r z 4 , z 5 , z 6, , ..
, zn , for each natural
number n .
The theorem of de Moivre states the general result.

Theorem 12-3a: (de ~ o i v r e ) If z = r(cos 8 + 1 sin 9 )


and n is a natural number, then

zn = rn(cos n 8 + Isin n B 1 .
Let us t u r n now to some s p e c i a l i n s t a n c e s of t h i s theorem
to see what it has to tell us about the geometry of the complex
plane,

&ample 12-3a: Find a l l p o s i t i v e I n t e g r a l poiiers o f I .


Solution: We have ( 1 1 = 1 and arg i = a Thus .
1
2
= ~ ( C O SP + i sill 8) = -1 f Oi = -1 .
1
3 = l(cos 3~ + 3-ir
1 sin =)= 0 - i = -i

l(eos 2 a + i sin 2r) = 1 + 01 = 1 ,


1
Prom here on the powers repeat: 3. 5 = i4 I= 1 , i6 = i 4i 2 = -1 ,
1 = l l3 -- ,
- A 18 = I4 1
4 = 1 , etc. Wecanexplainthis
r e p e t i t i o n in geometrical terms by noting t h a t each time the
exponent is increased by 1 , in steps through a quadrant of t h e
unit circle. We can express t h e s e fact8 compactly by w r i t i n g

where n Is 0 or any natural number.


Exam le -
2 12-3b: Let z = cos 1 t. I sFn 1 . (The angle with
radian measure 1 has degree measure -
180
'TT
, uhich is approxi-
mately 57.3 degrees.) Plot the f i r s t t e n powers of z . WhZch
quadrant contains the point r e p r e s e n t e d by z100 ,
Solution: Since

z = cos 1 + 1 sin 1 , de Molvreis Theorem g i v e s

These numbers a l l have a b s o l u t e v a l u e 1 and hence represent


p o i n t s on t h e u n i t c i r c l e . The length of a x a l o n g the c i r c l e
between successive powers of z is 1 u n i t . To determine the
quadrant c o n t a i n i n g z loo we may f i r s t determine how many complete.
Pig, 1 2 - 3
c i r c u i t s o c c u r as n steps from 0 to 100 . D5vidlng 100 by
we obtain 15.915. .. + Thus zn steps through 15 complete
r e v o l u t l o n s and over 3 of a h a n s t e p s f m m ~to 100.

Therefore, zloo represents a p o i n t in t h e f o u r t h quadpant. ~


Example 12-3c: - Calculate and p l o t the first five powers of
z = l + i f i .
Solution: Here lzl = fi
--
t- 3 = 2 , arg a
z = .g . SO
I

and

z 3 = 8(cos s + i s i n T) 8(-1 + 0 . I) = - 8 .

z
4
= 16(cos
4a
+ i sin T)
4~
= 16(- - i+)=-8-81&

z 5 = 32(cos 9+ i sin q) = 32($ - i = 16 - 161 J5 .


Using the theorem of de Moivre (Theorem 12-3a) and the
formulas
2
( a + b)* = a 2 + a b + b ,

we may derive an endless list of identities of the f o l l o w i n g kind:


12-3b s i n 2 0 = 2(cos 8 ) s i n 8 ,
2
12- 3~ cos 3 e = (cose13 - 3 cos e ( s i n e ) ,
12-3d s i n 3 8 = 3 ( c o s 8 ) 2 Sin 9 - ( s i n e )3 ,
12-3e cos 4 e = ~ ( C O S e)4 - ~ ( C O S8 )
2
+ 1 ,
12-3f s i n 4 0 = 8(cos e ) 3 sin 0 - 4(cos 8 ) s i n 0 ,
and corresponding formulas f o r cos 5 8 , sin 5 8 , cos 6 9 ,
sin 6 , ...
, c o s n e , sin n 8 , where n I s any n a t u r a l
number. We call such Identities multiplication formulas.
We may prove the i d e n t i t i e s 12-3c and 12-3d as follows:
3
cos 3 e+ i sin 3 8 = (cos8 + i sine)
= (cos 9 ) 3 + 3(cos e ) 2 (i sin e )
+ 3(cos 0 )(i s i n l2 + (i s i n el3
= ((case l3 - ~ ( c o se ) ( s i n 8 )
2

= 1(3(cos 0 l2 sin 0 - (sin 131 ,


and, equatlng real and imaginary p a r t s ,
cos 3 0 = (cos 0 l3 - 3 cas (sin 9 )
2

sin 3 0 = 3 ( c o s 0 1 * sin - (slne)


3
.
We leave the proofs of the other multiplication formulas as an
exercise.

Exercises 12-3

I n each of t h e exercises 1 through 5,


(a) Find 1z 1 , arg z , and express z in p o l a r form.
(b) Using the polar form found in Step ( a ) , calculate
4
2
, Z
3
, z .
(c) Check the results obtained In S t e p (b) by calculating
z ,z 3 z4 using the standard farm.
Id) Show in a diagram each of the points 1 , z , z2 , z 3 , z4 .

6. Deduce de Moivre 1s theorem f o r negative Integral exponents


from the verslon stated in the t e x t for exponents which are
natural numbera .
7. Prove the m u l t i p l i c a t i o n formulas 12-3a, 12-3b, 12-3e, 12-3f.

12-4. Square Roots.


The theorem of de Moivre in S e c t i o n 12-3 provides a compact
formula f o r any integral power oP a non-zero complex number:
Sf z = r(cos 8 + i sine) ,r) 0 , and
n is any integer, then zn = r"(cos n 8 + Isin n 0 ) .
In Sections 12-4 and 12-6, we consider t h e converse problem:

Given a complex number z and a natural number


n , to find a l l complex numbers w satisfying
the equation wn = 2 .
Sectfon 12-4 is devoted to the case n = 2 . Section 12-6 contains
t h e general theorems.
We recall t h a t every non-zero real number has t w o square
roots. I? x i a a p o s i t i v e real number, its square r o o t s are

[aec. 12-41
real; we have denoted them by 4 and - ~ j i ( ~ j i t h e one
being
which is positive, -
f i the negative one). In Chapter 5, we
extended this notation t o cover square roots of negative real
numbers. Thus for x < 0 ,
f i =i J r x T , -&= - im.
Let us consider the equation
W
2= z ,

where z is a given non-zero complex number (which we may suppose


I s not real). We a r e Interested In the s o l u t i o n s e t of t h i s
equation. L e t us assume that the s o l u t i o n s e t is n o t empty and
write
w = Iwl (COS $ + i sin $1
for one of I t s elements. If z = fzl (cos e + i a i n g ), we have,
2
by de Moivrets theorem and the assumptfon w = z ,
lw12 (COB 2 $ + Isin 2 j d ) = l z \ ( c o s 0 + i sin@) .
Equating absolute values we have

so that

Note that Iwl is uniquely determined: l z l is a positive real


number and lw 1 , being a p o s i t i v e real number, is its positive
aquare root. Knowing Iw 1 , we m u s t still f l n d $ in order to
get w . We have

cos 2 $ + 1 sin 2 j3 = cos e + 1 sin 0 ;

hence, by Theorem 12-2a,


2 j$ = 0 + 2 ka , for some integer k ,
or
$ =
1 e + k~ , for some integer k .
Now was given, and if 8 = a r g z then we know 8 t o o . Moreover,
z

-
0 ( 8 ( 2~ .
If we auppose $ = arg w , the restriction
0 ( @ < 2~ limits the possible values of the integer k . Indeed
-
( 8 < 2~ , we have
k can only be e i t h e r 0 or 1 ; for, w i t h 0 -
1
0 -( ~ e a< n d~i f

but if

We therefore find p r e c i s e l y two candidates for elements of


the s o l u t i o n s e t of the equation
2
W = z t z given, n o t zero.

R l e y are u0 = m[ c o s ( ? e
1
+ 0
1
T) + Is l n ( ? e + 0 . T)]

The q u e s t i o n s t i l l remains whether or n o t the s o l u t i o n s e t is


empty. As a matter of f a c t it is not, and both of our
candidates are members of it. To see t h i s we have to show that
they satisfy t h e e q u a t i o n . W e u s e de Moivrets theorem:

= lzl (COS e + i sin e )


and wl 2 = 1
( c o ~ ( ~+ e1 . T) + 1
1 sin(g e + 1 . a)])2
= lzl {COS( 0 + 1 - 28) + i sin( B + 1 21~))

= 121 (COS 0 + Is i n e)
= z .
These conclusions are summarized in the following theorem.

Theorem 12-4a: The solution s e t of the equation


W
2 = 2 ,
where. z is a given non-zero complex number, I s

where

and 8 = arg .
Three observations:
1. If z happens to be real, t h i s theorem agree8 with the
results in Chagtera 1 and 5. If z ) 0 , then 1z 1 = z and
arg z = 0 , so that

If z ( 0 , then 121 = -2 and arg z = T , so that


w0 =m[ c o s1( ? ~ + 0 r) + 1
i sin(? a + 0 r)] . =m( I )=fi
2, The roots wo ,w a r e additive inverses of each o t h e r :
ill = ~ [ C O
1 S +( 7~) ~+ i sin(50 + li)]

=m[- 1
C O S ( ? ~ ) - i ~ i n 1( - ~ e ) ]
-
- - W
0 '
o ow ever, -
I t would n o t be correct t o say that one
of them has t o be n e g a t i v e . Why?)
3 . If z = 0 , t h e s o l u t i o n s e t c o n t a i n s only one element.
That element is 0 , for w 2 = 0 if and only if w = 0 .
Example 12-4a: Find t h e square roots of i .
n-
Solution: Since li 1 = 1 and are i = 7
r:
, t h ~t h e o r e m g i v c
wO = COS($+ o T ) + I s ~ n ( $ +o - n) = -
1 + i
fl
,

Check: a

F i g . 12-4a
'rr
Note t h a t 2 arg wo = P (K ~ )= 2 = arg i ,
e 5.rr)
a r g wl = 2 ( T =
5a = 2r + 7D = 2~ + arg i .

-
Example 12-4b: Plnd the square roots of
12 + 51 .
Solution: Let

z =
12 -F 51
7 ,
; then lz 1 =
13 and t h e polar form of z is
13 12 + i .
( = $(cos e + i sine

Since cos 9 =
12
> 0 and sin 0 = &> 0 , 0 measures an angle
in t h e first q u a d r a n t . W may get an estimate of 0 by consulting
a t a b l e of cosines or s i n e s , Dividing this estimate by two we
igould have an estimate f o r the argument of wo Re-entering the .
8 B
t a b l e we c o u l d get estimates for cos 3 and sin 7 and using
them we could obtain an approximation to and hence also an
wo
approximation to wl = - wo .
The f a c t of the matter is that we
need n o t settle f o r such approximations t o square r o o t s . We can
c a l c u l a t e them exactly! For t h i s purpose we use the 'half-angle"
formulas of Chapter 10.

cos 5 = +<-, e =
sin 7 + $ .
- 2 i

( R e c a l l t h a t the choice of s i g n s in these formulas is d e t e m f n e d


by the q u a d r a n t c o n t a i n i n g %
. ) Returning to o u r example
e = a r g 1 2 + i lies in the first quadrant; hence is also in
the f i r s t q u a d r a n t . Thus cos
8
and sin 8 are both positive.
We g e t
Hence
wO 4 (COB
0
7 +i B
sin g )

fl
T( 5 m
i, in polar form,
+

= 5+ 3
1 In standard form.

The two square roots of 12 + I are therefore and 5 + 12


"

In each of Exercises 1, ...


, 6 find the square roots of the
'given complex number z .
Check your answers by squaring. P l o t
z and i t s square roots in an Argand diagram.
1. 2 ~ 4 . 4, z = f l - L
2. t = 2 + 2 i . 5. ~ = 3 + 4 1 .
3. z = -9 . 6. z=fl+IJT.

=JFT 1
[ c ~ a e( ~ ICT) + + i sin($ e +
7. Let wk k ~ ) ] . show that

= Wo and w&+~ = w1 for any natural number k .

12-5. -
Quadratic Equations with Complex Coefffcients.
We announced in Chapter 5, Section 5-9, that each quadratic
equation with complex coeff'iciants has complex roots. In t h i s
section we prove t h a t t h i s I s the case.
C o n a l d e ~the equation

12-9 Az 2 + B z + C = O ,

where A, B, C are given complex numbers (some or all of which may


be real), and A & 0 .
Completing the square, we have
In Chapter 5, where B, C were real, t h e right-hand member of
A,
Equation l Z 5 b was a real number. In the present case, it is a
complex number (perhaps real, perhaps n o t ) . Let us w r l t e

and

Then Equation l2-5b becomes

There are two cases: (1) E = 0 , and (2) E f 0 .


If E = 0,
whlch means B~ 4AC 0 - , the solution s e t of Equation 1 2 - 3 is
the set (0) . Since z = - B +w , the a o l u t l o n s e t of Equations
12% and 12-5b is the s e t ( ] . Thus in Case (11,
B* - ~ A c= 0 , Equation 1Zs has just one solution, - B .
In Case (2), 'B -
4AC # 0 , Equatlon 12-5c has two solutions,
say wo and wl .
We h o w , however, from Section 12-4, that
w1 = -
w O . Thus we may wrlte (wo , wo] -
f o r the solution
s e t of Equation 12-5c. The s o l u t i o n s e t of Equation 12-5a is then

wO being one of the solutions of

We s t a t e these results a s a theorem.


Theorem 12-ga: The s o l u t i o n set of t h e equation
Az 2 + B z + C = O , * + O ,

is [- 3 if B Z - ~ A C 0 ; if B~ - 4AC # 0 , the s o l u t i o n

s e t is B
(-,+w,, - 3
- wol ,
where wo I s either of the s o l u t i o n s of the equation

Example 12-s: - Solve z


2
- (2 + 4112 -t 41 = 0 .
Solution: Here

A = 1 ,B = -(2 + 41) , C = 4 , 'B - 4AC = 1 + 21)


2
-161 = -12 ,E = -3 . Let wo = i f l ; then
Zl =
B +
-= w,= 1 + 21 + = l + (2cJS)i

z 2 = ' ~B - W O = 1 + 2 1 - l o =1 i-(2 -8)i

Example 12-5b: Solve z


i?
+ (1 + i)z + I= 0 .

-
Solution: Here
A = 1 , B = I + i , B * - ~ A c = (1+1)~-41=21-41
= - 2 , E = - 1 1 38 3a
= ?(COB '2 + i a i n *)

Hence w
O
=l ( c o s
J7
3?T
+ 1 s i n T )=
37Y
- 1 + 1 , and

Example 12-5c: Solve t h e equation


z + (1 - 5112 - (12 + 51) =
2 o .

Esec. 12-51
Solution: Here

-
A = l , B = l - 5 1 , C = - 1 2 - 5 1 , ~ ~ - 4 A C = ( 1 - 5 1 ) 2

+ h(12 + 51) = 24 + 10i ,E = . In Example 12-4b, we


found t h a t t h e s o l u t i o n s of w 2 = . +12 + i are 3 e , .. 53-1 .
Taking wo = 5 + 1 , we have
B
Z ~ S - = + W , = *+-= 2 + 31,

(compare w l t h S e c t i o n 5-9 of Chapter 5 , where t h i s quadratic


equation was mentioned.)

mercises 12-5
S o l v e the following equations:
1. z 2 - i z + 2 = 0 . 4. z2 - ( P C 21)z + 21 = 0 .

*7. z
4
2
- iz2 1 - 31 = 0
+ .
(use half-angle formulas to obtain
z , t a b l e s to g e t approximations for z .)

8. ( z 3 . IZ
2
- (I + 2i)(z 2 - I=) {IZ + 11 = o - .

----
12-6. Roots of O r d e r n.
In Section 12-4 we discussed the aolution of the equation
2
w = z , where z is a given complex number. In thLs s e c t i o n we
consider the equation w" = z , where z is a given complex
number and n is a n a t u r a l number. Ffrst we r ( n s l d e r the case
n = 3 , and later we extend o u r results to an arbitrary natural
number n .

[sec. 12-61
For z = 0 , the equation w n = z has only one solution,
which is 0 . (Why?) We a h a l l find t h a t there are n distinct
roots when z 0 .
--
Cube roots.
Consider the equation

where zis given. Suppose z # 0 .


We proceed as we d i d in
2
Sectfon 12-4, when we discussed the e q u a t i o n w = z If .
w = Iwl (COB $ + i s i n $)
is in the solution s e t of Equatlon 12-6a, then
)w13 (COB 3 jd + Isin 3 $) = l z l (cos e + i sin e )
so lwI3 = ~ z I , cas 3 $ + i sin 3 $ 5 cos e + i sine;
and iwl = 3m,
3 $ =e + k 27r , for some fnteger k ,
or ~ ~ l = ~ m , ~ = + + k - $ I ,. k ~

Note t h a t 3m
is the (real) cube root of the p o s i t i v e
real number Iz 1 ; it I s therefore positive. We propose to show
that, if 0 ( 9 < 2~ , we have 0 - ( $ ( 2a if and only if
-
k = 0, 1, 2 . (compare t h i s to the analogous situation in
Section 12-4.) Indeed, for
square roots, d e M o l v r e f a theorem showa that each
As w i t h
of the numbers

WO = 3m[ c o s ( 1g e +o . + i s ~ 1n ( +~ oe T2~) l

r e a l l y is a member of the solutlon set. We summarize these


results as a theorem.

*
Theorem 12-6a: - The s o l u t i o n s e t of the equation
W
3 = z ,

where z is a given non-zero complex number, is

(w0 9 W1 9 w21 ,
where
1
wo = 3~ +o
[ C O S ( ~ ~ . 2T) + 1 [ ~+ oo
Is ~ n a) )
T

and 8 = arg .
&ample -
12-6a: Find the cube roots of 1 = 1 + 0 i .
Solution: I-Iere 121 = 2 and arg z = 0 . Our formulas g i v e
= COS(O + 0 527)t + i sin(^ +o

= cos
2a +
3- 1 sin 7 -1 +
2~ = 7 iJ3.

4w + 1
= cos 7 sin ~ T T -1 - 143
- 7 = 2 -

~ i g .12-6a
check: 13 = 1 ,

There is a very important connection between the r e s u l t s


obtained in Example 12-6a and the Formula 12-6b, L e t u s give
names to t h e s p e c i a l numbers

the three cube roots of 1. If we put


&/= w,
then

and rJ is the o t h e r non-real complex cube root of 1 The t h r e e


cube roots of 1 are, t h e r e f o r e , , 2 ,
w3 , s i i n e e u = 1
i .
The connection between the cube r o o t s of 1 and t h e cube roots of
any complex number z is g i v e n in the f o l l o w i n g theorem.

Theorem 12-6b: If z is not zero and w is any one


3
solution of the e q u a t i o n J = z , then t h e o t h e r two s o l u t i o n s
are u;:'and w 2w .
We give two p r o o f s of Theorem 12-6b. The first proof,
which involves less computation, accomplishes all t h a t is
a c t u a l l y requlred. The second proof exhibits explicitly the

-
r e l a t i o n s h i p between the Formulas 12-6b and the much more compact
2
expressions i d , ww, LQ w .
F i r s t Proof. Our f i r s t assertion is that the t h r e e numbers
1 ,&,LO K d l st l n c t .
This is evident on the grounds t h a t no
p a i r o f them have t h e same r e a l and imaginary p a r t s . Moreover,
2
it is impossible f o r any two of the numbers w ,ww ,m w to be
equal if ur3 = z # 0 .
For, on the c o n t r a r y , ue s h o u l d have

(say) or 1 = U since w cannot be zero. Thls c o n t r a d i c t s the


f a c t t h a t 1 and M/ are d L s t i n c t . We know then, that w ,w w ,
and w 2~ a r e t h r e e dlf f e r e n t numbers. We propose to prove t h a t
each of t h e s e numbers satlsfles t h e equation w 3 = z .
Since they
are three d i f f e r e n t numbers they must be the three elements of the
s o l u t i o n s e t , f o r t h a t s e t c o n t a i n s only three elements altogether.
By hypothesis w3 = z . Now, in a d d i t f o n ,

( . w w ) ' = w w 3 3 = w3 = z , f o r w 3 = I ,

and ('w 2w ) 3 = ,hW3 = = 3 = , 1Zr3


so & w and w 2w also satisfy the equation.
Second Proof: We show that
2
wO = w1 and ~ t wO
/ = w2 ,
and leave the o t h e r possibllitles as an exercise for the student.
Recall that

w, = 3m[ c o 1s ( ~ )+ i s i n ( + e ) l ,
2~ 2 4~
W = cos 3 f I S i " 3 =21~
- , ,
f' " C O " ~ + i " i
47r
" ~ .

We have, then,
w w, = 3m[ ~ o 1s ( ~ e+ )i s i n ( + e ) l (COS
2r
+ i sin $1
= 3~
1
[ ~ o s ( ~ e + + i
1
e + T2R) l

This theorem tells us a great deal about t h e geometry of cube


roots. It is the analogue f o r n = 3 of the fact t h a t wl = "0 -
when n = 2 .
We know t h a t the cube roots of z l i e on the
circle with c e n t e r at 0 and r a d i u s 3m
One of them, the.
one we call wo , has argument one-third of the argument of z .
Since the other two are u / w o and# 2wo , we can see at once t h a t
the three cube r o o t s are equally spaced around t h i s c i ~ c l e . T h i s
is c l e a r when z = 1 1 ,w
and the r o o t s are , ,. For any
other value of z we have merely to add
1 9 , or 1 arg z ,
3 to

the arguments of 1 ,w , w2 to get the arguments of wo , wl ,


w2 , respectively. Since we add the same quantity t o each of these
arguments we turn t h e whole configuration around 0 by the
amount added.

----
Roots of order n .
We now extend t h e r e s u l t s obtained for square r o o t s and cube
r o o t s . We give theorems for t h e roots of the e q u a t i o n

where n I s any natural number. The student will note that sub-
stituting 2 and 3 for n In the theorems and t h e i r proofs
gives us the theorems obtained f o r square roots and cube roots as
well as thelr proofs. Thus there is no new idea in the remainder
of t h i s section; we merely carry over what we did before to the
general case.

-
Theorem 12-6c: If n is a natural number and z is a
given non-zero complex number, t h e solution s e t of the equation
w n p z ,

Proof: By de Moivre *s theorem, each of the numbers


wk , k = 0 ,1 , ... -
, n 1 , belongs to the solution s e t since

= [coa( e +k 27~) + i sin( 0 + k . 2~)]

= lzl (COS e + i sin 8 )


= 2 .

Moreover, they are all d i s t i n c t f o r no two have the same argument.


On the o t h e r hand, suppose w belongs to the s o l u t i o n s e t and
that jd = arg w .
We must show that each element of the s o l u t i o n
set is one of the numbers wo , W , . . . , W n m l . We assume,
then, that nn = z . This implies

IwI" = 121 and n$= e + k . 20,


f o some
~ integer k . Thus

Iwl = " ! and


1
$ = ; B + k - -
2a
n '
Now 0 ( 8 ( 2rr
I
and 0 2 k <n - 1 give

while 0
-
( 9 < 2r and k
-n
) give

and 0
-< 8 ( 2a and k
-< -1 give

This theorem shows that each non-zero complex number z has


n distinct complex n th r o o t s , where n is any n a t u r a l number.
The complex number 0 has only one n " root f o r each n a t u r a l
number n .
It is 0 .
The n complex n th r o o t s of the number 1 are c a l l e d the
---
nth r o o t s of u n i t y . Let

so t h a t N
is a particular one of the nth r n o t s of u n i t y .
De Moivrets theorem shows ua that the n -
1 o t h e r n th r o o t s of
unity are

thus f o r k = 1 2 , ,
3 , ... , n we obtain precisely the same
r o o t s g i v e n by Theorem 12-6c on p u t t i n g z = 1 , 9 = 0 .

[sec. 12-61
The next,theorem generalizes a result we found in the cases
n = 2 , 3 .

-
Theorem 12-6d: If w is any one of the roots of the
equation wn = z ,
where z 0 ,
then the solution s e t of the
equation may be described as
(w ,N W n-1
, w & . * * * . w ~1 ,
where#= 2~
C O S ( ~ )+ l
27~
.
2
Proof: No t w o of the numbers 1 , y , w , , . ,
n-1
.
can be equal because their arguments are, respectively, 0 ,
-2lf
n, 2 . - 2~ 2~
n , 3 * y , --., n - 1) 2u and no t w o of these
arguments are equal, Hence no two of the numbers

can be equal; for otherwise w would have to be zero, which is


impossible. We know, by Theorem 12-6c, t h a t Equation 12-6c has
exactly n r o o t s ; we complete t h e proof by showing t h a t each of
t h e numbers 12-6d is a s o l u t i o n of wn = z .
But this 1s easy,
since
= 1 z = z ,
(&"W)" = (wn)%n
k

f o r any integer k .
This theorem extends to the general case the r e s u l t s w e found
for cases n = 2 , 3 on the location of the roots. A l l the roots
lie on t h e c i r c l e with c e n t e r 0 and radius
1
nm
. If
8 = arg z , one of these roots has argument 8 ; the other
r o o t s are located at equal distances around the c i r c l e . F o r
n ) 2 , the n r o o t s , t h e r e f o r e , represent t h e v e r t i c e s of a
regular polygon of n sides inscribed in the c i r c l e . It there-
fore s u f f i c e s to locate one of them--say wO , a f t e r t h i s t h e
p o s i t i o n s of all the o t h e r s are determined.
Exercises 12-6
In each of Exercises 1, ...
, 7 f i n d the cube roots of the
given complex numbers.
>I. 2 = 2 .
* 2. z = -2 .
3. z = i .
4. z = -i .
5. z = i - 1 .
.
6. z = 3 + 4i (Use tables to obtain approximations to the cube
roots. )
*7. z = 1 + 1 .
(Do - n o t use t a b l e s . )
8. S o l v e the e q u a t i o n s
4
(a) x = - I .

(b) x6 - 1 = 0 .
(c) x3 + (6 + 6 6 1 ) = 0 .
9. Using tables find the Qth r o o t s of
1 6 ( c o s 164' + i sin 164') .
10. Show t h a t the sum of the n nth roots of u n i t y 1s zero.
"11. Find complex r o o t a of each of the f o l l o w i n g equations:
n
(a) zn + zn-l f. n- 2 c . . . + z 3 + z 2 + z + 1 = 0 ,
where n is a n a t u r a l number;
(b) 2" - z n-1 + ,n-2 - Z 3 + Z...-
2 - Z + l = O ,
where n is an even n a t u r a l number.

S e c t i o n 12-7. Miscellaneous Exercises


1. Prove that y= and ur;, = ,2 where ull and u2
are t h e two non-real cube roots of unity.
2. Express in polar form:
( a ) -3 +gi. (c) c o s 217' - 1 sln 217'.
(b) -2 - 2 1 . (d) 0.5532 - 0.8293 i.
(e) The conjugate of r(cos g + i sin e ) .
(f) 7(- cos 25' 35t + i s i n 25' 35') .
(g) cos 182' + i s i n 358O .
(h) cos 23'+ 1 sin 32' , (use tables.)
3. mpress in polar form and perform the i n d i c a t e d o p e r a t i o n s :

(COS 137' + Isin 763')(eos 317' + i s i n 223')


(C)
cos (-30') - I sin 330'

(e) (cos 1
0' + i sin l ~ ~ ) ( c o 1s
5' - Is i n 1
0
'
) .
4. Simplify the product 1 * W - d 2 .w 3 . . .w n-1 1
(a) when n is even.
(b) when n is odd.
5 Let z be a complex number and w a non-peal cube root of
unity. Show t h a t t h e points z ,W z ,%, form an e q u l -
lateral t r l a n g l e on the Argand diagram.
6. Fxpress a s a function of z and n the length of one side
of a regular n-sided polygon Inscribed in a c i r c l e of
radius lzl , where z is a complex number.
7. Find a l l the roots of each of the f o l l o w i n g equations:
( a ) x4 - 2 - 2 1 = 0 .

(b) 87.6 + z
$ = 0 .
Chapter 12
APPENDIX

We have seen that if z is any non-zero complex number,


8 = arg z , and m is any integer, then

In Section 12-6 wn = z , where z is a


we studied the equation
given nonczero complex number and n I s a glven n a t u r a l number;
we found that the solution s e t consists of the n distinct numbers

The results in Chapter 9 tell us t h a t If a 1s a p o s i t i v e


real numbep, there is a unique p o s i t i v e real number b such that
bn e a . We write, in this case, b = a . noreaver
= , entitling us to write am/" to denote either
of these numbers. We also b o w from Chapter 9 that t h e famlliar
"laws" of integral exponents carry over entirely to these
n
frac t l o n a l " exponents .
In t h i s s e c t i o n we propose t o consider the question of
tl
f r a c t i o n a l n powers of complex numbers. It should be apparent
a t t h e outset that our task is much more involved in the complex
case than it was in the real case--if only because we have n
r o o t s of order n here instead of just one.
The first step in any study of rational powers is to give a
meaning to expressions of the form "1'2 , We cannot simply say,
-
as we could in the real case, t h a t it I s the s o l u t i o n of the

equation wn = z ; f o r there are too many solutions (more than 1


if n )1) .
To put the matter rather bluntly, the relation
wn = z does n o t define w as a function of z . In order to
c o n s t r u c t some s o r t of f u n c t i o n in t h i s context, we are therefore
forced to shift our point of view. We have a correspondence here,
but in order to obtain a f'unction we must f i r s t settle the
questions of what are i t s domaln and range.
Let us take the simplest case first; we therefore consider
2
t h e equation w = z .
We hope eventually to find how w can be
considered t o depend on z .
In order to understand t h i s relation,
however, we shall first turn it around and investigate in some
d e t a l l how z = w2 depends on w . By shiftlng o u r attention to
t h i s more familiar s i t u a t i o n we can l e a r n much that wfll h e l p us
in d i s a d n g the more complicated "inverse " relation. We now
have a f u n c t i o n z = w2 to work with. For our study of t h i s
f u n c t i o n l e t us draw two pictures.

F i g . 12-7a

[see. 12-71
In t h e ffrst we shall plot the complex number w and in the
second we shall p l o t z .
The functional ~ e l a t l o n s h i pbetween
these variables can be described by determining which points in
our "2-plane'I correspond to given p o i n t s in o u r '%-plane. II
Let us trace some of the pairs in t h i s correspondence. For
instance, if w = 1 , the corresponding point in t h e z-plane is 1 .
If w i a real and greater than 1 (on the "real If axis in the
w-plane to the r i g h t of 11, so is its 'image'' in the z-plane;
indeed it l a further away from 1 since 1 z 1 > Iw 1 If

z = w2 and l w l ) 1 . The image of any p a i n t in the w-plane


outside of the u n i t c i r c l e lies o u t s i d e the u n i t c i ~ c l ein t h e
z-plane f o r 121 > t w l > 1 If I w l > 1 . Also each point In
the w-plane and inside the u n i t c i r c l e corresponds to a point
i n s i d e t h e unlt circle of the z-plane. Finally each point on the
unit circle of the w-plane corresponds to a point on the u n i t
circle of the z-plane. Our discussion of the correspondence has
taken into account only t h e absolute values of w and z so f a r ,
and may be considered the geometrical versfon of the statements:
If z = w2 , then l z l ) ( w l for lwl > 1 ,
~ z I= I w l for twl = 1 ,
IzI < I w l for lwl ( 1 .
To complete our picture, we consider arg u and arg z .
We may do t h i s by t r a c i n g the image in the 2-plane of a point
moving around the unlt c i r c l e of the w-plane. The image p o i n t
moves meantime on the u n l t circle of the 2-plane. Note that as
w makes the trip t h r o u g h t h e f i r s t quadrant on its u n i t c i r c l e ,
going from 1 to i , z manages to travel through both t h e f i r s t
and second quadrants, going all the way from 1 through i and
on to -1 (Figure 12-7b) ,
Pig. 1 2 7 b

L e t w continue, passing through the second quadrant on i t s


u n i t c i r c l e . When t h i s happens, z shoots on around i t s unit
.
c i r c l e completing a f u l l c i r c u i t ( ~ l g u r e12-7c )

Fig. 1 2 - 7 ~
0
What happens next is a good question. It depends on how you
choose to describe it. If w keeps going, passing through the
t h i r d quadrant, z will shoot along through i t s first and
second quadrants again. And f i n a l l y as w passes through I t s
fourth quadrant, z will race through i t s t h i r d and fourth
quadrants; and, lapping w , they will come into t h e i r respective
points 1 together. T h i s is the t r o u b l e . If z went through
each of the p o i n t s on its u n i t c i r c l e exactly once a s w makes
i t s f u l l c i r c u i t there would be no problem; there would be just
one value of w corresponding to each of these values of z ,
and we would have a f u n c t i o n w = f ( z ) to talk about. As I t is
we have not g e t g o t such a f u n c t i o n since each z gives rise to
a p a i r of w @s .
We can g e t around this d i f f i c u l t y by a trick--at least It
was a t r i c k when it was introduced about a hundred years ago.
But no trick can remain a t r i c k for a hundred years--certainly
not one as good as t h i s . It has become q u l t e a respectable method
since I t was introduced and has come to be considered one of the
most important methods f o r treating q u e s t i o n s of t h i s aort.
Our trouble amounts to the f a c t that we have to use the
poin6s in the z-plane twice to describe a t o u r such a s the one
consideroed. Suppose then that we use two z "planes" going
through each of them just once. Can we do t h i s somehow? The
famous German mathematician Bernhard Riemann found t h a t we can,
provided we are s u f f i c i e n t l y ingenious about it. He visualized
the "twott z "planes arranged as follows : We "cut " each of them
along the positive real a x i a and then "glue" them together in
criss-cross f a s h i o n aa shown (Figure 12-7d). The resulting
c o n f i g u r a t i o n is an example of hat we c a l l a Riemann Surface.

Fig,
Our object in all this is to obtain w as a f u n c t i o n of z
if w 2 = z , Can we do it now? Again l e t w t r a v e r s e its unit
circle. This time, however, let us lmagine z as moving on the
Riemann Surface. Very well, when w is 1 , z is 1 . As w
moves through t h e first quadrant on its u n i t c i r c l e we imagine z
as moving halfway around i t s u n i t circle in one of the sheets of
i t s Riemann Surface. As w passes through its second quadrant,
z comes completely around and r e t u r n s to 1 . Now--here is the
trick--as w goes into i t s t h i r d quadrant, z will pass over to
its o t h e r sheet and go through two quadrants of the u n i t c i r c l e on
that sheet. emem ember t h e sheets cross each o t h e r a l o n g the
.
p o s i t i v e real axis. ) When w f i n i s h e s its c i r c u i t , so does z
k t , by introducing t h i s way of looking at the matter z has
gone through no p o i n t twice, except that it ends at 1 where it
s t a r t s . This statement must be interpreted w i t h care. There
would appear to be a duplicatton s i n c e o u r "gluing" seems to
i d e n t i f y the t w o p o i n t s 1 of t h e two sheets. Let us imagine
that z = l1 (in the f i r s t sheet) f o r w = 1 , z = l2 (in t h e
second sheet) for w = -1 , and z = 11 when w = 1 again.
We need a l l the points of each sheet--we cannot a f f o r d t o throw
any away by allowing some points to be in both sheets. W e look
on these p o i n t s a s distinct although I t is hard t o make a con-
vincing drawing; the pieces are connected cross-wise but we
think of them as n o t touching anywhere o t h e r t h a n 0 .
Hence, corresponding to each point on the u n i t "circle " i n
the Riemann Surface, there i s one and o n l y one p o i n t i n t h e
w-plane. Here is our f u n c t i o n ! Its domain is the two-sheeted
Rlemann Surface, I t s range is the w-plane. This function is
denoted by w = zI / z
The same theme, with variations, r u n s through the discussions
of I,* = z for o t h e r natural number v a l u e s of n .
Thus f o r n = 3 , the function z = w3 opens each of the
II
fans "

in t h e sense the images in the z = w3


plane of the p o i n t s in each of them
fill out the z-plane.

T h u s t h e z-plane is covered three times by t h e images of


points in t h e w-plane. In t h i s case we replace the z-plane w i t h
a three-sheeted Riemann Surface shown Ln Figure 12-7e. As before,
we then o b t a i n a function w = z whose domain i s t h i s surface
and whose range is the w-plane.
The idea is analagous for a general n . The function
w = z 'in has an n-sheeted Riemann Surface f o r its domain; its
range, as before, is the w-plane.
Chapter 13
SEQUENCES AND SGRIES

-
13-1. Introduction.
It is a comon experience to be c o n f r o n t e d w i t h a s e t of
numbers arranged In some order. The order and arrangement may be
given us, or we may have t o discover a law f o r 1% from some data.
For example, t h e millunan comes e v e r y o t h e r day. He came on
July 17; w i l l he come on August lP? We m i g h t consider that we are
given t h e s e t of dates
17, 19, 21, .. .
arranged from left to right in the order of increasing time. We
wish t o know how to c o n t i n u e the s e t . In t h i s simple case the
scheme is t r i v i a l ; we have

and the ansirer to t h e o r i g i n a l q u e s t l o n is yes. Any s u c h ordered


arrangement of a s e t o r numbers is c a l l e d a sequence.

Definition 13-la: A finite sequence of n terms is a


function a whose domain Is the set of numbers (1, 2, n) ..., .
The range is then the s e t [ a ( l ) , a(2), ...,
a ( n ) j , usually
written {al, a*, ".,
a,] .
The elements of the range are called
t h e t e r n s of the sequence.
An infinite sequence is a f u n c t i o n a v~hosedomain is t h e
set 2 ..
3, , n , . ..
] of a l l positive integers. The range of a
.
is then the s e t { a ( l ) , a ( 2 ) , a ( 3 ) , , , , a(n), ...
] , usually
written {al, a g , a3, ,.., an, . . . I . The element an o f the
range is called the nth term o f the sequence.
The terms of a f i n i t e or infinike sequence may be a r b i t r a r y
o b j e c t s of any kind, but in t h i s course they will be real or
complex numbers.
Example 13-la :

(a) 1, 2, 3, * * . , 17
(b) 17, -23, 15, 5280
(c) 17, 12, 7 , 2, -3, -8
(dl 3 , 6, 9, 12, . * *

(e) 2 3
T, T , T , ...
?r
(f) sin r , s i n T , s i n 3x , ..., sin;, -tr ... ,

The first three sequences are finite; the last t h r e e are


i n f i n i t e . In a l l but (b) a d e f i n i t e law governing t h e formation
of successive terms is e a s i l y discernible.
Suppose now t h a t in t h e sequences above we r e p l a c e t h e commas
between successive terms w i t h plus signs, The resulting expressions
are c a l l e d series. he noun "series"is both s i n g u l a r and )

E f i n l t i o n 13-lb: L e t (al, a2' ...,


an) be a g l v e n
f l n l t e sequence of r e a l or complex numbers; then t h e indicated sum

is c a l l e d a finite s e r i e s . The numbers al, ap, ...,


an are
called t h e terms of t h e series.
Let [al, a2, .. .? an, ..
. ) be a given i n f i n i t e sequence of
real o r complex numbers. men the indfcated sum
a1 +- a2 + ... + a, + ...
1s called an lnf i n i t e serles. The number an is called the nth
term of the i n f i n i t e series.
Accordfng to t h e definition, the expression
1+4+7+10+13+16
I s an example of a series. It is a finite series having s i x terms.
Note t h a t the operation of a d d i t i o n suggested by the plus signs is
-n o t actually involved in the definition. Of course, we s h a l l
e v e n t u a l l y want t o perform the addition in order to find t h e sum -
-of-a series, but it is wrong to confuse a series i:ith i t s sum. -
Example 13-lb:

This is an infinite series. Note t h e plus sign


1 9
before t h e dots. The loth term of t h e series Is - = 10- ,
109
11
The student is srarned against referring t o t h e last term" of an
Infinite s e r i e s ; there is none!

-
Example 1 3 - l c ; Find the llth term o f the i n f i n i t e s e r i e s
1 2 + 3 . 4 + 5 . G + 7 * 8 + . , . ,
where t h e d o t between the tigo integers of each term indicates
multiplication.
S o l u t i o n : The second f a c t o r of each term is e v i d e n t l y twice
the number of t h e term. Thus,
t h e llth tern i s 21 + 22 = 462 .
It is frequently dealrable to use l e t t e r s for the terms of a
sequence or a series, and often a subscript i s a t t a c h e d t o
i n d i c a t e the number of t h e term c o u n t i n g from the beginning, or
from some f l x e d p o i n t . Thus, the most general i n f i n i t e sequence
r a y b e w r i t t e n i n t h e form
13-la al, a2, a 3 , ... Y

and t h e most general infinite series as


The use of d o t s may turn o u t to be ambiguous, because the
composition of the sequence ( o r serxes), if complicated, may not
be evident from a few initial terms. To avoid t h i s difffculty,
mathematicians frequently write j u s t t h e general term. T h i s is
the kth term, starting f r o m any fixed point. Of course, 5
letter may be used instead of k ; the l e t t e r used l a called t h e
dummy v a r i a b l e . Thus i n place of Sequence (13-la), we use s e t
notation and write

This symbol means that If we replace k in t u r n by I, 2, 3, .. .


we w i l l have the Sequence (13-la). The upper and lower symbols
appearing outside t h e braces are called upper and lower indexes.
If a sequence is f l n i t e , say w i t h n terms, then the upper i n d e x
I s n r a t h e r t h a n w , and t h e last term is an .
In the case of
an i n f i n i t e sequence such as (13-la), there is -
no last term such as
am because w is not a number. We use m a s the upper index
in t h i s case simply to indicate that the sequence is infinite.
A similar shorthand n o t a t i o n I s used to represent a series.
Since a series is an indicated sum we use what is called
"summation n o t a t i o n " and represent Series ( 1 3 - l b ) by the symbol

The symbol Z is the Greek letter "sigma" which corresponds in


English to the f i r s t letter of the word l'surnft. The indexes mean
the same t h l n g here t h a t they do in the sequence notation. Thus
If a series is finite, say w i t h n terms, we write Instead
Example 1 3 - l d : Write out the f i n l t e s e r i e s

Solution: To obtain the terms of t h e series we have only to


s u b s t i t u t e the sequence of v a l u e s 5, 6 , 7 , 8 for k in the
general term 8 . We g e t

Example 13-le: The following symbol is merely a n o t h e r


n o t a t i o n f o r the infinite series of Example 13-lc:

You have only to write o u t t h e first f o u r terms to assure


y o u r s e l f of t h i s . Try it.
We conclude t h i s sectlon by defining t h e sum of a finite
series,

Definition 1 3 - 1 c : The sum S


, of a finite series is the
swn obtained by adding all of its terms.
n in the symbol Sn f o r t h e sum of a f i n i t e
The subscript
series indicates that n terms are added. By d e f i n i t i o n ,
S = al + a 2 f
n . + an ..
The symbol.
is a l s o used to denote t h e sum of an infinite series. It s h o u l d be
emphasized that each of t h e symbols

has two meanings as f o l l o w s : each symbol d e n o t e s not only t h e


f i n i t e series b u t also its sum. It will always be clear from the
context which meaning is intended.
It should be observed t h a t D e f i n i t i o n 13-lc does n o t define
t h e sum of an infinite series. Ne must postpone t h e statement of
such a definition until the concept of the limit of a sequence
has been introduced f n Section 13-4.

Example 13-lf: What is the sum of the f o l l o w i n g series if


n is odd? IZ n is even?

Solution:
We obtain t h e first term o f t h i s s e r i e s by
substituting k=l f o r the dummy v a r i a b l e k i n the general term
1 . We o b t a i n the second term by substituting k=2, e t c .
Thus t h e series is

If n is odd, t h e sum is -1 ; if n I s even the sum is 0 .


Exercises 13-1
3. Complete each of the following sequences through 7 terms:
( a ) -1, -4, -7, -10, . ..
( b ) 3/4, 6/7, 9/10, 12/13, ...
( 4 &', 2, 2 8 , 4, . * -
( d ) 2 x 5, 4 x 10, 8 x 20, 16 x 40, .. .
( e ) a, 3fi,5 ~ 7fi, , ... .
In t h e p r e v i o u s problem find t h e
kth term of each sequence.
Repeat P r o b l e m 1 using the abbreviated notation for a
sequence. Thus Part ( a ) Is

Complete each of the following series through 7 terms:


(a) 7 - 2 + 7 -
2 + ...
(b) 7 + 0 -7 + O + ...
( c ) a + 2a + 3a + 4a + ...
(d) l c 2 + 3 - 4 + ...
Find t h e sum of the f i r s t 7 terms of each series in
Problem 4 .
Write each of the following series using the Z n o t a t i o n :
( a ) -1 +1+3+ 5+ ... + 17
(b) 2 - 4 + 8 - 16 + . + . - 256
(c) l m 3 - ! - 2 .4 + 3 * 5+ *..
( d ) (1 + i) + 1 + (1 -
I) + (1 21) + - .,. .
Write a l l terms f o r each of the f o l l o w i n g :
8. Shwx by w r i t i n g out a l l of the terms that the following
symbols a l l represent the same series:

9. Write the nth term of the series

10. Admitting %hat the sequence

has its f i r s t two terms odd numbers, i t s next two even, e t c . ,


f i n d t h e general term of the series

11. Show t h a t

[ a e c . 13-11
12, Is it t r u e t h a t
n i n \ I n i

-
13-2. Arithmetic Sequences and Series.
Certain sequences and series are of such frequent occurrence
that they have been g i v e n special names.

D e f i n i t i o n 13-2a: An arithmetic sequence is a sequence in


which the d i f f e r e n c e obtained by subtracting any term from i t s
successor is always t h e same. This d i f f e r e n c e is called the
common d i f f e r e n c e of the arithmetic sequence and is designated by
the l e t t e r d .
i
An arithmetic sequence Is also called an arithmetic pro~ression
and we say that the terms of the sequence are "in arithmetic
progression." The common d i f f e r e n c e o f a n a r i t h m e t i c sequence is
o b t a i n e d by subtracting any tern from Its successor. Thus if al,
a*, a 3 # ". a k,l, ak, , ..
are the terms of an arithmetic
sequence, then
a2-al=a3-a2= ...
=ak-akml= ... = d .

Following are examples of arithmetic sequences:


( a ) 1, 2, 3, . . .,
17; d = 1 .
( b ) 17, 12, 7 , 2, -3, -8; d = -5 .
(c) 3, 6, 9, 12, ...; d = 3 .
The sequence 1, -1, 1, -1, ...
is not arithmetic because
the d i f f e r e n c e s between successive terms are alternately -2#2;
t h i s sequence has no common d i f f e r e n c e .

From D e f i n i t i o n 13-2a it f o l l o w s that if al is the f i r s t


term of an arithmetic sequence, then

The last line provides us w i t h an easy formula for f f n d i n g


the nth term of an arithmef ic sequence whenever the f i r s t term
and t h e common difference are known. We i l l u s t r a t e an a p p l i c a t i o n
of Formula 13-2a by means of an example,

Example 13-2a: If the 2nd term of an arithmetic


sequence is 0 and t h e gth term is 14 , what are t h e lSt and

Solution: Using Formula 13-2a wlth a2 = 0 and a9 = 14 ,


we see t h a t
and

IJhenc e d = 2 , al = - 2 , and ale, = 196 .


From t h e d e f i n i t i o n o f a series given in Section 13-1
- (Definition 1 3 - l b ) it follows that an a r i t h m e t i c series i s the
; indicated sum of t h e terms of an arithmetic sequence. The most
: important arithmetic series is the one whose terms are the
'I p o s i t i v e Integers. The common d i f f e r e n c e of t h i s seriea is 1 ,
I and t h e sum Sn of the f i r s t n terms is g i v e n by the e q u a t i o n
B

I Slnce the sum Sn is not a f f e c t e d by the order of addLtion, we


c a n reverse t h e o r d e r of the terms on t h e r i g h t s i d e and also
wrlte

Adding Equations 13-2b and 13-2c , we o b t a i n


2Sn = ( n + 1) -i- (n + 1) + ... + (n + 1) + (n + 1) ,
and d l v i d i n g both members by 2 , we have

Making u s e of the 2 n o t a t i o n introduced in S e c t i o n 13-1, t h e


l a s t equation can a l s o be wrltten as

The r e s u l t we have obtained can be stated a s a theorem.


Theorem 13-2a:

where n is a p o s i t i v e integer.
One could almost have guessed t h i e result. Slnce the differ-
ence between successive temna is the same, I t l a reasonable to
suppose t h a t the average term of t h l s series is half the sum of
the first and last terms, or half the sum of t h e second and next-

value . , is n2 + .
t o - l a s t , etc. The sum of n terns, each of which has the average
as s t a t e d i n Equation
s h a l l soan see t h a t t h i s is a general rule f o r a l l f i n l t e a r i t h -
13-2d . We

metic series; t h a t is:


Sum = (number of terms) x (average of first and last terms).

Theorem 13-2a may be used to find the sum of any arithmetic


series. The Following examples are I l l u s t r a t i v e .

Example 13-2a. Find t h e sum of the series


3+7+11+15+19+23.
Solution: Subtracting t h e first t e ~ mfrom each of the six
terms of the series and compensating by adding an equal quantity,
the sum is equal to
6(3) + [ ( 3 - 3) + (7 - 3) + (11 - 3) + (15 - 3)

+ (19 - 3) + (23 - 3) ]

Applying Theorem 13-2a to the expression w i t h i n parenthesis, we


f i n d t h a t t h e sum is
-
&ample 13-2b: Find the sum of 500 terms of the a r i t h -
metic series
( - f) + (-1;) + (-1;) + ... .
S o l u t i o n : To f4nd the sum of this series we need t o know
the last term. Using Formula 13-2a with al = - Ti 3 and
d = - 1 , we see that

a, = (-
3
+ '199(-
1
-
Thus the series can be w r i t t e n in the form

Whence we see t h a t the sum is

-
Example 13-2c: If t h e sum of the first n positive
integers is 190 what is n ?
5
n + n 190
Solution: 7 u

(n - lg)(n + 20) = 0

The s o l u t i o n n = -20 has no meaning In the present context


because n I s a positive integer.

Although It is p o s s i b l e to find the sum of any arithmetic


series by employing the acheme used in the preceding examplea, it
w i l l be h e l p f u l to have formulas t h a t can be a p p l i e d d i r e c t l y .
We can obtain two useful formulas by applying the method of
Examples 13-2a and 13-2h in the general case. Thus, consider
the arithmetic series of n terms, having first term al ,
common d i f f e r e n c e d , and nth term [al + (n l)d] : -

If we let Sn represent the sum, then

Applying Theorem 13-2a t o t h e q u a n t i t y w i t h i n brackets, we have

n
= rial + [(n - 1 1 2 + (n - 1)
l d , o r

Combining terms in the r i g h t member of 13-2c we g e t


%al + n(n
Sn = 2
l)d -

But by F o m u l a
n
13-28 , an = al + (n - 1)d; so
13-2f Sn = V(al + an ) .
E q u a t i o n s 13-2c and 13-2d give us u s e f u l formulas f o r f i n d i n g
t h e sum of any a r i t h m e t i c series. Which one we use in a given
case depends on what facts we are g i v e n . Using t h e s e formulas
w i l l greatly simplify the work in Examples 13-2a and 13-2b .
Example 13-2d: Find the sum of the series
Solution: Noting t h a t the lower and upper indexes a r e
respectively 2 and 15 , we see t h a t the series has 15 terms.
Substituting k = 2 and k = 16, we f i n d that al = -13 ,
aI5 = 1 .U s i n g Formula 13-2d we g e t

Note. Occasionally the last l i n e is written a s

When mathematicians u s e t h e X n o t a t i o n in t h i s way, they h a v e


-
in mind the sum of the series rather than t h e series itself.
Although we introduced the Z n o t a t i o n as a symbol f o r a series,
t h e dual usage of t h i s n o t a t i o n should cause no difficulty,
because it will usually be c l e a r from c o n t e x t which usage is
intended.

Example 13-2e: A body f a l l i n g from rest i n a vacuum falls


approXlmately 16 ft. t h e first second and 32 ft. f a r t h e r in
each succeeding second. How far w i l l It fall in 11 seconds?
In t seconds?
Solution: The series is
16 + 48 I- 80 + ... .
Using Fcrmula 13-2e with al = 16 ,d = 32 , and n = 11 (or t),
we see t h a t for If terms

and for t terns


Exercises 13-2
Determine which of t h e following series are arithmetic. Find
d and the next three terms f o r t h o s e that are.
( a ) 4 + l o + 1 6 + 22+ ...
(b) 5 + 9 + 12 + 18 + ...
( c ) 2.0 + 2 . 5 +.3.0 + 3.5 + ...
(d) 2 + 3 . 2 + 5 ' 2 + 7 . 2 + . . .
( e ) -10 - 6 - 2 + 2 + ...
F i n d the sum of t h e series

Find the aum of t h e series

Write t h e f i r s t 5 terms of an arithmetic series in which


the second term is m and the t h i r d term is p .
W r i t e the f i r s t five terms of an arithmetic s e r i e s in which
the second term is m and the fourth term is p ,
If the t h i r d term of an arithmetic s e q u e n c e is -1 and the
16th is 11 , what is t h e f i r s t term?

If 2 - n is the nth term of an a r i t h m e t i c series, write


the f i r s t term.
F i n d t h e i n d i c a t e d tern in each of the f o l l o w i n g series:
(a) 15th term in + 5+ 3 ...
(b) 11th term in -2 + 1 + ...
(c) 9th term in T5 + 13
F + ...
How many i n t e g e r s are there between 35 and 350 which are
divisible by 23 ?
10. The arithmetic mean between a and b is a + b . Find
its value if
(a) a = 5 , b = 6 5 .
(b] a = - 6 , b = 2 .
( c ) a = 3 - f l ,b = 7 + 5 f l .
(d) a = ( c + d )
2
,b=c2-d2 .
11. Take every 5th term from an arithmetic sequence and form a
new sequence. Is the new sequence a r i t h m e t i c ?
12. If 3 1 and 8 $ are the f i r e t a n d eighth terms of an
arithmetic sequence, f i n d the six terms t h a t s h o u l d appear
between these t w o so t h a t a l l e i g h t terms will be in arlth-
metic progression. (The six terms you are asked to find a r e
called the - -
six a r i t h m e t i c means between 3 1 and 8 T, 1 1
Find t h e sum of the following series by using Theorem 13-2a.
(a) 1 + 2 + 3 + ...+ 10
(b) 1 + 2 + 3 4- . . * + 999
( c ) -3 -6 - 9 - 12 - 15
14. On a ship, time is marked by striking one b e l l a t 12:30, two
bells a t I:oO, three b e l l s at 1:30, e t c . u p to a maximum of
8 bells. The sequence of bells then b e g i n s anew, and it is
repeated In each successive i n t e r v a l of four hours t h r o u g h o u t
the day. How many b e l l s are s t r u c k during a day (24 h o u r s ) ?
How many are s t r u c k at 10j30 p.m.?
15. Find t h e sum of t h e series
17. Find a and b if

18. Find the sum of the series

Show t h a t the sum is the number of terms m u l t i p l i e d by the


average of the first and last terms. ere k rmns through
all integers f r o m -n to m I n c l u s i v e . )
19. The digits of a positive integer having three digits are in
a r i t h m e t i c progression and t h e i r sum Is 21 . If the d i g l t s
a r e reversed, the new number is 396 more than the o r i g i n a l
number. Find the o r i g i n a l number.
20. Find formulae f o r al and S, when d , n and a, are
glven .
21. Find x if ( 3 -
x ) , -x ,4- are in arithmetic
progression.
22. The sum of t h r e e numbers which are In arithmetic progression
is - 3 and their product is 8 . Find the numbers.
2 3 . Plnd the sum of all p o s i t i v e integers less than 300 which
(a) are multiples of 7 ,
(b) end in 7 .

-
13-3. Geometric Sequences -
and S e r i e s .
Another very important s p e c i a l sequence is t h e geometric
sequence.

Definition 13-3a: A geometric sequence is a sequence in


which the r a t i o of any term to I t s predecessor is the same f o r
a l l terms.
Thus i f the f i r s t term of a geometric sequence is al and
t h e common ratio i s r , then

13-3a a
n = airn-1 , w h e ~ e n is a p o s i t i v e
integer.
The l a a t l i n e gives us a formula for t h e nth term. Geometric
sequences a r e a l s o referred to as geometric progressions, and the
terms of t h e sequence are said to be "in geometric progression ".
From the d e f i n i t i o n of a series given in S e c t i o n 13-1
( D e f i n i t i o n 1 3 - l b ) it follows t h a t a geometric series i s t h e
l n d l c a t e d sum of the terms of a geometric sequence. For the
sequence introduced above we have the geometrlc series
n- 1 n-1

which is f i n i t e and has n t e r m s , o r t h e i n f i n i t e series

As w i t h a l l i n f i n i t e s e r i e s , (13-3c) has no l a s t term.

Following are examples of geometric series:


(a) 1 + 2 + 4 + 8 + 1 . 6 + ...; r = 2 .
(b) i - l - i + l + i -... ; r = i .

(d)
10
+ 1 + .3 + .09 + .027 ; r = .3 .
(el rr- a2+n3 - a4 + s5 - ... ; r = - T .

(f) 3(10) + 3(10)' + 3(10)-' + 3(10)-' + 3(10)-~ ; r = ,.1


The l a s t series 1s f i n i t e so we know t h a t it has a sum. -
Interestingly enough i t s sum is 3 3 . 3 3 3 .
This i l l u s t r a t e s t h a t
any number a l l of whose digits a r e i d e n t i c a l is r e a l l y t h e sum of
a geometric series w i t h common ratio 1 10 .
The series

is n o t geometric because it has no common ratio. The ratios are


a l t e r n a t e l y 1 and -1 .
To obtain a formula f o r the sum of t h e finite geometric
S e r i e s 11-3b recall the following formulas from Chapter 1 for
f a c t o r i n g polynomlals;
2
l - r = (1 + T}{I - r ) ,
1 - r3 = (1 + r + r2}(l - r) .
A n e x t e n s i o n of these formulas to t h e form 1 - rn suggests t h a t

The e q u a l i t y in the last line can be checked by m u l t i p l i c a t i o n .

If r f 1 we may divide both sides of Equation 13-3d by (1 - r)


to g e t
1 + r + r + ... + r
2 n-1
=m
1 rn -
Multiplying b o t h members of the last equation by al we o b t a i n
the sum of S e r i e s 13-31 .
We have proved the following theorem:
-
Theorem 13-3a:

where n = 1, 2, ...
Note t h a t if r = 1 , the series has n terms all equal to
a
1 ' s o t h a t t h e sum is rial .
Equation 13-3e can be used as a
formula f o r application problems in which r 4 1 .
In t h i s
connection, however, we usually represent the left member of
Equation 13-3e by s, and employ the shorter form

Another u s e f u l f o r m u l a for t h e sum of a finite geometrfc


s e r i e s is
ran - al
-
n' - r - 1

This formula can be easily obtained by making use of Formula 13-3a.


Since

Re'driting 13-3f in the form

and substltutlng ran = alrn we have 13-3g .


Example 13-3a: If t h e bth term o f a geometric series is
6 and the 9 7 e r m is 1458 , find the lSt term, the 10th
term, and the sum of the first t e n terms.
S o l u t i o n : Use Formula 13-3a twice: first w i t h n = 4 ,
a,+ = 6 ; then with n = g , = 14%
a9
.
We get two equations:

Solving, we obtain r = 3 , al = 32 ; from which a10 = 4374. Using


Formula 13-3f with n = 10, we g e t

Example 13-3b: If a f i n i t e geometric aeries has the last


t e n 1296 , r a t i o 6 , and a sum of 1555 , find the lSttern.
Solution: Using Formula 13-3g with an = 1296 ,r = 6 ,
and sn = 1555 , we have

Hence

Exercises 13- 3 -
1. Write the n e x t t h r e e terms in each of the following geometric
sequences:
(a) - 2 , -10, .. .
,
3
(b) -{,
q , rnrn.3
(c) 7 , 1, ... .
2. If a + b + c is a geometric series, express b in terms
of a and c .
Find the sum o f t h e following series:
2
(a) l + 2 + 2 + . . . + z 9 ,

(b) 1 - 32 - 33 + 34 ,
3 +
(c) 1 - 1 1 +
1 + -
+ I .,. - l ( 1 0 0terms) .
Find the sum of the series

Find n if zk = 63 .
Find n if 3 + 3 2 + 33 + ...
+ 3" = 120 .
Can t w o d i f f e r e n t geometric series have t h e same sum, the
same first term, and t h e same number of terms? ( T r y
2
l + r + r = 7 . )
Find the sum of the series

rk , both when r = 1 and when r # 1 .


Find the sum of t h e series

Find t h e numbers x to make the f o l l o w i n g s e r i e s geometric:


3 8
( a ) - T + x - - z7 f a - r n ,
(b) 42-x+Jn+
J = +... .
How many terms a r e there In the geometric series
12. Find, if possible, the lSt and Pd terms of a geometrlc
series w ~ t h 3rd term = -4, 5th term = -1 , 8th term = 8 - 1
'
13. Find a l l sets of 3 integers in geometric progression whose
p r o d u c t is -216 and the sum of whose squares is 189 .
14. If M is the foot of a perpendicular drawn from a point P
of a semicircle t o t h e diameter AB , show t h a t lengths AM ,
MP , are in geometric progression,
1 . The terms of a finite geometric series between the ffrst and
the last a r e called ~ e o r n e t r l cmeans between the f i r s t and
last. If the series has only three terms, t h e m i d d l e term
is called - the ~eometricmean between the other two. Insert
( a ) 3 geometric means between 1 and 256 ,
( b ) 2 geometric means between f i and 5 ,
( c ) the geometric mean between a8 and 16b4 ,
(d) the geometric mean between a and b .

13-4. ---
- Limit of a Sequence.
Recall again t h e d e f i n i t i o n of a sequence of numbers stated
in S e c t i o n 13-1 (Definition 1 3 - l a ) . we wfll find it convenient
t o plot the numbers al , a2 , a3 , ... a , .. . on a number l i n e .
To avoid confusion we w i l l label the points associated w i t h t h e
numbers of the sequence by the symbols which represent them in t h e
sequence.
a3

In t h i s way we can establish a correspondence between the terms o f


a sequence of numbers and a set of points on the number line.
To s t u d y the behavior of a sequence of numbers and the points
corresponding to them l e t us look b r i e f l y at several examples.
Example 13-4a:

I . ,
a3 =* I

ohf -
I
4 -P
1
I
Points corresponding to successlve terms of the sequence get
closer and c l o s e r to t h e point 0 as n becomes l a r g e ; t h a t is,
an approaches zero as n becomes large.

If n Is odd, the points corresponding to an g e t c l o s e r and


c l o s e r to 1 as n becomes large; if n is even, the points
corresponding to an approach -1 . Hence an a l t e r n a t e l y
approaches 1 and -1 .

P o i n t s corresponding to an are alternately to the r i g h t and left


of 0 ; however, as n becomes large successive p o i n t s get c l o s e r
and closer to 0 .
Hence,
an approaches zero as n becomes l a r g e .
(d) 1 , -1 , 1 ,
-1 , 1 , -1 9 .,.
In t h i s case a, is alternately equal to 1 and -1 .

In t h i a case I t is easy to see that as n becomes large so does


an .
The foregoing examples show clearly that there are two k l n d s
of sequences which d i f f e r according to the way in which an
behaves as n becomes large. In both ( a ) and ( c ) bre see that as
n becomes large a, approaches some fixed number A , and we
-
say t h a t an approaches a limit A as n becomes l a r g e . Such
sequences a r e said to be convergent. On the other hand, in { b ) ,
( d ) , and ( e ) , there is no fixed number that an approaches.
Sequences of this kind are said to be d i v e r ~ e n t .
The n o t i o n of limit may be familiar t o you. In geometry, f o r
example, you learned t h a t the area of a regular polygon inscribed
in a c i r c l e approaches t h e area of the c i r c l e as the number of
sides increases. In Section 6-6 of t h i s t e x t it was shown that
the distance between a p o i n t on a branch of a hyperbola and an
asymptote approaches zero a s the p o i n t moves o u t i n d e f i n i t e l y f a r
on the curve.
To make t h e n o t i o n of the limit of a sequence p r e c i s e , we
s t a t e t h e following d e f i n i t i o n .

D e f i n i t i o n 13-4a: The sequence al , ap , ag , has ...


a l i m i t A ff an becomes and remains arbitrarily c l o s e to A
a6 n gets larger and larger. A sequence that has a limit is
said to be convergent.
Under the c o n d i t i o n s of the definition we also say that "the
llmlt of a, a s n becomes infinite is A ," and we wrlte the
statement which appears in q u o t a t i o n marks w i t h the symbol

The f a l l o w i n g examples i l l u s t r a t e the d e f i n i t i o n . The limlt


in each case Is given. Write enough additional terms in each
example to s a t i s f y y o u r s e l f that the given sequence has the
indicated l i m i t .

[ s e c . 13-41
Although it may seem obvious t h a t the limit of t h e given sequence
should be zero, this example is not t r i v i a l and will be u s e f u l
l a t e r . Symbolically we o r d i n a r i l y write

-
Example 13-4c:

Here an = 1 when n is odd, and an = 1 - FT when n is even.

It is n o t r u l e d out by the definition t h a t a


,
may be equal to its
limit f o r some v a l u e s of n , or even f o r infinitely many values
of n .

(2) 1 , ..; A = 2 .
Note that an is alternately larger and smaller t h a n A .
Sequence ( c ) In Example 13-4a behaves similarly.

Note: A fact which deserves mention at t h i s p o i n t is that a


sequence cannot have two different limits, because it I s n o t
p o s s i b l e f o r a, to be a r b i t r a r i l y close to each of two different
-
numbera f o r a l l n s u f f i c i e n t l y large. What is meant here is
i l l u s t r a t e d by sequence ( b ) in Example 13-4a .
This sequence, a s
-
already stated, is not convergent; it is d i v e r g e n t .
In t h e preceding examples the limit f o r each sequence was
given and it was relatively easy to see that t h e indicated limit
was indeed the limit of the given sequence. On t h e o t h e r h a n d ,
determining whether a gfven sequence has a limit and f i n d l n g I t s
value when there is one c a l l s f o r some specialized knowledge of
the properties of limits, I n advanced c o u r s e s in mathematics
these properties are u s u a l l y stated as theorems. Before stating
such theorems we f i r s t observe t h a t the sequences

can be used to form many new sequences; f o r example,

a2 a3 "n
" ; ' 6 ; ' "''y
. . . (if each bn 4 0) ,

etc.

W e conclude t h i s section by stating w i t h o u t proof t h e


following theorems i n v o l v i n g limits. The student w l l l find these
u s e f u l in finding the l i m i t of a sequence,

Theorem 13-4a: The constant sequence c , c , c , .. ,


has c as I t s llmit; t h a t is
Theorem -13-4b: If lim an = A , and lim b
, = B , then
n+m n+m

(1) l i m (ca,) = c lim an = cA;


n+m n-00

(3) l i m (a,b,) = (llm a,) ( l i m b,) = AB ;


n+rs n+w n+m

Example 13-4e : F i n d t h e limit of t h e sequence f o r which

Solution: Since
1 1 1
7 = (fi) 9
n
th
t h e g i v e n sequence Is the p r o d u c t of two sequences having n
1
t e r n s - and T;.
n . Thus, by Theorem 13-bb(3) and Example 13-'lb,

Example 13-4f: Find the limlt of t h e sequence f o r which

n
-
Solution: Dividing the numerator and denominator by n we
see that
n 1

Thus, the g i v e n sequence I s the q u o t i e n t of two sequences iihose


1
n th terms are 1 and 1 c F; respectively.

By Theorem 13-4b(d) we see t h a t


lirn

But lirn (1 1
+ --) = lim 1 + lirn -1n = 1 + 0 = 1

by Theorem 13-4b(b), 13-4a, and Example 13-4b.


Theref ore, lim n= 11
1 .
n+w

Example 13- 4g: Find Urn .


n+m 7n
3
Solution: D i v i d i n g t h e n u m e r a t o r and denominator by n ,
we note that

Hence, lim w = 0 + 1 : + 5 - - 27 ,
n-m 7n

Example 13-4h: Find the limit of t h e sequence


Solution: ~f n is an odd integer, say 2k + 1 then

If n is a n even Integer, say 2k , then


a, = a2k -
-
2k + 2
. Again w as n-rm.

The n o v e l f e a t u r e of t h i s example is t h a t as n increases a,


a l t e r n a t e l y gets c l o s e r to and f a r t h e r from i t s limit. B u t the
a, does, none the l e s s , "become and remain a r b l t r a r l l y close1' to
2 . in a p p r e c i a t i o n of what happens when n becomes infinite
may be v i s u a l i z e d by p l o t t i n g successive v a l u e s of a, on a
number line.

-
Exercises 1 3 - 4
1. E v a l u a t e t h e limit for each sequence that is convergent.
2. Make use of D e f i n i t i o n 13-4a to decide which of the
f o l l o w i n g sequences converge. Make a guess a s t o the limit
f o r those that converge.
(a) o , l , 0 , 2 , 0 , 3 , 0 , 4 ,...

(d) -1, 2 , - 3 , 4 , -5, * * -

(f) (1 +;I , 1 1
(1 + g + $ . 1 1 1
(1 + g + $ + ~3 ) *..
1
(PI 1 , , -1 , ,1 1 , 1 , -1 , ,1 ...
(h) 0.6 , 0.66 , 0.666 , 0.6666 , ...
8 11
(11 3 , 7 , *..
3

(J) 1 ? f l ?8 * f l ?- * *
(k) 1 2 , 2 2 , 3 2 # 4 2 , .*.

3. Find the following limits:

n2- & + I
(b) lim
n+m n2+ n 1 -
(c) lim 3n3 -n 9

n+m 5n3 c 17
4. Show t h a t
{a) lim 1 = 0
7 ,
n+m n

(b) lim ;;( 1 2 = 0 .


- =)
n+m n
5. Prove that

does n o t exist.
6. Prove t h a t
Em3
Ilrn
n--*m
does n o t exist.
7. Prove that
lim a n 2 + b n + c = a -d ' if d f O .
n- dnd + e n + f
Compare w i t h results in 3{a) 3 ( b ) and 3 ( c ) .
8. ~f d = 0 In Exercise 6 , can the limit s t i l l exist for
c e r t a i n values of the constants? Compare w i t h results in
Exercises 5 and 6 .
* 9 . Prove t h a t f o r any p o s i t i v e integer r

10. Admitting t h a t lim rn = 0 (lrl < 1) , find


n--*=
764

11. Find

Do n o t expana the products.


12. Find the f o l l o w i n g limits if convergent.
(a) lirn 1
n+ar

(b) lim 0
n4oo
( c ) lim 7
n

13-5. Sum of an Infinite Series.


In t h i s section we w i l l make use of the concept of the limit
of a sequence developed In t h e last section to formulate a
definitfon for the sum o f an i n f i n i t e series.
Recall that the d e f i n i t i o n given in Section 13-1 f o r the aum
of a series applies only to finite s e ~ i e s . Even so you may have
an intuitive idea of what is wanted in the i n f i n i t e case. For
example, if you meet the number .5 , you may feel quite sure
t h a t t h e number is intended. The i n f i n i t e decimal I s , af
course, equivalent to the series.

and presumably the sum of an infinite series should be defined in


such a way t h a t t h i s series w i l l have the sum 7 . w e emphasize
that we have the right to make d e f i n i t i o n s as we l i k e , if only we
agree to stick to t h e t e d n o l o g y we adopt. However, we a l a o want
to keep t h i n g s reasonable and consistent. For example, if we have
a f i n i t e series
t h e n the sum ha8 already been d e f i n e d . But sn in 13-5b can
a l s o be regarded as an i n f i n i t e series a11 of whose terms are
zero a f t e r a c e r t a i n point. A d e f i n i t i o n of t h e sum of an
i n f i n i t e series must c e r t a i n l y n o t c o n f l i c t w i t h o u r p r e v i o u s
d e f i n i t i o n in t h i s s p e c i a l c a s e .
Suppose t h a t we are confronted w i t h a s p e c i a l Infinite series,
say

and are asked to guess what i t s sum is. We m i g h t proceed a s


follows. Denoting the sum of the first n terms by sn , we
observe t h a t

The numbera sn l i s t e d above are c a l l e d a-sums- they

are p a r t i a l sums of Series 13-5c .


If you look c a r e f u l l y at them
you may have the feeling that as n increases sn approaches
some number A which ought t o be called the sum of i n f i n i t e
Let us review the argument used in the introductory examples
which precede the definition.
(1) In each case we sought a number whlch we might c a l l the
sum of an I n f i n i t e series such a s

2 We used Definition 13-lc f o r t h e sum of a f i n i t e


serles to generate a sequence of partial sums o f t h e
given inflnite series. That is, we used
n

to obtain the sequence of partial sums


m

( 3 ) F i n a l l y , we examined the sequence of p a r t i a l sums f o r


convergence. By the d e f i n i t i o n which we have j u s t s t a t e d
(~efinition1 3 - 9 ) we know t h a t If t;he sequence of partlal sums
of a given i n f i n i t e series has a limit, t h l s l i m i t 1 s t h e sum of
t h e infinite series. If the sequence of p a r t i a l sums has no
lLmit then t h e series does not have a sum.
A t f i r s t thought one might conclude t h a t we a r e now equipped
with a general method f o r investigating any Infinite series and
obtaining its sum, if it has one. This Is true, but the method
outllned above is not g e n e r a l l y u s e f u l because o f technical
difficulties. Except for some easy s p e c i a l c a s e s t h e method Is
hard t o apply. The difficulty l i e s in determining an expression
f o r s, whose limit can be c a l c u l a t e d .
Thus, mathematicians r e l y on f a r more powerful techniques.
Unfortunately, t h e background on which these depend has not been
developed in C h i s t e x t so we shall not introduce them. Thus, in
t h i s text, t h e work of f i n d i n g sums of infinite series I s limited
to series t h a t can be handled with t h e methods presented,
Example 13-5a: Find the p a r t i a l sums of the series

Solution: Using Definition 13-lc we obtain the sequence

Examplel3-5b: F i n d t h e s u m ( i f t h e r e i s o n e l o f t h e series
L*l

Solution: sl = 1 , s2 =
1
, s3 =
1
3 , ... , Sn =
1
ji

By Example 13-4b the desired llmlt is 0 . Hence the


series converges and has the sum 0 .
~ x a m ~ 13-5c:
le Find the sum (if there is one) o f the series

S o l u t i o n : The general term of t h i s series is a u n i t more


than the general term of the previous series.
Hence sn = n + F;
1
.
~ u tlim (n + $)
n+w
does n o t exist; t h a t is, t h e series diverges and
has no sum.
am le 13-5d:
Ex- The harmonic series 1 + 71 + 7
1 + 1 + ...
can be proved to be d i v e r g e n t .
Solution: The p a r t i a l sums are
1
Sl = 1 = 2(-2) ,
1 1
s* = 1 + H = 3(g).
1 1 1 1
s, = (1 + ;T) + (3+ > 4(T) 9

+ 31 + -13+ 1 I 1
sg = (1 + (5 + + 'g) > 5($) *
s16 = (I 1 +
+7 ...

Starting at some p o i n t ,
1
, where n is some power of
(n = 2m) , examine the next block of 2m terms,
2 ,

+ 1 4- ... + 1
P+l P+2 P + P
1
Each of t h e s e is certainly greater than or equal to
Hence, t h i s block of terms 2 m + p .

Since we can f i n d a n i n f i n i t e number of such blocks, the sum has


no limit; t h a t is, the series is divergent.

Example 13-5e: On the o t h e r hand the alternating series


1 - 1 + 3
-2 1 - 1 + , .. is convergent, and comparison w i t h Problem 19
in the exercises w i l l indicate t h a t t h i s is the series f o r
l0ge2 .
We conclude t h i s section by considering a problem which
a r i s e s as an interesting s i d e l i g h t to the concepts that have been
developed. Suppose t h a t we a r e given any sequence of numbers. Can
we c o n s t r u c t a series such that t h e g i v e n sequence is a sequence
of p a r t i a l sums for the series? The answer is yes, and He shall
make t h e matter clear with an example.

Example 13-5f: C o n s t r u c t a series whose p a r t i a l sums


correspond to the sequence given in Example 13-4h:

Solution: Let sl = 5 , s2 4
= 3 , s3 = 7
4 , ... .
Since sl = al ,

... ,
s
n = al + a2+ ... + anml + a n = s ~ +-an~ '
we see that

Thus the r e q u i r e d series c a n be obtained from


s1 + (s2 - S1) + m . + (sn - Sn-l) + ... rn

Making the proper substitutions f o r sl , s2 , e t c ., we have the


series

The last line suggests how to continue t h e s e r i e s ,

[ s e c . 13-51
I n f i n i t e series a r e one of the most important tools of
advanced mathematics. For example, it can be shown t h a t

5
~ i n x =X x x- ~ ...
+ ~, x- in radians ,

c o s x = l - zX+ - Qx4
- ... , x in r a d i a n s .
h r t h e r m o r e , these series converge f o r every v a l u e of x , and the
sum of each series for any x is the v a l u e of the function on the
l e f t f o r that value of x . Infinite series a r e also important I n
the calculation of tables of logarithms and tables of trigonometric
functions.

Example 13-5g: Find the value of e c o r r e c t to f o u r


declmal places.
Solution: As indicated above,

The p a r t i a l sum 'sn is a good approximation to t h e sum of an


I n f i n i t e series if n is large. We r i n d

If S8 is rounded o f f to four decimal places, we obtain 2.7183 ,


which is t h e v a l u e of e correct to four decimal places.
Exercises 13-5
Find the partial sum f o r t h e f i r s t n terms of t h e s e r i e s
2 + 7 + 1 2 + 17 + ... .
Find t h e p a r t i a l sum f o r the f i r s t n terms of t h e series

Find the p a r t i a l sum for the f i r s t n terms of t h e series


15 + 1 2 + g + 6 + ... .
Find the p a r t i a l sum f o r the first n terms of the series

Find a series whose partial sums are


Pind a series whose partial sums are
Find a series whose partial sums are
Find a series whose partial sums are
Pind a series whose partial sums are
Pind the sum of t h e series

Pind a series whose partial sums Sk are given by t h e formula

Find the sum of the series

Show t h a t the series k2 k2 diverges.


4- -k+2
k=l
Determine whether t h e series 1 -2+3-4+5- 6 + ...
diverges or converges.
15. The area under y = x from x = 0
to x = 1 , approximated by n
rectangles is the sum
n-1
;(; +, + ... +
1 n-1
= %
n
k *
k=O
n-1
Find the lirn 7

n
16. The area under y from = x2
x = 0 to x = 1 , approxi-
mated by n rectangles is
the sum

n-l

" k=O

.-..7
n n- 1
lf kP = n(n+;)(&+l] , flnd the llm
k=O k=O

17. Find the l i m i t of the sum (if t h e r e is one) of

Hint : 2 1 1
& + 1 ) ( 2 k + 3 ) = 2 k - - '

18. With the series g i v e n for ex , find t h e approximation ,forf i .


19. An approximation f o r the n a t u r a l l o g a r i t h m i s given by t h e
following serles for -1 < x 1 < .
log, (1 + x) = x 1 2 1 3
-p + 3x - ...
Use t h i s series t o estimate loge 1 1 to 4-place decimals,
Compare t h i s r e s u l t w i t h the appropriate area under the c u r v e
1

y = - in Chapter 9.
I
X
13-6.
- -
The Infinite Geometric Series.
No infinite arithmetic series converges unless a l l o f i t s
terms a r e zero; h e n c e , the convergence of infinite arithmetic
series will not be c o n s i d e r e d further. On t h e other hand, we have
already seen that certain infinite geometric s e r i e s may converge.
-
For example, t h e infinite decimal . 3 mentioned at t h e beginning
o f Section 13-5 has t h e value , and this i s e q u i v a l e n t t o
saying that a c e r t a i n geometric series converges and has the sum
1 .
By Theorem 13-3a the nth p a r t i a l sum sn of the infinite
geometric series

Can t h i s p a r t i a l sum have a limit as n becomes i n f i n i t e ? It


depends on rn . If r = 2 , then r = 4 , r3 = 8 , e t c ., a n d
2
rn i n c r e a s e s rapidly as n i n c r e a s e s . No limit e x i s t s f o r
3
m ,
nor f o r sn . If r = - 2 , then r = 4 , r = -8 , e t c . , and
2
again no limit exists. On t h e o t h e r hand, if r = f- 1/2 , then
r 2 = 1/11 , r3 = !: 1/8 , etc., and rn approaches zero. That is,
a1 The result 1s e v i d e n t l y golng
=n will have the limit -
to depend on t h e a b s o l u t e value of r .
The above argument shows
that i f J r () 1 then Series 13-6a diverges: if Irl ( 1 then
S e r i e s 13-6a converges and has the sum al
.
F i n a l l y , if
r = f 1 t h e s e r i e s reduces t o

These c e r t a i n l y diverge unless al = 0 . We summarize o u r results


by s t a t i n g a theorem.
Theorem 13-6a: The i n f i n i t e geometric Series 13-6a
converges and has t h e sum - al -
if Iri ( 1 .
It diverges if
I rl > 1 (unless a, = 0 , hen it converges).

Example 13-6a: F i n d t h e sum of the series


w

Solution: al = 1 ,r = - 1
T , 1 < 1 . The series converges
and has t h e sum 1 4
7= 5
I'B

Example 13-6b: F i n d t h e value of t h e repeating decimal

Solution: This I s equivalent t o the geometric s e r i e s

The serles converges and has t h e sum

Example 13-6c: A t r a i n is approaching a p o i n t P 3 0 m i l e s


away, at 30 miles p e r hour. A fly with twice the speed lezves
P , t o u c h e s t h e t r a i n , r e t u r n s to P and repeats the p r o c e s s
until t h e train reaches P . How far does the f l y t r a v e l ? '
S o l u t i o n : To simplify matters let v represent the velocity
of the train, even though we know that v = 30 , and let 2v
represent the v e l o c i t y of the f l y , Suppose t h e f i r s t meeting is
tl h o u r s after the s t a r t . Thus a f t e r tl hours the fly will be
atl miles from P and the train will be 30-vtl miles f r o m P .
-2vt T V t 7
Hence 30-vtl = atl , vtl = 10 ; and s i n c e v = 30 , t 1 = '13 .
So t h e f l y has t r a v e l e d miles and the t r a i n 10 miles when
20
they meet f o r the f i r s t time. Therefore the f l y ' s --
f i r s t round
1 4v
t r i p is 2 ( & ) ( 3 ) = -j- miles. The t r a i n is now 10 miles from
P , and we r e p e a t the computation. L e t t2 be t h e time required
f o r the f l y t o go from P to t h e t r a i n the second time. Thus
1 0 - v t 2 = 2vt2 , vt2 = 7
lo ; and since v = 30 , t2 = -$ . The

1 4v
f l y 1s second round trip is 2(2v) (=) = 7 , etc. The answer, in
series form, I s 3 3

uhere al =
4v ,r =
1
-5 < 1 . By Theorem 13-6a the sum is

The f l y t r a v e l s 60 miles. T h i s result c a n be checked directly


w i t h o u t using series. W e have only to note that the t r a i n needs
one hour'to g e t to P , and if the f l y wastes no time It can do
60 miles in t h a t time. We have deliberately done t h i s example
the hard way t o i l l u s t r a t e Theorem 13-6a for a case in which we
know the answer in advance.
Exercises 13-6
1. Find t h e sum of the series
(a) 1 + g +
1 1
+ , ...

2. Find t h e sum of the series


(a) r+ r 2 + r3 + ... , (14 < 1) ,
(b) (1 - a ) + (1 - a ) + (1 -a P
) + 1 - a ) + ... .
For what v a l u e s of a is the series convergent?
3. Write each of the following repeating decimals as an equiva-
lent common f r a c t i o n .

(a> 0.5
( b ) O.O=

(c) 3.797
(d) 2.m
. What distance w i l l a golf ball t r a v e l if it is dropped from a
height of 72 i n c h e s , and if, after each f a l l , it rebounds
9/10 of t h e d i s t a n c e it fell.
5 S o l v e the following e q u a t i o n for x :

6. Solve the f o l l o w i n g e q u a t i o n f o r x :

7. Solve f o r al and r if

al + air + air 2 + . . . = 73 , and


8. An equilateral triangle h a s a perimeter of 12 i n c h e s . By
Jolnlng t h e midpoints of I t s sides with llne segments a
second triangle is formed. Suppose this operation is
continued f o r each new t r i a n g l e t h a t is formed. Find the
sum of the perimeters of a l l triangles including the
original one.
9 . A hare and a tortoise have a race, t h e t o r t o i s e having a
5000 yard handicap. The harefs speed is V = 1000 yards
per minute; t h e tortoise~sspeed is v = 1 yard per minute.
It is sometimes said t h a t the hare can never c a t c h t h e
t o r t o i s e because he m u s t f i r s t c o v e r half t h e distance
between them. Detect the fallacy.
10. A r e g u l a r hexagon has a perimeter of 36 inches. By
joining t h e consecutive midpoints of its sides w i t h l i n e
segments, a second hexagon is formed. Suppose this is
c o n t i n u e d f o r each new hexagon. Find t h e sum of the
perimeters of a11 hexagons i n c l u d i n g t h e original one.
11. A square has a perimeter of 12
inches. Along each side, a p o i n t
3
is located one-third the distance
to the r i g h t of each v e r t e x . By
j o i n i n g c o n s e c u t i v e points, a new
square is formed. Suppose t h i s
process is continued for each new
square.
( a ) Find the sum of t h e perimeters
o f all such sauares.
(b) Find the sum of the a r e a s of
a l l such squares.
13-7+ Miscellaneous Exercises.
1. Find t h e sum o f t h e s e r i e s

2. The f o l l o w i n g series a r e either arithmetic or geometric;


c o n t i n u e each through 6 terms:
(a) 8 + 4+ 2 + .., ,
(b) 3 +6+ 9 + ... ,

(d) -13 - 1 + 3 + ... I

4 6
(e) a
2
+ a + a + ... ,
(f) i - 1 i + - ... ,
3. Find the sum of the series

4. Use the i d e n t i t y
k = k + k 1) -( k - 1 ) t o give a new proof of
Theorem 13- a.
5. By use of t h e i d e n t i t y
+ 1 = (k + 1 - k2 , prove that
6. By use of the equation
1 1 1
E - k = m ' show t h a t

7. Shou t h a t the geometric mean between two positive i n t e g e r s


i s n o t g r e a t e r t h a n the arithmetic mean between them.
8. The harmonic mean between t w o numbers a and b is a
number h whose reciprbcal is t h e arithmetic mean between
t h e reciprocals of a and b :

Show that in t h e s e r i e s

each term a f t e r t h e first I s t h e harmonic mean between i t s


t w o neighbors.
9. Shox that t h e number of vertices o f a cube is t h e harmonic
mean between the number of i t s faces and the number of i t s
edges.
10. Show that geometric mean between two numbers is also t h e
geometric mean between t h e i r a r i t h m e t i c and their harmonlc
means. F i r s t t r y the result f o r 2 and 8 .
11. Find t h e sum o f the following series correct to 2-place
decimals.
5
1 , where 2 f l x 3.464 .
k=O
Twenty-five stones are placed on the ground a t intervals of
5 yards apart. A runner has to s t a r t from a basket 5 yards
f ~ o mthe first stone, p i c k up the stones and bring them back
to the b a s k e t one a t a time. How many yards h a s he to run
altogether?
Find the sum of n terms of t h e series whose rth term is

TL-10hikers s t a r t together on the same road. One of them


t r a v e l s uniformly 10 miles a day. The o t h e r t r a v e l s 8
miles the f i r s t day and increases h i s pace by half a mile each
succeeding day. After how many days will the l a t t e r overtake
t h e former?
How many terms of the sum 1 + 3 + 5 + ,., are needed to
give 12321 ?
Find a3 = 5 , and
(a) Find the sum of all even integers from 10 to 58
inclusive.
f b ) F i n d the sum of all odd integers from 9 t o 57 in-
clusive.
A p e r s o n saved t h i r t y cents more each month than in the
preceding month and In twelve years all o f h i s s a v i n g s
amounted to $9,424.80 . How much did he save t h e f i r s t
month? 'Phe last month?
If f o u r quantities form an arithmetic sequence, show that
( a ) The sum o f t h e squares of the extremes is greater
t h a n t h e sum o f t h e squares of the means.
( b ) The product of the extremes is less t h a n the product
of the means.
( a ) A constant c is added to each term of an arithmetic
progression. Is the new series also an arithmetic
progression; if so, what is t h e new difference and how
is the new sum r e l a t e d to the o r i g i n a l sum?
(b) If each term of an arithmetic progression is multiplied
by a constant c is t h e new s e r l e s a n arfthmetic
progression; if so, what is t h e new difference and how
is the new sum related to t h e o r i g i n a l ?
( c ) A new series is o b t a i n e d by adding a constant c to
each term of a geometric Progression. Is t h e new
series a geometric progression; if so, what I s t h e new
r a t i o and how is t h e new sum related t o the old?
( d ) A new s e r i e s is obtained by t a k i n g the reciprocal of
each term of an arithmetic progression. Is t h e new
series a n arithmetic progression? What is the new
difference?
( e ) A new series is o b t a i n e d by taking the reciprocal of
each term of a geometric progression. Is the new series
a geometric progression? What is the new ratio?
(f ) Do t h e negatives of each term of a geometric progression
form a geometric progression? If s o , ~rhatis the new
ratio?
al+l
21. Find the value of 1

22. Prove that if


1
,-
1
+ ,-
a are +
in arithmetic

progression, then a , bP , and c 2 are in a r l t h r n e t l c


progression. he converse is also true. )
3 Find the sum of 1 + 1
+ ... to n terms.

Hint: r a t i o n a l i z e t h e denominators.
24. If t h e sum of an arithmetic progression is t h e same for m
terms as f o r n terms, m # n , shou t h a t the sum o f
m + n terms is zero.
*25. The sum of m terms of an arithmetic progression is n ,
and the sum of n terms is m . Find t h e sum o f m -F n
terms, (m # n) .
Chapter 14
PEFWTATIONS, COMBINATIONS, A N D THE BINOMIAL THEOREM

14-1. Introduction. C o u n t i n g Problems.


The process o f counting involves three fundamental Ideas.

( I ~ )The first is that of a pairing, o r one-to-one corre-


spondence. Thus we count o u r fingers, o r o u r guests a t dinner,
by associating with each one of t h e things being counted one of
t h e n a t u r a l numbers beginning w i t h 1 and taking them " i n order".
We stop t h i s process when we run out of fingers, guests, o r what-
ever it is we are counting.
( I ~ )The second idea is t h a t underlying addition. Oiven t w o
f i n i t e sets s h a r i n g no elements, the number of elements in t h e i r
union is t h e sum of t h e number of elements In each. Thus the
number of people a t a swimming party is t h e sum of the number in
t h e pool and the number n o t In t h e pool.

(Ij) The t h i r d Idea is that underlying m u l t i p l i c a t l o n .


Oiven n s e t s (where n is a n a t u r a l number) no p a i r of which
share any elements and each one of which may be paired w i t h the
set 1 , 2 , , , , m of a l l n a t u r a l numbers n o t exceeding the
n a t u r a l number m , the number of elements in t h e union of t h e n
given s e t s is n x m . Thus we may count the s t u d e n t s (or t h e
s e a t s ) in a classroom by m u l t i p l y i n g t h e number of rows by the
number in each row (provided each row has as many as any other row).

To i l l u s t r a t e these ideas we present a method ( i n v o l v i n g all


three of them) for proving

his formula was d i s c u s s e d in Chapter 13.)


Let us consider a collection of d o t s arranged In n rows,
each containing n + 1 d o t s (F'igure J 4 - l a ) . We p a i r the n
rows to t h e n a t u r a l numbers 1 , 2 , .
, n and the n + 1
columns to t h e n a t u r a l numbers 1 , 2 , ...
, n , n + 1 ; and
write the number a s s o c i a t e d w i t h each row at i t s left, that associ-
ated w i t h each column above. This is a use of t h e first idea, I1.
Now we draw a line across t h e array of dots as shown, d i v i d i n g it

.
i n t o two p a r t s ,
r 1 2 3 4 5 n n+l
I.\*

Fig. 1 4 - l a
In t h a t part below the l i n e , we f i n d that
the l S t POW has 1 dot ,
t h e Pd row has 2 dots ,
t h e 3rd row has 3 dots ,
--- --- --- --- - ----
t h e nth row has n dots .
Thus, s i n c e no two rows share any dots, o u r second fundamental
idea, I2 , asserts that t h e r e are

dots below the line. Call this number s . Then


Now, above the l i n e , we find t h a t
the 'Cnd column has 1 dot,
the 3rd column has 2 dots ,
the bth column has 3 dots ,
--- --- ------ --- - ----
the ( n + l ) S t c o l u m n has n dots .
Hence there a r e a l s o s dots above t h e d i a g o n a l .
Now, a p p l y i n g o u r second idea to these two parts of our array
we find t h a t s + s is the tots1 number of dots i n it.
However, our third idea, I3 , t e l l s us t h a t the t o t a l number
of dots is n(n + 1) , the number of rows times t h e number of
columns. Combining t h e r e s u l t s of our t w o counting methods, we
have
2s = n ( n + 1)
or'

Thus

In t h i s chapter we s t u d y a number of counting problems--i.e.,


problems whose s o l u t i o n may be made t o depend on the three funda-
mental ideas of c o u n t i n g . Such problems occur f r e q u e n t l y in
mathematics, science, social studies and many other fields. One
of t h e r i c h e s t sources of these problems is t h e t h e o r y of games
of chance. How many ways may one draw a s t r a i g h t f l u s h or a f u l l
house from a f a i r deck, or r o l l a seven w l t h a pair-of d i c e ? A
water molecule ( ~ ~ has 0 )three atoms, and therefore it is p l a n a r .
But a sugar molecule ( ~
011 ) has 45~ .
9 ~
How many ways may ~
these 4 5 atoms be arranged In space, and, how many of these-
arrangements are chemically feasible? We w o n f t answer all o f
these questions--certainly not t h e last one--but we s h a l l study
ways t o h a n d l e a great many of t h l s s o r t .
Relatively few of the examples we shall give and the
exercises we s h a l l s e t can be considered earth-shaking. The
interest w i l l always be in the theory, in the methods, and in
the Ideas they illustrate, and only very rarely, in t h e "practical
v a l u e " of t h e i r answers.
One of o u r objects i n developing t h i s t h e o r y is to o b t a i n
results which are ttgeneral"in the sense that the numbers involved
are arbitrary. Of course, when one uses any of o u r results--for
example, to answer questions about batting orders, snake-eyea,
molecular s t r u c t u r e , r o y a l f l u s h e s , or how to l i n e up a f i r i n g
squad--he is dealing w i t h a problem which comes w i t h definite
numbers of the t h i n g s i n v o l v e d . The value of our general t h e o r y
is t h a t it can cope with any number of such problems, matter
-
what numbers may be involved in each of them.
Given enough time (in some cases, millions of years) and a
large, fast, computer (some of which c o s t m i l l i o n s of d o l l a r s ,
plus upkeep) one could solve many of these s p e c i f i c c o u n t i n g
problems by listing all the p o s s i b i l i t i e s and tallying them. Our
object is just t h e opposite. We set for ourselves the task of
working such problems w i t h o u t actually listing a l l the possi-
bilities. For example, in S e c t i o n 14-4, we determine the number
of S e n a t o r i a l "committees" t h a t are theoretically possible. T h i s
number is so monstrous it would be e n t i r e l y o u t of t h e question
even to contemplate making a list of the committees. Very l i k e l y ,
t h e r e i s n t t enough paper in t h e world. But the answer to the
question of the Senatartal committees--which depends on the
number of Senators (100)--1s little or nothing compared to t h e
numbers in some of the counting problems which arise In connection
w i t h , say, a mole of gas, for there are 6.025 x lo2? molecules
in a mole. Indeed t h e number of possible committees which can be
formed among any group of people in t h e world is a t r l v i a l f t y
compared to the number of p ~ ~ ~ l b i l l tf fo er ~the chemical r e a c t i o n s
which m i g h t occur in a t o y balloon.
Many counting problems are a c t u a l l y i n f i n i t e l y many problems
exppessed a s o n e , Examples are hod many ways may a n a t u r a l number
n be w r i t t e n as a sum of four squares, or as a product of natural
numbers. The a n s w e r s depend on n , which may be any n a t u r a l
number. Thus specific cases l i k e n = 3 o r n = 10 , e t c . , m y
be handled by enumeration. B u t g e t t i n g a f o m l a good for
arbitrary n is a n o t h e r matter e n t i r e l y . Many such problems are
beyond a l l methods known at present. In t h i s chapter we discuss
a few of t h e known ones.

Exercises -1k-1

1. Consider t h e following array


L
L 0
L O G
L O Q A
L O G A R
L O G A R I
L O G B R I T *

L O G A R I T H
L O G A R I T H M
L O G A R I T H M S
Determine the number of ways one may s p e l l LOGARITHMS start-
ing with any one of the L I B and moving either down or to
t h e right to an 0, then either down or t o the r i g h t to a G,
eto., ending with the 5. ( ~ i n t : begin wlth the t o p two o r
three l i n e s , and determine the number of ways to s p e l l LO
and - LOG' t h e n work w l t h the f i r s t f o u r Lines, first f i v e ,
e t c . , u n t l l you recognize a pattern In your answers.)
2. (a) Write an addition t a b l e for the numbers from 1 to 6 .
(b) U s l n g t h i s table answer the fbllowlng questions about
honest dice games:
(i) What number 1s one "most likely" t o roll?
(ii) Is one "more l i k e l y " to r o l l a power of 2 or a
multlple of 3 ?
(ifi) Is one ''more likelyn to r o l l a prime or a non-prime?

1 - 2. Ordered m - tuples.
Suppose we wish to count the number of r o u t e a from A to C
v i a B in F i g u r e 14-2a. There are three paths from A to B
(denoted by a , b , c ) and four paths from B to C (denoted by w,x,y,z). '

Fig. 14-2a
Now a route is completely described by naming a pair of these
l e t t e r s , provided we choose one from the set [ a , b , c ] and the
o t h e r from t h e s e t {w,x, y, z ] . Thus ( a , x ) , (b,w) , (c, z )
d e s c r i b e such r o u t e s . We t a b u l a t e a l l the p o s s i b i l i t i e s in
Table 14-2a.

Table 14-2a
Each e n t r y in the body of t h e table d e s c r i b e s one of t h e
possible r o u t e s . We see there are twelve of them.
Although we have enumerated a l l t h e cases in a r r i v i n g at our
answer, we may now see that t h i s is q u i t e unnecessary.
Our problem can be described as determining t h e number of
ordered pairs which can be formed using an element of the s e t
[ a , b , c ] as f i r s t member and an element of [w,x,y, z ) as second
member. h he body of Table 14-2a exhibits a l l of t h e s e p a i r s . )
Since we are interested only in the number of p a i r s here we are
interested merely in the product of the number of members In each
of our sets--the number of rows in the table times the number of
columns.
We may s t a t e the fundamental I d e a involved in a general way
as f o l l o w s .

Given a pair of finite sets A1 and A 2 w l t h ,


respectively, nl and n 2 members each, there are
n1 x n2 ordered pairs, or couples, which may be formed
w i t h a member of A1 as f i r s t member and a member of A 2
a s second member. For o u r "route" problem, A1 = f a , b , c ) ,
A 2 = (w,x,y,z) , and nl = 3 , n, = 4.
We i l l u s t r a t e t h i s general p r i n c i p l e by t u r n i n g t o a number
of examples in which it m y be used.

-
Example 14-2a: A q u a r t e r and a dime are t o s s e d . How many
head-tall pairs a r e possible?
S o l u t i o n : There are two possibilities f o r the q u a r t e r {H,T)
and t w o for the dime [ h , t ) ,

The number of pairs I s 4 , the product 2x 2 .


Example 14-2b: A library c o n t a i n s 7 geometry books and
13 algebra books. How many ways may a student select two books,
one of them a geometry and the o t h e r an algebra?
S o l u t i o n : Here we are Interested in t h e number of c o u p l e s
(geometry, algebra). According to o u r principle the number of
such c o u p l e s Is 7 x 1 3 , o r 91 .
Example 14-2c: Bow many fractions may be formed whose
n u m e r a t o r is a n a t u r a l number n o t exceeding 10 and whose
denominator is a natural number n o t exceeding 15 ? ( I g n o r e the
f a c t t h a t some of these fractions represent the same r a t i o n a l
number. }
Solution: 10 x 15 = 150 .
Example 14-2d: Given a hundred men and a hundred women, it
is possible to form 10,000 different couples--although not -
simultaneously!

Let us now extend o u r "route" problem by supposing t h e r e a r e


five p a t h s j o i n i n g C to a fourth p o i n t D .

Fig. 14-2b
Let [rn,n,o,p,q) be t h e s e t of paths joining C and D .
Now
how many r o u t e s are available f o r a t r i p from A t o D via B
and C , using only the paths p i c t u r e d ?
[ s e c . 14-21
We have a l r e a d y found there are twelve routes from A to C
via B :

Taking advantage of t h f s knowledge, we may describe any r o u t e


from A t o D via B and C by couples such as

As before, we make a t a b l e .

Table 1 4 2 a
We have indicated only a few ot the entries in t h e body of the
t a b l e . Uslng o u r p r i n c i p l e we see at once t h a t there are 12 x 5,
or 60 , posslbilitles.
S i n c e each of o u r neiq "couples" d e s c r i b e s a route made u p of
t h r e e paths, we may drop t h e extra parentheses, x r i t i n g simply
(a,w,m) f o r ( ( a , w ) , m ) , e t c . , and r e f e r to ( a , w , m ) a s an
ordered triple. Thus each r o u t e f r o m A to D via B and C
in Figure 1 4 - 2 b may be described by an ordered triple. Some more
of these r o u t e s a r e (atx,n) , ( c , y , p ) , ( b , z , p ) Our principle .
t e l l s us there are 60 ordered triples whose flrst component is
an element of (a,b,c] , whose second component is an element of
[ w , x , y , z ) , and whose t h i r d component I s an element of [m,n, o,p,q).

We may make f u r t h e r extensions to ordered quadruples,


quintuples, e t c . :

and generally t a ordered -


m- t u p l e s :

(a1 , ap 9 . .. . am)

with m components. Here m is any n a t u r a l number.


As we saw in the case of ordered triples we may extend our
general p r i n c i p l e to covered ordered m - t u p l e s :
If A1, A 2 , ,.. , A m are f i n i t e s e t s having,
respectively, nl , n2 , . . . , nm elements, there
are nl x n2 x ... x ,n ordered m-tuples of the f o r m
(al , a2 , . .. , am) where al is a member of A1 ,
a2 a member of A2 , ... am a member o f Am .
Example 14-2e: In a certain club no member may r u n f o r more
t h a n one o f f l c e a t the same time. If In one election there are 8
candidates for president, 7 f o r vice-president, 4 for secretary,
and 1 f o r t ~ e a s u r e r ,how many ways may t h e s e offices be filled?
Solution: We uant the number of ordered quadruples
(al , a2 , a3 , a $ ) where A1 has 8 elements, A p has 7 ,
A3 has 4 , and Ab has 1 . Our principle tells us the answer
Example 14-2f: C o n s i d e r a club h a v i n g 4 members, 3
o f f i c e s , and a rule permitting any member t o h o l d any number of
these offices at the same time. How many waye may the o f f i c e s be
filled?
S o l u t i o n : The number of ways of filling t h e offices is the
number of ordered triples, each of whose components is any one of
t h e members of the club. Here m = 3 , A1 = A 2 = A3 and

"1 = "2 = "3 = 4 . Our p r i n c i p l e s tells us the ansvier is

Extending the result in Example 14-2f to cover t h e possibili-


ties for m - t u p l e s each of whose components are members of a set
h a v i n g n elements, we rind t h e r e a r e nm such m-tuples. For,
in t h i s case, we have

and
Thus

gives the number of all p o s s i b l e rn-tuples whlch can be formed,


each of whose components belongs to a g i v e n s e t having n elements.

Exercises 14-2
1. A f u r n i t u r e company has t w e l v e designs f o r c h a i r s and f i v e
designs f o r t a b l e s . How many d i f f e r e n t p a i r s of t a b l e and
c h a i r designs can the company provide?
2, How many p i t c h e r - c a t c h e r p a i r s may b e formed from a s e t of
f o u r pitchers and two c a t c h e r s ?
3. How many pitcherrmug p a i r s may be formed from a set of e i g h t
p i t c h e r s and eleven mugs?
How many d i f f e r e n t committees consisting o f one Democrat and
one Republtcan may be formed from twelve Democrats and eight
Republicans?
How many ways m y a consonant-vowel pair be made using t h e
letters of t h e word STANFORD ?
Row many consonant-vowel pairs may be formed from the l e t t e r s
of the word COURAGE ?
How many numerals having two d i g i t s m a y be formed using the
digits 1 , 2 , 3 , . .. ,8 ,9 ?
Ten art students submitted posters in a contest which was to
promote s a f e t y . How many ways could two p r i z e s be awarded
if one prize was t o be given on the basts of the art work and
the o t h e r on the basis of the safety slogan chosen?
There are four bridges from Cincinnati to Kentucky. How many
ways may a round t r i p f r o m Cincinnati t o Kentucky be made if
t h e r e t u r n is not necessarily made on a d i f f e r e n t bridge?
How many ways may a two-letter'kord" be formed from a twenty-
s i x l e t t e r alphabet? ( A "wordt1need not have meaning.)
Bow many different triples of Ace-King-Queen can be
s e l e c t e d from a deck of 52 cards?
How many three digit numerals representing numbers l e s s than
600 may be formed from the digits 1 , 2 , 3 , .. . ,8 ,9?
Using the digits 1 , 2 , 3 , 4 , 6 , 8 , how many three
d i g i t numerals may be formed if the numbers they represent
are even?
A t a c l u b e l e c t l o n there are f o u r candidates for p r e s i d e n t ,
four for vice-president, s i x for s e c r e t a r y , and s i x f o r
treasurer. HOW many ways may khe e l e c t i o n result?
A freshman student m u s t have a c o u r s e schedule c o n s i s t i n g of
a foreign language, a natural science, a social s c i e n c e , and
an Engliah course. If there are four choices f o r t h e foreign
language, s i x for t h e n a t u r a l science, three f o r the s o c i a l
science, and two f o r English, how many d i f f e r e n t schedules
are available for freshmen?

[ a e c . 14-21
TQg.,Super-Super t o f fe m twalve bcscly rrtylsrs, three
W .

d&g$s~ante-ql, arrd , m ehundred


.. . twsrr$y color schemes.
How m x y csrrs n l l l a dealer .need in order t o show all
pgselble caps? ,. . = ,
.-
# "

HOW many guadmpleb ~ c ~ - b $ d & u e ~ n - d W+y k .be formed


from a brlclge deok?
From twelve mapeulipe, nwe' Pdmlnine, and ten neuter words,
. .

how many ways arm there t o aelect an example c o n a l a t a of


one of asch tme?
~n ~ = - c ~ a o ~mP' Q Cof n o t a ~ t m jhow .am ymezuls i n
there miokt -e';~ivm :digits?.:
A g a g ~ o l o ~ $ r ~u ~wt ar mze having ten points at
whlch. , &vm le%t or r w t . Hpw q n y waya c o U d er
r a t rn. thb .
. r ~ a ii.
< r he aollovrsd w diit$epnt route asoh time?
~ s h t h Ub ~ t 03 , 5 , 6 ,,7 . , . : 9 h o g - a n y hr*.digit
numerals reprrsenting numbers gpemter than .500. can be
i o ~ & aii (a) repetition o i h i g i t t i IS-allowed; (b) no. A * .

repetitfan of digits I s alSowed?


on a n y c o m ~ t t c e mc o n a 1 8 ~ i & & ~ - ~ am m o c r e t and a Rapublican
may be io-d i = o i f i G e w c r a t s aid eight ~egkbllbanati a
. ,

c e r t a i n Democrat mfuaes to work w i t h either of two


.. . .
Republioana? , ,

There are five boy8 and e m t girls a t a d-0, 3X B@@$bah


and m e u a , o e f i a g t~ dance wi$h a i t h r 3- or W ,. ~ d
obediah will .not +ace w i t h athir..wpaiW QF- e # A b W , how

2 -
i 14-3: Pemutatlona,
G ~ e tA be the-'set
.. , (a,b, c) .
Me. W d d &.:ordered couples
,

which my be fo-d u a l h g elements of 'A


,
1 ,. .4
.
We bee there are two
k i n d s : (i) those in which d u p l i c a t i o n s occur, (it) those w i t h o u t
d u p l i c a t i o n s . Thus ( a , a ) , ( b , b ) , ( c , c ) are of t h e first kind;

Table 14-3a
the o t h e r s of the second. There are 3 of t h e former--one f o r
each element of A --and 6 , or 32 - 3 , of the latter.
2
In general, given a set having n elements we may form n
ordered c o u p l e s whose components are members of the g i v e n s e t . Of
these n 2 couples, there are n (one f o r each element) which
have dupllcations. Hence there are n2 -
n w i t h o u t duplication.
Those ordered rn-tuples of elements of a set having n
elements which have no duplications are called permutations -- of t h e
-
n elements taken m - -
at-a- time or, f o r b r e v i t y , m permutations
of t h e s e t . Of course, m < n - .
of a s e t having n elements is n - n
Thus t h e number of 2-permutations
2
.
The n-permutations of an n-element s e t are called simply
permutations of the s e t .
There a r e many problems in mathematics, science, and o t h e r
fields--including gambling--which may be solved w i t h a knowledge
of the number of m-permutations of an n-element set. We have
determined t h i s number f o r m = 2 .
We proceed to larger values
of m .
As a preliminary, let us look again at the couples. We con-
sidered a table w i t h n rows and n columns. To avold d u p l l -
c a t i o n s we omitted one couple from each row. Since we want only
the number of 2-pemutations, and - not a l i s t of them, it makes no
difference in o u r c o u n t i n g problem if we simply remove a whole
column from t h e complete table, rather t h a n j u s t one couple here-
and-there in each row. Deleting one of the columns givea us a
"reducedt't a b l e w i t h n rows, n - 1 columns, and hence n ( n - 1)
e n t r i e s . This number checks w i t h o u r previous "count" n 2 - n
and can be made t o appear " p l a u s i b l e " if we t h i n k of the formation
of ordered couples without duplication as a p a i r of llcholces''. We
are free to choose any of the n elements as first component and
any of t h e remaining n - 1 elements as second component. Since
o u r "reduced" table has n(n - 1) entries we can say t h a t t h i s
p a i r of c h o l c e e may be made in n(n - 1) ways.
Moving on to 3 - p e m t a t i o n s of a s e t having n elements
(i.e., ordered triples w i t h o u t duplication) we can imagine a table
listing the 2-permutations on the l e f t (there are n ( n -
1) of
them) and the n elements of the s e t across t h e t o p . (FOP
example, Table 14-3b, where n = 3 ) .

Table 14-3b
To a v o i d duplicatLons in the t r i p l e s we must o d t 2 t r i p l e s
from each of the n rows. As there are (n2 n)n spaces In -
the t a b l e (number of rows timee number of columns) and 2 blanks
in each row, there are
-
(n2 n)n - 2(n 2 - n) ,

or (n2 - n)(n - 2) , or n(n - l)(n - 2) , entries in the table.


As w i t h the couples, we are only interested in the number of
t r i p l e s . The same r e s u l t may be obtained by simply deleting 2
columns, leaving n ( n - 1 ) rows and n - 2 columns. Hence we
have a n o t h e r way of seeing t h a t n ( n -
l)(n - 2) is the number of
3-permutations of a s e t h a v i n g n elements.
Carrying on the same reasoning we may move to quadruples,
quintuples, ...
, m-tuples, and we g e t

respectively, f o r the number of each having no duplication.


A great v a r i e t y
of symbols is used to denote the number of
rn-permutations of a s e t having n elements. Some of the more
p o p u l a r ones are

We shall use the l a s t of these in this book.


Writing ~ ( n , m ) for the number of m-permutations of an n
element s e t , our result may be expressed by t h e formula
P(n,m) = n(n - l)(n - 2).,.(n - m c 1) .
When m = n , we have

[ s e e . 14-31
The l a a t product occurs so frequently In these and other
problems, a s p e c i a l notation has been ,Introduced f o r it:

The expression " n! " l a read "n f a c t o r i a l It .


As examples of "factorials" we have

Observe the following property of f a c t o r i a l s :


n! = n(n - l)! or [n + I)! s (n + l)(n!) .
With t h i s formula we may calculate n! for each n a t u r a l number n
by a step-by-step process. However, s i n c e the numbers grow so
f a s t these calculations soon get t o o involved. Recourse to tables
is recommended. ( ~ the t end of Section 14-4 we give a table of
the common logarithms of n! for n up to 100 . This table will
be useful f o r many of the computations arising in the next section.
B u t it may a l s o be used, in conjunction w i t h the logarithm table
f o l l o w i n g it, to get approximations to n! for n up to 100 ) .
The equation

suggests the possibility of extending t h e definition of a! to


include n = 0:

We s h a l l ffnd that doing this will enable us to simplify many of


the problems we consider in t h i s chapter.
Using f a c t o r i a l notation, our formula f o r ~ ( n , m ) may be I
expressed q u i t e compactly: 1

= n(n - l)(n - 21...(n -m (n-m n-m-1 ,- 3 2 1.


+ 1 ) (n-m;[n-m-1{..,3 - 2 . 1
. .

Interpreting O! t o be 1 , t h i s expression holds even when m = n , ,


I
fora then
1

which agrees h - i t h o u r previous expression for ~(n,n) .


Example 14-3a: In a contest w i t h twelve e n t r i e s , how many
ways c a n a f i r s t and a second prize be awarded if no e n t r y Is
e n t i t l e d to more than one prize?
S o l u t i o n : O u r problem calls f o r the number of couples of the
form (one entry, another entry) w i t h o u t d u p l i c a t i o n , where the
f i r s t entry wins f i r s t prize and the other wins second p r i z e . The
number of such couples is ~ ( 1 2 ~ 2 , )s o the answer is 12 x 11
o r 132 .
Example 14-3b: A map of four countries is t o be c o l o r e d ; t i t h
a d i f f e r e n t c o l o r for each c o u n t r y . If s i x c o l o r s are a v a i l a b l e ,
ho-d many different ways may the map be c o l o r e d ?
S o l u t i o n : We xant the number of 4-permutations of a s e t
having six elements. Each quadruple has the f o m ( c o l o ~ ?o f f i r s t
country, color of second, c o l o r of third, color o f fourth). The
answer is p ( 6 , 4 ) , vrhich is 6 x 5 x 4 x 3 or 360 trays.
-
Example 14-3c: Suppose a class of twenty students decides to
l e a v e t h e room in a d i f f e r e n t order each day. How many days would
be required f o r the class t o leave the room in all p o s s i b l e o r d e r s ?
Ansder: 20! days. If t h e y work at it 365 days a year, it
will take approximately 6.7 quadrillion years. (20! 2 2,4329 x 1013 ) .
Even if t h e y lrlent through the door In a d i f f e r e n t order every second,
it would take over 70 billion years.

Example 14-3d: H A many ways may the numbers be arranged on


a r o u l e t t e wheel? (There are 38 "numbers": 00.0 , and the
n a t u r a l numbers 1 , 2 , 3 , , 36 . ) ..
First S o l u t i o n : If it is an honest wheel we cannot distinguish
any one place from any o t h e r . Thus no matter i:here 00 may be i!

p l a c e d , there a r e 37! days of arranging the numbers


I
0 , 1, 2 , ... , 36 . (1f it is n o t an honest wheel, so t h a t the :I
places a r e distinguishable, the number Is 3 8 ! .) I
I
Second S o l u t i o n : Let us consider the s e t of a l l 38-pemnuta- ' !
t i o n s of {OO, 0 , 1 , 2 , 3 , ...
, 36) .
T h e i r number is j
~ ( 3 8 , 3 8 )= 38.'. Corresponding to each such 3 8 - ~ e r m u t a t i o n ,
:I
' I

(al ap, 3 a38) #

there a r e 37 o t h e r permutations i

.
I

(a2 ' a3 9 aq , . , a38 , all


(a3 . aq , ... , a38 , al , a2) ,
-- ---------------------*-----

(a38 , "1 , ..- a a3fj, a36 a37) r

which cannot be distinguished from it on the (honest) irheel.


Thus i f N i s t h e number of "distfnguishable" permutations of
[Oo, 0 , 1 , ... , 361 , we have

In the general case, there are (n - 1 ) circular permuta-


t i o n s of a s e t h a v i n g n elements.

Exercises 14-3
How many five letter "kiordsl' may be formed from the l e t t e r s
A, B, C, D, E, F, and G ? Ho+r m n y if no l e t t e r i s repeated?
How many gays may a p r e s i d e n t , vice-president, and secretary
be elected from a club of twenty-five members if any member
may h o l d any one of t h e t h r e e o f f i c e s , but no member may h o l d
more t h a n one o f f i c e simultaneously?
HOX many t h r e e digit numerals may be formed u s i n g the digits
1 , 2 , 3 , 4, 5 , 6 if no digit is repeated in a numeral?
H a r many i f r e p e t i t i o n s a r e allowed?
How many four d i g i t numerals may be formed using the digits
1 , 2 , 3 , 4 , 5 , 6 if no digit is repeated in a numeral?
How many i f r e p e t i t i o n s are allowed?
How many four d i g i t numerals may be formed u s i n g t h e d i g i t s
2 , 4 , 6 , 8 if no d i g i t is repeated In a numeral? How
many if repetitions are allowed?
How many seven l e t t e r "wordsT1may be formed u s i n g the letters
of the word STANFORD ? Bow many i f no l e t t e r i s repeated?
How many different arrangements may be made f o r seven books
on a shelf i f t h e books a r e each of a d i f f e r e n t size?
Four persons are to r i d e In an a i r p o r t limousine having six
empty s e a t s . How many d i f f e r e n t ways c o u l d they be seated?
Three t r a v e l i n g salesmen arrived a t a town having f o u r h o t e l s .
How many ways could they each ohoose a different h o t e l ?
How many different combinations may be s e t on a lock having
twenty numerals if the combination is a 3 - p e ~ m u t a t i o n ?
How many d i f f e r e n t b a t t i n g o r d e r s may a b a s e b a l l team manager
form if he does n o t consider changing any but the l a s t t h r e e
places in t h e order?
How many d i f f e r e n t ways may the l e t t e r s a, b, c, d , e, f
be arranged w i t h no repetitions so as to begln w i t h ab in
each case?
How many t h r e e digit numerals having no repeated digits may
be formed from t h e digits 1 , 2 , ? , 11 , 5 so t h a t the
middle d i g l t is 3 ?
How many 5-permutations including t h e l e t t e r C may be
formed from the l e t t e r s A, 3, C, D, E, F, G ?
How many ways may a photographer arrange four women and f i v e
men in t w o rows If women must stand in t h e first row and men
i n t h e second?
How many l i c e n s e p l a t e s may be made u s i n g t w o letters of a
twenty-six letter a l p h a b e t f o l l o w e d by a f o u r d i g i t numeral?
( Z e r o may be used at any p l a c e in t h e n u m e r a l . )
H3w many ways are there f o r eight children to form a r i n g
around a May Pole?
If the number of ways t o l a g a s e t of t i r e r i e i g h t s in a line
is s i x times the number of ways they may be p l a c e d on the
t i r e rim, how many weights are there?
How many ways c o u l d K i n g A r t h u r and eight of h i s k n i g h t s sit
at t h e Round T a b l e i f one of the s e a t s was a t h r o n e c h a i r for
King Arthur o n l y and there a r e eight o t h e r s e a t s ?
20. Find t h e exact n u m e r i c a l v a l u e of each of the following:

(4 ~
15'
(f) m
15!
(I) 5(5J + 4 ! )

21, Solve f o r t h e n a t u r a l number n :

(b) ~ ( n , 5 )= 20 - P(n, 3) . (d) ~ ( cn 1,3) - 10 - -


P(n 1,2)= 0 .
22. Simplify (n and m are natural n u m b e r s ) :

(e) (n -m - 2)!(n - rn - l)(n - m) , m l n


(f) nln! + (n - l)!] , l < n .
1 1
(9) m+s*1 < n .

(h) ntn!
n -t.
+l)y I l.311
(n + 2)! + (n - l)! ( n + 1)
(n - lJ!(n + 1) 1 l < n 4

23. h o v e each of the following f o r n a t u r a l numbers m and n


(a) ~ ( n , 3 )+ 3 .
~ ( n , 2 )+ ~ ( n , l )= n
3
.
(b) (n + l)[n - n! + ( & - l)(n - l)! + (n - l)(n - 2)!]
= ( n + 2)!, 1 (n.
(c) P(n + 1,m) = (n + 1) . P(n,m - 1) , r n-< n + l .
(d) ~ ( n , m )= m . ~ ( -n 1, m 1) + ~ ( -n 1 , m ) , m<" -
(e) ~ ( n , m )= ~ ( -n 2 , m ) + h . ~ ( n 2 , m - 1) + m(m 1) - -
- ~ ( -n 2 , m 2) , m s n - 2 . -
1h-4. Combinations.
In t h i s section we c o n s i d e r the f o l l o w i n g c o u n t i n g problems:
( 1 ) Given a finite s e t having n elements, how many s u b s e t s
does it h a v e ?
(ii) Given a f i n i t e s e t having elements, how many
n
1-element, 2-element, 3-element, ...
, m-element subsets does it
h a v e ? (Here m Is any n a t u r a l number not exceeding n . )
Problem (i), being the e a s i e r , we consider first. Suppose
our s e t h a s the elements

.
The v a r i o u s s u b s e t s of' {al , a 2 , , , , a n ) may be formed by
going down the list of members and f o r each member e i t h e r t a k i n g
it or n o t t a k i n g i t . The process of f o m i n g a subset of
(al , a2 , ... , a n ) c a n therefore be described by giving a n
ordered n - t u p l e , each of whose components is e i t h e r T (meaning
"take") or D (meaning "don It t a k e " ) .
For example, with n = 4 , our s e t is (al , a p , a3 , a b ]
and the quadruple (T , T , D , D) yields t h e s u b s e t (al,a2) :
[al , a2 , a3 , a41
( L T , D , Dl
' a* I .
The quadruple (D ,T ,D , T) gives [a2 , a4):
(al , a2 9 a3 9 a41
That each subset is described by such a ''list of i n s t r u c t i o n s " is
illustrated by the following scheme. Given the subset {a3 , a h )
we have
[al , a2 9 a3 ' a43

Since each subset corresponds to e x a c t l y one of these ordered


n-tuples, the number of subsets of {al , a 2 , , , , an) is the .
same as t h e number of ordered n-tuples one can form from t h e
elements of the set IT , D) .
A t the end of Section 14-2, we
found t h a t the number of such ordered n-tuples is 2" Thus we .
h a v e the following theorem.

Theorem 14-4a: There are 2" subsets of a f i n i t e set which


has n elements.
Note, in particular, two " s u b s e t s " which have been counted.
They are the extreme cases i n which the n-tuple has a11 T t s
(the "subset" corresponding to t h i s n-tuple is the whole s e t ) ; and
t h e case in which t h e n - t u p l e has a l l D ' s (the "empty" or "void"
subset, c o n t a i n i n g none of t h e members of the given s e t ) .

mample 14-4a: S i n c e there are 100 Senators, the t o t a l


number of Senate Committees which can be formed is 2loo - 1 , I T
committee of t h e whole but exclude the committee
we i n c l u d e t h e
w i t h no members. ThLs number is

W e now consider P r o b l e m (ii): Glven a finite s e t having n


elements, how many rn-element subsets does It have, where m I s
any n a t u r a l number not exceeding n ? An rn-element subset of a
set having n elements is often called a --
combination of the n
elements taken m - at-a- t i m e .
Let us look a t some examples. Given t h e s e t [ a , b , c ) with
t h r e e members, we have the f o l l o w i n g nsn-empty subsets

This example is rather simple, b u t it t e l l s us a good d e a l


about the general q u e s t i o n . Thus a s e t having n elements has n
s u b s e t s each w i t h 1 element (one such s u b s e t corresponding to each
-
element--the one In t h e s u b s e t ) . A s e t having n elements has n
subsets each w i t h n - 1 elements (one such subset corresponding
to each element--the one - n o t in the subset). And, o f course,
there is only one n-element subset of a s ~ hta v i n g n elements;
it is the whole set.
There are many d i f f e r e n t ways in use t o denote the number of
m-element s u b s e t s of a set h a v i n g n elements. Some are

The l a s t one of t h e s e we adopt in this book.


We have j u s t seen t h a t

Now let us consider t h e s u b s e t s of [ a , b , c , d ] . H e r e n = 4 .


W e already know C ( 4 , l ) , C(4,3) , C ( 4 , 4 ) . We have only t a
determine ~ ( 4 ~ .2 )The f o l l o w i n g scheme exhlbits the 2-element
subsets of {a,b, c, d ) :
E a , b , c , d l

There a r e six. Thus ~(4,2) = 6 .


We may d e t e c t a connection between our c u r r e n t problem and
t h e permutation problems considered in S e c t i o n 1 4 - 3 if we compare
o u r l a s t list w l t h Table 14-4a exhibiting the o r d e r e d c o u p l e s
which may be formed with elements of [ a , b , c , d ] .

Table 14-4a
The subsets

Ea,bl Ca,cl Ia,dl ,


Ib,cl , b,d3 ,
k,dl
a r e represented in Table 14-4a by the couples

appearing in the upper r i g h t - h a n d corner. B u t they are also


represented by the couples in t h e lower left of Table 1b-4a:

Esec. 14-43
Let us match these couples to the s u b s e t s as follows

Examfng t h i s arrangement, we see t h a t each ordered couple to the


right of the l i n e is a 2-permutation of t h e set a t the left of i t s
row. For each such 2-element s u b s e t , there are therefore ~ ( 2 , 2 )
2-permutations. Hence ~ ( 2 , 2 ) is t h e number of c o l u m n s t o the
) the number of rows. S i n c e the
r i g h t o f the line. ~ ( 4 ~ 2is
t o t a l number of 2-permutations we can form from { a , b y c , d ] is
~ ( 4 , 2 ) , we have

and hence

We now consider a set having n elements. C ( n , m ) denotes


the (unknown) number of its m-element s u b s e t s . Let us imagine a
table in which each of these m-element subsets determines a row.
In each of the rows we write t h e p ( m , m ) m-permutations of the
subset which identifies the row, The total number of e n t r i e s i n
t h i s table is ~ ( n , m ) , t h e number of a l l m-element permutations
of the g i v e n s e t . M u l t i p l y i n g the number of rows and the number
of columns we have
c ( n , m ) ~ ( m , n >= ~ ( n , m )

n!
m! m!(n - m)!
Example 14-4b: Two cards are d e a l t from a deck of 52 cards.
How many ways may t h i s be done?
Solution: We want t h e number of 2-element subsets of a
52-element s e t .

Example 14-4c: How many 5-card poker hands containing the


ace of spades a r e possible w i t h a 52-card deck?

Answer: ~(51~=
4%)3 . 7! , Using the t a b l e for lag n!
(following these examples) we find
l o g 4! = 1,3802 log 51! = 66.1904
log 47.' = l o g b!(47!) = 60.7929
l o g 4!(117!) = 60.7929 log ~ ( 5 1 , b )= 5.3977
hence C(g1,4) 2 2.5 x 105 ,

-
Example 14-4d: Show that c (n,m) = ~ ( n , n m ) and i n t e r p r e t
t h i s formula in terms of the subsets of a g i v e n s e t .

S o l u t i o n : ~ ( n , n- m ) = ( n n! n!
- rn]!(n - (n - mj]!
=
I n - m]!rnT

~ ( n , m ) is the number of rn-element subsets of a s e t h a v i n g n


elements. Each of t h e s e s u b s e t s may be paired w i t h an (n - m)-
element s u b s e t of t h e same s e t ; namely, the subset c o n t a i n i n g none
of the members o f t h e o r i g i n a l subset. This p a i r i n g shows that the
m-element s u b s e t s and the (n - m)-element subsets of a given s e t
are e q u a l l y numerous; and t h a t is exactly what the formula s t a t e s .
Example 14-be: How many ways may a n a r b i t r a r y n a t u r a l number
n be represented a s a sum of m n a t u r a l numbers if we regard sums
d i f f e r i n g fn t h e order of t h e i r terms a s d i f f e r e n t "representations It?
S o l u t i o n : We look first a t a special case: n = 5 ,m = 3 .
L e t us c o n s i d e r 5 " t a l l i e s t tin a row

Our problem is e q u i v a l e n t to s p l i t t i n g t h i s row of tallies i n t o


three parts. Thus

Irl l*I 1
yields the sum 1 + 2 + 2 ;

yields 1 + 3+1, and


1*1 ll*l

yields 1 + 1 -t 3 . Splitting the row of t a l l i e s into three p a r t s


is accomplished by selecting 2 of the four spaces between adjacent
t a l l i e s . T h i s can be done in C(4,2) , or 6 ways.
In the general case we have n t a l l i e s with n - 1 spaces:

Each representation of n a s a sum of m terms c o r r e s p o n d s to a


s e l e c t i o n o f a n [m - 1)-element s u b s e t of the set of n - 1
spaces. The number is t h e r e f o r e

-
Example 14-4f: If a c l a s s h a s 20 students and t h e c l a s s -
room has 2 doors and 3 windows, how many d i f f e r e n t ways may the
t e a c h e r and the students l e a v e in case of a fire if a t l e a s t one
p e r s o n goes t h r o u g h e a c h of these e x i t s ?
S o l u t i o n : Since i t ' s "every man f o r himself", we t r e a t a l l
21 s o u l s on an equal basis. The n a t u r a l , number 21 may be
written a s a sum of 5 n a t u r a l numbers (one term f o r each of the
-
e x i t s ) in ( 2 1 , 5 - 1) ways, regarding a s distinct such
representations differing In t h e order of their terms.

(The s i z e of the answer j u s t i f i e s having a plan of egress ahead of


time, obviating numerous hasty decisions.)

Example 14-4g: How many b r i d g e hands of 13 cards c o n t a i n


exactly 5 spades?
S o l u t i o n : We want the number of ordered couples ( A , B ) ,
where A is a set of 5 spades and B is a s e t of 8 non-spades.
There a r e C(13,5) possibilities for A and ~ ( 3 9 , 8 ) p o s s i -
b i l l t i e s for B . Hence there are ~ ( 1 3 , 5 )x C(39,8) such
couples.
11
C ( 1 3 , 5 ) ~ ~ ( 3 9 , 8 ) ~ : : . 9 2 x l O.

Csec. 14-4 I
COMMON LOGARITHMS OF n !
n lop; n! n lop; n! n log n! lop; n!
0 0.0000 25 25.1907 50 64.4831 109,3946
1 0.0000 26 26.6056 51 66.1906 111.2754
2 0.3010 27 28.0370 52 67.9067 113.1619
3 0.7782 28 29.4841 53 69.6309 11 5.0540
4 1,3802 29 30.9465 54 71.3633 116.9516

118.8547
120.7632
122.6770
124.5961
126.5204

128.,1498
130.3843
132,3238
134.2683
136.2177

40 47.9117 65 go. 9163 138,1719


41 49.5244 66 92.7359 140.1310
42 51.1477 67 94.5620 142.0948
43 52.7812 68 96.3945 144.0633
44 54.11246 69 98.2333 146.0364

148.0141
149.9964
151.9831
153.9744
155.9700
!E TABLE OF COMMON LOGARITHMS
FOUR- ??LAC
Exercises 14-4
Using t h e set [a, b, c, d):
( a ) Find t h e number of subsets.
( b ) Ljst the 3-element subsets.
(c) L i s t the
3-permutations for each of t h e 3-element
subsets.
( d ) F i n d the value of ~ ( 4 ~ 3 ) .
E v a l u a t e each of the following:
(a)
(b)
~(10,2)
C(8,3)
(dl ~(25,241
(4 C ( l 2 , 1 0 )
w (9)
c a 5

c 26 21
( 4 c(12,5) (f) ~ ( 1 0 0 , 9 8 ) M (h)
Calculate the v a l u e of log loo! to f o u r decimal p l a c e s .
A student is i n s t r u c t e d to answer any eight of t e n q u e s t i o n s
on a n examination. How many different ways a r e t h e r e f o r him
t o choose the guestions he answers?
There a r e t e n entrles in a round-robin tennis tournament.
HOW many matches m u s t be s c h e d u l e d ?

How many distinct l i n e s a r e determined by f i f t e e n points on


a p l a n e i f no three of t h e points are c o l l i n e a r ?
How many t r i a n g l e s are determined by e i g h t points on a plane
if no t h ~ e eof t h e p o i n t s a r e c o l l i n e a r ?
A seed company t e s t s its t u l i p b u l b s in s e t s of sixteen.
Four bulbs a r e selected f o r p l a n t i n g from each s e t . If all
f o u r grow, t h e remaining twelve a r e s o l d with a guarantee
t h a t a t least e i g h t of them w i l l grow. How many ways can
the f o u r b u l b s be selected for test planting from a s e t o f
sixteen?
How many committees of f o u r members may be formed from a s e t
of n i n e possible members?
How many committees consisting of two Democrats and two
Republicans may be formed f r o m a s e t of seven Democrats and
six Republicans?
How many parallelograms a r e determined by a s e t of e i g h t
p a r a l l e l l i n e s Intersecting another set of five p a r a l l e l
lines?
A basketball squad c o n s i s t s of four centers, f i v e forwards,
and s i x g u a r d s . How many different teams may the coach form
if p l a y e r s can be used only at t h e i r one position?
From a s e t of twenty consonants and the five vowels, how
many "words" may be formed consisting of t h r e e different
vowels and two d i f f e r e n t c o n s o n a n t s If one of the vowels
must be a ?
How many five l e t t e r " i ~ o r d s "containing t w o vorrels and t h r e e
c o n s o n a n t s may be formed from t h e l e t t e r s of t h e word
LOGARf T r n ?
Referring t o the a r r a y g i v e n in Exercise 14-1, 1, determine
the number of ways one can s p e l l LOGARITHMS starting from a
g i v e n one o f t h e L t s , going right or down f o r the next l e t t e r
each t i m e , and ending at S. (Suppose the g i v e n L lies in
t h e m th POW from t h e bottom; then it is necessary t o move
down m - 1 times between successive l e t t e r s . ) Check y o u r
result by u s i n g t h e formula you o b t a i n t o solve mercise
14-1, 1 "again. "

Using the t a b l e f o r log n! f i n d approximate answers f o r


Exercises 1 6 , 17, ... , 22.
A sample o f f i v e items is to be selected from a s e t of one
hundred. How many d i f f e r e n t samples m y be i'ormed?
How many d i f f e r e n t poker hands of f i v e cards each can be
formed from a deck of fifty-two cards?
How many saiiples of' t e n u n i t s may be formed from a s e t of
one hundred light bulbs?
How many subsets of f i v e cards c o n t a i n i n g exactly three aces
may be formed from a deck of fifty-two cards?
How many bridge hands can have two six-card s u i t s ?
How nany b r i d g e hands have one seven-card s u i t and three
two-card s u i t s ?
Row many b r i d g e hands have a "5-4-3-1" distribution?
23. If C(n,12) = c(n,8) , find
the v a l u e of ~(11,171 .
24. -
If ~ ( 1 8 , 4 ) ~ ( 1 8 , m + 2) = 0 , find the value of ~ ( r n , 5 ) .
25. Prove P a s c a l 's Theorem: c (n,rn) = C (n -
1, m - 1 )
+ ~ ( n - l , m ) ,l ( m ( n - 1 .
- -
26. Show t h a t Pascal 1s Theorem may be I l l u s t r a t e d by the follow-
ing table ( c a l l e d Pascal's ~ r f a n g l e ) ,where entrles in the
t a b l e are of the form c (n,m) f o r 1 I m n , and extend
t h e table t h r o u g h t h e line f o r n = 10 .

27. Prove t h a t ~(n,n - -


2) = ~ ( n I , n - 2) ~ ( -n 2,n
-+ - 3)
+ ... + ~ ( 2 , 1 )+ C(1,0), <n
if 3 - .
14-5. The Binomial Theorem.
We a r e a l l f a m i l i a r w i t h the formula
2 2 2
(x+y) = x + a y + y 4

Higher powers of the b i n o m i a l x + y may be expressed a s poly-


nomials in x and y by m u l t i p l y i n g each r e s u l t in turn by
Proceeding this way, we may derive the expansion of each h i g h e r
"
power, (x + Y ) , in a step-by-step f a s h i o n ,
Ho:4ever, it i s possible to apply o u r t h e o r y of combinations
to obtain the expansion of (x + y ) n where n is an arbitrary
n a t u r a l number. Thus we may a v o i d t h e step-by-step process and
write out the entire expansion f o r any given n n i t h o u t f i r s t
determining t h e expansion f o r each s m a l l e r v a l u e o f n The.
saving, therefore, in c a l c u l a t l o n is very great. Suppose, f o r
example, t h a t you need t o know the f l r s t 6 coefff c i e n t s in t h e
expansion of (x + y ) 100 (For reasons we cannot explain here,
such quest Ions of t e n a r i s e in scientif l c and sociological problems, )
Using the formula we s h a l l derFve,you would n o t have t o f i n d f i r s t
-
a l l the c o e f f i c i e n t s in a l l t h e expansions up to (x + y ) loo; the
6 coefficlents you wanted could be w r i t t e n darn w i t h o u t any
preliminary c a l c u l a t i o n s .
Before we attack the general problem, we recast it in a
simpler form. Note that
(x + Y 1" = [x(l + 5))" = xncl + 5)" .
If we set Z = , our problem amounts to determlnlng the expansion
of xn(l + z 1" .
This can be done if we determine the coefficlents
in the expansion of (1 + 2)" .
For a l l we need do w i t h t h i s ex-
pansion is multiply each term by xn .
Finally replacing z by
X
we can obtain t h e expansion or (x + y ) " ,
Me (1 + z l n
t u r n now to the expansion of In order to .
obtain the coefficients in t h i s expansion, we s h i f t our attention
to the product
(1 + zl)(l + z2) ...( 1 + zn) .
Note that when zl = z2 = ... = z
n = z , the product r e d u c e s t o
(1 + a)" s i n c e it haa n factors.

We look first at some examples. For n = 2 , we have


(1 + zl)(l + z2) = 1 + (zl + z2) + z1z2 .
For n = 3 :
(1 + zl)(l + zg)(l + z3) = 1 + (zl + z 2 + z3) 4- ( ~+ 22p 3 ~
+ z1z2) + z1Z2Z3 .
For n = 4 :

(1 + zl)(l + z2)(1 + z3)(1 + zl) = + (5+ z 2 + z3 -I- z 4 )


1
+ (z1z2 + z1z3 + Z1z4 + z 2 z 3 + z2z4 + z 3z 4 1
+ (z2z3z4 + zIz3z4 + z1z2z4 + z 1z 2z3 )
+ Z1z2Z3z4 .
Studying these examples gives the clue to t h e general p a t t e r n .
For n = 2 , consider the s e t {zlz2] Its non-empty subsets a r e .
b,I , b21 1 and [z1z21 i

each of xhich corresponds to a term in the expansion:

fz,l , ("*I , [z1z21 9


n = 3
S l r n l l a r l y for . W e list the non-empty s u b s e t s of
(zl , z2 , z3) and t h e terms appearing in the expansion which
correspond t o them:

The expansions themselves ( a t l e a s t for n = 2 , n = 3 ) are the


sums of t h e terms listed, plus the extra term "1' .
The same
p a t t e r n holds in t h e case n = 4 .
What happens in these cases, when zl , z2 , z3 , @tc., are
all replaced by z ?
The terms contributing t h e first power of z to the sum are
those corresponding to 1-element subsets; those contributing the
second porrer of z to the sum are those corresponding to 2-element
subsets; e t c . Thus t h e number of z f s (and hence the coefficient
of,
z i n the expansion) 1s c (n,l); the eoeffielent of z2 is
~ ( n , 2 ); and for n ) 2 , the coefficient of z3 is ~ ( n , 3 ) .
These bbservations are valid at l e a s t when n = 2 , n = 3 , n = 4 .
The binomial theorem asserts t h a t t h i a is the case f o r n an
arbitrary n a t u r a l number.
Tn t h e general case, expanding the product

ylelds terms of t h e following forms:


A t the right of each l i n e we g i v e t h e number of terms in the llne.
Replacing .
zl , 2 , ,, , zn each by z yields t h e terms

Adding these terms we have o u r expansion:

or, more compactly, n

if we agree to write C(n,O) = 1 .


R e t u r n i n g t o o u r original q u e s t i o n regarding t h e expansion
of (X + y)" , we have, p u t t i n g in place of z :

2
= xn(l + c (n, 1 )$ + ~ ( n 21%
, +
X
. .. + c (n, X

+ c (n, 2 ) x n - s 2 + . . . + c (n,n)yn
n
= x + c (n, l)xn-ly ,

= 1 ~ ( n , m ) n-m
x ym .

This is t h e binomial theorem.


-
Theorem 14-5a: If n is any n a t u r a l number, and if x and
y are any real (or complex) numbers, then
n

-
Example 14-9:

(X + Y)5 = x
5 + G ( 5 , l ) x4Y * C(5,2)x 3Y 2 + cI5,3)x 2Y 3

+ c(5,4)xY4 + ~ ( 5 . 5 ). ~ ~

Example 14-5b :

(x2 - 3 m4 = (x2)' + 4(x213(-3 fi]+ 6(x )


2 2
(-3 m2

Note that if we take x = y = 1 in Theorem 14-4a, we o b t a i n


n n

Thus the sum of t h e number of


0-element s u b s e t s (c(n,o))
I-element subsets (c(n,l))
2-element subsets (C (n,2)
--------------------
n-element subsets ( c (n,n) >
I of a s e t having n elements I s 2" .
Note that we have already
! seen that 2" is t h e t o t a l number of subsets ( i n c l u d i n g the empty

i
s e t and the whole s e t ) of a s e t w i t h
n elements. Thus the
binomial theorem ties together o u r s o l u t i o n s of the two problems
we considered in Section 1 4 - 4 . ( ~ f . The solutions of E x e r c i s e s
14-3,l a n d 14-4,15 . )

Exercises 14-5
1. Find the expansion f o r each of the following:

(4 ta+bI5 (i) (x2 + Xl8

(d) (a + b J 7 (j) (c2 -


pod) 9
-1 + a - 2 ) 6
( 4 (a+ v16 (k) (x

2. (a ) What i s the sum of t h e a, b exponents in t h e kth term


in the expansion of (a + b)n , n , k i n N and k ( n ?
(b) How many terms are t h e r e in t h e expansion of ( a + F)73?
In ( a + b l n , n in N ?
(c) which term in the expansion o f ( a + b13* 1 s the
m i d d l e term?
(d) F o r which values of n will the expansion of ( a + b)n ,
n i n N , have no middle term?
( e ) Give the C ( n , m ) f o r m of the c o e f f i c i e n t of the t ~ e n t y -
first term in the expansion of (a + b)35 ,
( f ) W l c h terms in the expansion of (a + b17* have their
coefficients equal to t h e coefficient o f the t h i r t y -
f i r s t term?
(g) If t h e coeff~cientsof the s i x t h and sixteenth terms in
t h e expansion of (a + b)" , n in N , a r e equal,
what is t h e value of n ?
(h) If t h e coefficients of t h e f o u r t h a n d s i x t e e n t h terms i n
the expansion of ( a + b)" , n in N , a r e equal, find
t h e middle term in the expansion.
3. (a) Find t h e s e v e n t h term in the enpanslon of (a + b ) 1 5 .
( b ) F i n d t h e f o u r t h term in t h e expansion of (x - 5)1' .
(c) F i n d the t w e l f t h term in the e x p a n s i o n o f ( a- 1)13 .
3 x 10
(d ) Find t h e middle term in t h e expansion of ;;( + J) ,

(e) Flnd t h e middle term in t h e expansion o f (1y - x ).


2 12

(1 - T) .
x 2 14
(f) Find t h e e i g h t h term in t h e expansion of
(g) F i n d t h e term h a v i n g b7 a s a f a c t o r in the expansion
of ( d -f- b)l0 ,

(h) Find the term h a v i n g y5 as a f a c t o r in the expansion


of ( A 2 - y )9 .
(1) F i n d t h e term h a v i n g x14 a s a factor in t h e expsnsion

(j) Find t h e term having y3 as a f a c t o r in the expansion


1

(k) Find t h e term -


NOT h a v i n g a f a c t o r of x in the
2
expansion of ( x -
1 12
x)
Find t h e n u m e r i c a l v a l u e f o r each o f t h e follo,:~ing to f o u r
decimal p l a c e s :
(a) 1 . 0 2 ~ (~lnt: 1.02=1+0.02) (e) lm9810
(b) 1.02~~ (f) ( 1 - 1 )8

(c) 0 . 9 8 ~ ~
(d) 2 . 0 1 ~ ~
14-6. Arrangements and P a r t i t f ons .
We have considered t h e p e r m u t a t i o n s a n d combinations o f
eleaents of a given finite s e t . In this section w e c o n s i d e r
another type of counting problem, one whose s o l u t i o n can be based
on o u r p r e v i o u s r e s u l t s .

Example 14-63 : How many distinguishable arrangements a r e


t h e r e o f t h e l e t t e r s i n t h e word "loon" ?

S o l u t i o n : I f t h e word were "loan" Instead of "loon", the


methods discussed i n S e c t i o n 14-3 would apply directly and give
t h e answer ~ ( 4 , 4 ) , i . e m , 24 . However, we may expect t h a t t h e
answer t o the present problem is much smaller, s i n c e iqe have
d u p l i c a t i o n s . Thus the permutations "loan " and "la on I' cox*respond
to the indistinguishable arrangements "loon " and "loon " in our
current problem. Suppose, indeed, t h a t we consider a complete
l i s t of the 4-permutations of the set [l,o,a,n] . These permuta-
t i o n s may be paired w i t h one a n o t h e r a s f o l l m s :
loan , laon; olan , alon; oaln , aoln;
nloa , nlao; nola , nalo; noal , naol;
anlo , onla; an01 , onal; lnoa , lnao;
oanl , aonl; lano , lona; alno , olna .
In each of these p a i r s the letter "1" occupies t h e same place, t h e
letter "n" occupies the same place, b u t t h e letters "o" , "a1' are
interchanged. R e p l a c i n g each "a" h e r e by an 'lo" iJe see that each
of t h e s e pairs yields a p a i r of i n d i s t i n g u i s h a b l e arrangements of
t h e letters of "loon". Thus t h e number of arrangements of the
letters of "loon" Is just half t h e c o r r e s p o n d i n g number f o r "loan".
O u r answer is therefore 12 .
This example provides the key t o t h e s o l u t i o n o f the general
problem of determining the number of arrangements o f a l i s t con-
taining r e p e t i t i o n s .
Corresponding to each arrangement of t h e letters of "loon"
we have ~ ( 2 , 2 ) permutat i o n s of (1,o , a , n ) a r i s i n g Sronl permuta-
t i o n s o f {o,a) ,
C o n s i d e r the problem of counting the number of arrangements
of t h e 12 letters of "divisibility". Here the letter "it' occurs
5 times, b u t each of the o t h e r 7 l e t t e r s o c c u r s just once. No?!
~(12,121 is the number of arrangements of t h e elements of a s e t
having 12 elements such as { d , a , v , e, s , i, b, o , l , u , t,y) .
Let us
w r i t e A f o r t h e (unknown) number o f different arrangements of t h e
l e t t e r s of "divisibility". Corresponding t o each of t h e s e arrange-
ments a r e ~ ( 5 ~ 5permutations
) of the letters d,a,v,e,s,L,b,g,l,
u,t,y .
Hence A x ~ ( 5 , 5 )= P(12,12)

and

S u p p o s e , i n g e n e r a l , we h a v e a lfst o f
n items, m of
which are t h e same but no two of the remazning n - rn are the
same. For example

Corresponding t o the given list, l e t us c o n s i d e r a second l i s t i n


which t h e duplicated ltema a r e distinguished (say, by subscripts).
In our example,
X L , X 2 , Y , Z , X 3 , U ' X 4 , v .

The number of a r r a n g e m e n t s of the second l i s t is ~ ( n , n ) BcIi.


arrangement of the first list corresponds t o ~ ( m , r n ) arrangements
of the second. If A is the number o f distinguishable a r r a n g e -
ments of the o r i g i n a l list,
Consider the corresponding problem for a list of n items,
rnl of one kind, m 2 of a second in3 of a t h i r d k l n d , e t c , , w i t h

If A is the number of distinguishable arrangements, then

T h i s number is w r i t t e n as

We n o t e t w o s p e c i a l cases.
If ml = m and m;, = ... = ink = 1 ,
as in t h e p r e v i o u s examples, we have

so t h a t o u r earlier formulas a r e special cases of t h e general


formula. If k = 2 and ml = m , then m2 = n - m and

Example 1 4 - 6 b : How many distinguishable arrangements are


t h e r e of the l e t t e r s i n MISSISSIPPI ?
S o l u t i o n : There are 11 l e t t e r s , 4 of one kind (I) , 4
,
of a n o t h e r ( S ) , 2 of a third (P) and . 1 o t h e r (M) . Our
f o r m u l a gives
Exercises 14-6a
1. How many d i f f e r e n t s i x d i g i t numerals may be written using
the digit 5 once, a 4 three times, and a 3 twice?
2. How many five symbol code 'bords" may be formed using t h r e e
d o t s and two dashes?
3. How many distinct "words" may be formed as arrangements of
the l e t t e r s of PARALLELEPIPED ?
4. Suppose t h a t on one dark day i n a c e r t a i n h o s p i t a l , f o u r
s e t s of i d e n t f c a l male twins, two s e t s of i q e n t i c a l female
twins, n i n e males (single b i r t h s ) , and e l e v e n females
(single births) are born, and cheap i n k is used on t h e i r
name-tags. me next day (even darker) t h e ink fades away.
How many ways is it posslble to mix the c h i l d r e n up? (Use
log n! table of S e c t i o n 14-5 to approximate t h e answer. )
5. How many d i f f e r e n t ways may t h e l e t t e r s of QUARTUS be
arranged so t h a t t h e l e t t e r u f o l l o w s the letter q ?
6. Hou many d i f f e r e n t arrangements of the l e t t e r s of PALLMALL
may be formed so t h a t all of t h e 1's are -
not t o g e t h e r ?
7. How many d i f f e r e n t ways may the l e t t e r s of QUISQUIS be
arranged so t h a t each q is followed by a u ?
8. How many three l e t t e r arrangements of t h e l e t t e r s of SNOOP
m y be formed? ( ~ l n t : c o n s i d e r cases as to t h e three l e t t e r
word h a v i n g 0 , 1 , o r 2 01s. )
9. How many d i f f e r e n t arrangements of f o u r letters may be made
f r o m the letters of SPOOL ?

P a r t i t l o n a of a Set.
-c-

By a p a r t i t i o n of a s e t A we mean a c o l l e c t i o n of subsets
of A having t h e p r o p e r t i e s
(1) no p a i r of the subsets share any members,
(ii) the union of a l l t h e subsets I s A .
Thus each element of A is in one and only one of t h e subsets.
The subsets themselves are c a l l e d cells of t h e p a r t i t i o n .
Example 1 4 - 6 c : The t w o sets :
(1) t h e s e t of even n a t u r a l numbers,
( 2 ) the s e t of odd n a t u r a l numbers,
form a p a r t i t i o n of t h e s e t of n a t u r a l numbers, Its cells are the
two s e t s l i s t e d . The three s e t s :
(1) the s e t of positive r e a l numbers,
( 2 ) t h e s e t of negative real numbers,
( 3 ) t h e s e t ::Those onLy member I s z e r o ,
form a p a r t i t i o n of t h e set of real numbers.

Example 14-6d: The s e t [ a ,b,c ) has t h e S o l l a d i n g p a r t i t i o n s


(and no o t h e r s w i t h o u t empty cells):

([a1 , [bl , fell; W , b I , &I?; Ua,c) , [bll;

CCc,bl Ia31; (Ca,b,c?l


S i n c e a p a r t i t i o n is a c o l l e c t i o n , or s e t , whose members a r e
themselves sets, a e are obliged to be rather generous w i t h o u r
b r a c k e t s when writing partitions. In the interests o f economy (of
ink) and ease of reading, we i n t r o d u c e a n a l t e r n a t e n o t a t i o n a n d
write, f o r the partitions listed in Example 14-6d:

[a;b;cl , [a,b;cl Y (a,c;bl , [c,b;a] , [a,b,cl ,


respectively. If we need speak o n l y of the c e l l s , w i t h o u t
exhibiting t h e elements i n them, we shall write--as uaual--

f o r the p a r t i t i o n o f A i.ihose cells are A1 , A 2 , . .. , Ak .


e ere A1 , Ag , . .. , Ak are c e r t a i n subsets of A ,)

When we c o n s i d e r k-permutations of the set


we deal w i t h ordered k - t u p l e s such a s

Each of the k-permutations of a given partition of a s e t i n t o k


cells w i l l be c a l l e d an ordered partition of t h e s e t . If in the
ordered p a r t i t i o n (Al , A * , , , , A t h e r e a r e nl elements
I n A1 , n2 elements in A 2 , .. .
, nk elements in Ak , we
s h a l l c a l l t h i s partition a n (nl; n2 ; . ..
, ; nk p a r t i t i o n .
The problem we now put is t h i s : Given a f i n i t e s e t A ,
h a v i n g n elements, how many (nl; n2; . ..
, nk) p a r t i t i o n s
( A , ~ A p , . .. , Ah) of A a r e t h e r e ? Since AI has nl
elements, A2 has n , , .. , Ak has nk . We have, in view
of the d e f i n i n g properties (1) , (ii) of a p a r t i t t o n

We s h a l l see that we have r e a l l y solved t h i s problem a l r e a d y .


All i i e must do t o see t h i s is t o rephrase It appropriately. First,
however, we l o o k a t a n example.

-
Example 14-6e: Some of the (3; 2;2) p a r t i t i o n s of
b,b,c,d,e,f,gl are
fa,b,c;d,e;f,g) , Ca,b,d;c,e;f,gl , [a,b,d;c,f;e,gl .
T h e r e a r e , of c o u r s e , many more. Notice, however, t h a t

are, r e s p e c t i v e l y , simply &her ways of r r r i t i n g the same t h r e e


p a r t i t i o n s as before. Consider, f o r example, the f i r s t in each of
these l i s t s . Using the more e l a b o r a t e notation, these a r e

CCa,b,cl , Id34 , Cf,gll and (la,c,bl , Cd,el , I f , g l l .


But the s e t s [ a , b , c ) and [a,c,b) a r e equal for they have t h e
same elements. S i m i l a r l y f o r t h e other s e t s in each of these
partitions.
T h i s example gives away the secret! The v a r i o u s cells are
unchanged if t h e i r elements a r e rearranged--so far as t h e i r
r e l a t i o n s h i p t o t h e p a r t i t i o n itself is concerned, elements in the
same c e l l a r e " a l i k e " . Thus permuting the elements in a given
c e l l h a s no e f f e c t on the p a r t i t i o n I t s e l f . Hence each ordered
partition

corresponds t o 3! x Z ! x 2! permutations of t h e whole s e t .


Since there are 7! permutations of t h e g i v e n s e t t h e r e a r e

(3;2;2) partitions o f it.


We may e x p l a i n this result another way. We want an ordered
t r i p l e of subsets where t h e r e is no duplication of eleme3ts. We
can "choose" the f i r s t cell (which has 3 elements) in ~ ( 7 , 3 )
ways. Since no p a i r of c e l l s may share any members, we have only
C (4,2) "choices " for t h e second cell (which has 2 elements ) .
F i n a l l y there a r e C (2,2} "choices" for the t h i r d c e l l .
Altogether, such a n o r d e r e d p a r t i t i o n may b e formed i n

In t h e general case, t h e number of (nl;np; . ; nk) ..


partitions of an n-element set (xnere n = nl + n2 + ... + nk) is
Example 14-6f: There a r e t e n entries in an elimination
tennis tournament. H o w many ways may the f i r s t round o f matches
be scheduled?
Solution: We :$ant the number of (2; 2; 2; 2; 2) partitions
of a s e t w i t h 10 elements ( t h e LO entrles) . This number is
lo! or, approxlrnately, 1.1 x lo5 .
(2!)5

Example 14-6g: Given a set h a v i n g n elements, how many


(m;n -
m ) p a r t i t i o n s does it have?
Answer: m! (n n! - m)! . This is ~ ( n , m ) .
T h a t it should be
~ ( n , m ) may be seen if we note that the first c e l l h a s m elements,
and a l l t h e other elements--if any--are in the second. Thus the
(m;n - rn) p a r t i t i o n sw of an n-element set are paired w i t h t h e
subsets o f t h e given s e t .

Exercises 14-6b
1. E i g h t men attend a sales convention and find they a r e to be
in f o u r double rooms. How many ways may they be asslgned to
t h e s e rooms?
2. How many subcommittees of two, t h r e e , and t h r e e members may
be formed from a committee of eight members if each committee
member can be on one and only one subcommittee?
3. In how many ways can 10 indistinguishable blue t l c k e t s a n d
30 Indistinguishable red t i c k e t s be distributed among 4 0
people if each person is t o receive exactly one ticket? (use
log n! table of S e c t i o n 14-4 to approximate the answer.)
4. How many ways are there to arrange eight c o i n s in a row so
there w i l l be three heads and five tails showing?
5. How many s e t s of b r i d g e hands can be dealt to f o u r players
from a f i f t y - t w o card deck? (Use l o g n! table.)
6. (a) In how many ways can 6 people be partitioned into
three teams each consisting of t w o people?
(b) In how many ways can 12 people be partitioned Into
f o u r teams each consisting of three people?
( c ) Generalf ze .

1&-7. -
S e l e c t i o n s w i t h Repetition.
Suppose you a r e in a store having n kinds of items and
more of each k i n d than you can a f f o r d to buy. How many different
selections of m items can you make?
This problem differs In t w o ways from the "arrangement"
questions we have considered, For one t h i n g , we no longer take
account of t h e o r d e r i n which the items are "selected", For
a n o t h e r , we suppose t h a t - - f r o m t h e p o i n t of view of our r e s o u r c e s
--the supply of each kind Is unlimited. The l a s t supposition is
f o r t h e sake of slmpllcity; without it the problem is much more
difficult .
We begin w i t h m = 1 .
The answer here is j u s t n , f o r if
we may select only one item o u r s e l e c t i o n reduces to selecting one
of the n k i n d s . The number o f ways is t h e n ~(n,l).
For rn = 2 , the
question is more interesting. The t w o items
may be a l i k e or they may be different. B u t in e l t h e r case, t h e i r
tl
order" of selection is i r r e l e v a n t .
L e t us p a i r each of the kinds (of which there are n ) ~ i t h
the numbers 1, 2, 3, . , ,, n . Our problem i s then to determine
t h e number of unordered couples of the numbers 1, 2, .. . , n .
We look at the table of t h e ordered couples of p a i r s of elements
of [I, 2, . , n] . .
... ... ...
I
I . . . . a . . a

The couples on the d i a g o n a l a r e t h o s e representing t h e


selection of 2 items of t h e same kind; t h o s e n o t on t h e d i a g o n a l ,
t h e selection o f 2 items o f different k i n d s . B u t each couple
below the d i a g o n a l represents the same s e l e c t i o n a s one above t h e
d i a g o n a l . Suppose we erase all couples below the d i a g o n a l . Then
we have Just one c o u p l e for each of the s e l e c t i o n s we want to
count. T h e l r number is g i v e n by

a n d is t h e r e f o r e ~ ( n + l ,2).

Before we go on t o the c a s e m = 3 , l e t us observe t h a t we


have c o u n t e d the number of ordered couples ( a , b ) of the fomn
1
-< a -( b -< n , i.e., whose first component --
d o e s n o t exceed i t s
second component. This is a n o t h e r way of s a y i n g we c o u n t t h e un-
o r d e r e d couples which may be formed from a s e t of n elements.
F o r m = 3 , we w a n t the number of ordered t r i p l e s ( a , b , c )
with 1 < a < b < c ( n
- - .
As when m = 2 , the c h a i n of inequali-
ties r u l e s out
each of the permutations o f t h e s e trlplea but one.

Csec. 14-71
Suppose we have made one selection, say a , 1 - < a n. We <
have still to make two more, with a 5 b c n <
Selecting .
b , c is equivalent t o selecting t w o numbers from the s e t
[a, a + 1 , ... , n)
which has (n - a ) + 1 members. Thus -- for each s e l e c t i o n , or
1 ( a g n , t h e r e are (" - 2,
+ ways t o select b a n d c
s a t i s f y i n g a < -b < c < n, S i n c e no pair of triples with
different first components can be the same (the first component
being t h e l e a s t component), our second f'undamental I d e a ( s e c t i o n
14-1) tells us the total number of selections is g i v e n by

Using Pascal's Theorem (~xercise14-4, 25)

and the fact t h a t ~(2,721 = ~(3,3), we have

Hence ~ ( n , 3 )= ~ ( n + 1 , 2 )+ ~ ( n , 2 )+ ~(n-1,2)-l- . .. 4- ~(4,2)+ ~(4,3)


= ~(n+1,2)+ ~ ( n , 2 )+ .. . + ~ ( 5 , 3 )

The p a t t e r n emerges:
In general,
s(n,rn) P ~(n+m~l,m) .
The general formula may be obtalned by c a r r y i n g on t h e same llne
of reasoning we have used in t h e 'cases m = 1 , m = 2 , m = 3 .
Example 14-?a: Suppose you have 5 a p p l e s t o give to 3
teachers. How many ways can you do t h i s ?
S o l u t i o n : Here m = 5 and n = 3 , for you are t o select
the teachers receiving t h e 5 items. The answer is given by

-
Example 14-7b: Suppose a millionaire has 50 h e i r s and
legatees. If he c u t s none of t h e m o f f w i t h o u t a c e n t and has just
one m i l l i o n d o l l a r s to bequeath (after taxes and legal fees), how
many d i f f e r e n t w i l l s could he write?

Solution: n = 50 , rn = lo8 ( c e n t s ) . The number of w l l l s I s

therefore ( 100~000904g) , which I s approxfmately 7 . 9 x 10196


50
This r e p r e s e n t s q u i t e a few decisions.

-
Example 14-7c: How many ways may the n a t u r a l number n be
wr3tlen as a sum o f m -non-negative integers, if we distinguish
between sums differing in t h e order of t h e i r terms. (compare
Example 14-4e. )
S o l u t i o n : When we considered t h e problem of representing n
a s a sum of 'rn n a t u r a l numbers we selected ( r ~ i t h o u trepetition)
m - 1 s p a c e s between n tallies arranged in a row. Extending
t h l s i d e a , selecting these s p a c e s w i t h r e p e t i t i o n s w i l l give us
sums w i t h 0 as a term. T h u s , f o p n = 5 , m = 4

the selection ( 1 3 gives 5 = 1 + 0 + 2 + 2 , H o ~ e v e rto

[sec. 14-71
a l l o w f o r t h e f i r s t and l a s t terms b e i n g zero, w e s h o u l d i n t r o d u c e
2 more spaces: one before the f i r s t t a l l y a n d another a f t e r the
last:

Thus, no;+, ( , 3 ) .represents


; = 0 + 0 + 2 + 3

and (,3,6 represents


5 = 0 + 2 + 3 + 0 .
U i t h t h e s e e x t r a s p a c e s , we now have n +
spaces in the
1
general c a s e , of which we a r e t o s e l e c t m - 1 a l l o w i n g
repetitions. The number of such selections is g i v e n by

S(n + 1 , rn - 1) =c[(n+l) + (m-1) - 1, m-11 =c(n+m-1, m-l) .

Exercises 14-7
1. A post office has t e n t y p e s of stamps. How many ways may a
p e r s o n buy t w e l v e stamps?
2. How many ways a r e t h e r e t o s e l e c t f i v e packages o f cheese
from a b i n c o n t a i n i n g t e n k i n d s ?
3, A piggy bank is passed to five people who place in it one
coin each. If the c o i n s are pennies, nickels, dimes,
quarters, h a l f dollars, o r s i l v e r d o l l a r s , how many s e t s of
c o i n s might t h e r e be in the bank, assuming it t o be empty at
the s t a r t ?
4. If the faces of t w o d i c e a r e numbered O , 1 , 3 , 7 , 15 , 31 ,
how many d i f f e r e n t totals can be cast?
5. How many dominoes a r e t h e r e i n a s e t ranging from double
blank to double t w e l v e ?
6. Delete the last eleven words o f Example 14-hf a n d answer t h e
q u e s t i o n t h u s formed.
14-8. Miscellaneous E x e r c i s e s .
How many different arrangements &y be formed from t h e l e t t e r s
of the word MADAM ?
Hod many committees of s e v e n persons may be formed from a s e t
of ten persons?
How many distinct l i n e s are determined by twelve points on a
plane if no t h r e e of t h e p o i n t s are c o l l i n e a r ?
How many diagonals can be drawn in a convex polygon of
n-sides?
How many permutations or the l e t t e r s of COMPLEX w l l l end
in X ?
How many of the 5-permutations of the l e t t e r s A, B, C , D, E,
F, G w i l l have A at the beginning or a t t h e end?
How many d i f f e r e n t ways may exactly three heads show in a
toss of five coins?
How many ways a r e there to seat t e n persons around a t a b l e ,
if a certain p a i r of persons must s i t next to each o t h e r ?
How many four digit numerals may be formed from the s e t
[l , 2 , 3 , .. , , 8 , 93 if no d i g i t s may be repeated and
t h e numbers they represent are odd.
How many tlwords " containing three consonants and t ' d o vowels
may be formed from a s e t of t e n conaonants and the five
vowels?
Haw many f i v e l e t t e r "words" may be formed from a t w e n t y - s i x
l e t t e r alphabet if the f i r s t l e t t e r is not repeated, but
repetitions may occur in t h e o t h e r four places?
How many arrangements of three men and three women may be
made at a round t a b l e if t h e men and women must sit
a1 t e r n a t e l y ?
If a l l p o s s i b l e pairs of numbers, repetitions of digits not
permitted, are s e l e c t e d from the S e t [l , 2 , 3 , 4, 6 ) , in
how many cases w i l l the sum be even?
A bag c o n t a i n s five red b a l l s , four w h l t e balls, and t h r e e
black b a l l s . How many different ways m y t h r e e b a l l s be
d r a m if each b a l l is to be a different color?
How many different h a n d s c o n t a i n i n g t h r e e queens and a p a i r
may be formed from a deck of f i f t y - t w o c a r d s ?
How many d i f f e r e n t s i g n a l s may be formed from t w o red f l a g s
and three b l u e f l a g s i f any f o u r of t h e f l a g s are hoisted on
a flagpole in a vertfcal line and the flags differ only in
color?
H a w many ways may t h r e e boys a n d t h r e e g i r l s s t a n d i n l i n e if
no two boys stsnd n e x t t o e a c h o t h e r and no two girls s t a n d
next t o each o t h e r ?
HON many ways a r e t h e r e to arrange a set o f f i f t e e n different
books by size on a shelf If five of t h e m a r e l a r g e , seven a r e
medium s i z e , and three are small?
Hex many t h r e e digit numerals are t h e r e t h a t do n o t c o n t a i n
the digits 8 o r 0 ?
Hod many ways may nine hooks be clipped onto a steel ring?
How many ways a r e t h e r e to s e a t seven persons in 3 row if t i x o
o f them will n o t sit next t o each o t h e r ?
If a s e t of s i x d i f f e r e n t books is used, ho:d many vlays c o u l d
t h r e e o r more of t h e m be a r r a n g e d on a s h e l f ?
How many ways are t h e r e to form a d i n n e r p a r t y f o r seven
persons from a s e t o f t e n p e r s o n s i f a certain p a i r of t h e
t e n w i l l n o t a t t e n d the ssme d i n n e r p a r t y ?
How many ways may f o u r boys o f unequal h e i g h t s s t a n d in a
line if no boy stands b e t > ~ e e nt w o t a l l e r ones?
Hubv many 5 - p e r m u t a t i o n s of t h e l e t t e r s a , b, c, d , e, f, g do
not cont2in b ?
Hod many xays are there f o r a man to invite one or more o f
h i s s i x f r i e n d s t o h i s home?
F i n d t h e number o f arrangements of t h e l e t t e r s of BOULDER
if no two vowels a r e together.
28. How many three d i g i t numerals representing even numbers
greater t h a n 234 may be formed using the d i g i t s 1 , 3 , 4 ,
5 , 6 , 8 , 9 w i t h no repetitions of digits permitted?
* 2 9 . How many t h r e e digit numerals representing even numbers
g r e a t e r than 234 may be formed using t h e d i g i t s 1 , 3 ,
4 , 5 , 6 , 8 , 9 ?
*3C. H O ~ Jmany t h r e e digit n u m e r a l s representing even numbers
greater than 234 may be formed using t h e d i g i t s 2 , 3 , 4 ,
5 , 6 , 8 , 9 If repetitions of digits are permitted? If
repetitions of d i g i t s are n o t permitted?
31. Suppose n tickets, numbered s e r i a l l y , are p r i n t e d for a
r a f f l e . Suppose t h e y are a l l sold and each purchaser
counterfeits (m - 1 ) copies of h i s stub and sneaks them
I n t o 3 b 0 ~ 1( s o t h a t each o f the n numbers appears on m
tickets i n the bowl). Two prizes a r e to be a d a r d e d and hence
two stubs must be drawn,
(a) How many ways is it possible t o d r a ~t w o s t u b s ?
( b ) Ho-~imany of t h e s e ways r e s u l t in both n u m b e r s being t h e
same?
( c > The ratio o f t h e ansiter in ( b ) to that in (a) l n d l c a t e s
t h e chances o f exposing one of t h e c o u n t e r f e i t e r s .
Compute t h i s r a t i o for each p a i r ( n , m ) , ; i t h n,m in
t h e ranges 1 ( n ( 5 , 1 ( m ( 5 , m,n in N.
(d) What conclusions d o you draw concerning the risk of
being caught if
( i ) n increases f o r f i x e d m ,
(ii) m i n c r e a s e s f o r f i x e d n ?
Chapter 1 5
ALGEBRAIC STRUCTURES

15-1. Introduction.
b r i n g o u r c o u r s e of study of t h i s book, we have met several
number systems: the systems of the n a t u r a l numbers, the integers,
t h e r a t l o n a l numbers, the real numbers and t h e complex numbers.
In each of these systems we saw t h a t our concern was w i t h the
following:
(1) Obiects or elements, here numbers;
(2) - Two o p e r a t l o n s , addition and m u l t i p l i c a t i o n ;
-
( 3 ) Laws s a t i s f i e d by these operations, such as the
commutative and associative laws of addition and
multiplfcation and the d i s t r i b u t i v e law.
If ne stop and reflect for a moment, we see t h a t many of the
a l g e b r a i c computations which we carried o u t were independent of
the nature of t h e numbers with which we were operating and
depended solely on the fact that the operations in question were
s u b j e c t to laws respected in each system. Thus, f o r example, if
we c o n s i d e r t h e i d e n t i t y

and t h i n k of t h i s assertion as applying to a and b taken a a


(1) i n t e g e r s ,
(2 rational numbers,
( 3 ) real numbers,
( 4 ) complex numbers,
we see that, lf we e s t a b l i s h e d t h e I d e n t i t y 15-la at the earliest
atage f o r integers and observed
(1) that the verification depended o n l y on the distributive
l a w , t h e aasociatlve laws and commutative laws and
properties of the a d d i t i v e inverse, and
( 2 that each of t h e laws and properties invoked were in
force f o r t h e complex number system,
t h e n it would be unnecessary to repeat the verification for the
case where a and b are complex numbers.

Without such laws a l g e b r a i c computation a s w e know it would


cease to exist. The whole source of a l g e b r a i c computation is to
be found in these laws.
We can, if we l i k e , seek to abstract what i s algebraically
essential and coman t o several s p e c i f i c number systems and
develop algebraic results which hold f o r each of these systems
w i t h o u t having to repeat our work i n each s p e c i a l case. T h i s
approach is of great importance in many parts of modern rnathe-
matics, especially in modern hlgher algebra which is sometimes
also c a l l e d a b s t r a c t algebra.
What is the nature of the fundamental algebraic operatfons
which we have met? Let us take the addition of real numbers. We
are given r e a l numbers, say a and b , in order, or, if we
like, t h e o r d e r e d p a i r ( a , b ). The o p e r a t i o n of addition assigns
to t h e ordered p a i r ( a , b ) a unique real number which we desig-
nate a + b . The words "assigns" and "unlque" give t h e secret
away. The operation of addition (of real numbers) is a f u n c t i o n
d e f i n e d f o r each ordered p a i r of real numbers which assigns t o
each s u c h ordered palr (a,b) of real numbers a real number, the
sum a + b . It should be observed that w h i l e most of the
f u n c t i o n s which we have met assigned real numbers t o real numbers,
the f u n c t i o n concept is an extremely general one and we may
c e r t a i n l y c o n s i d e r a f u n c t i o n f which assigns to each element
a of a glven c l a s s A a unique element ( l a b e l l e d f ( a ) ) of a
given class B . In t h e example of addition of reals, the class A
is the set of ordered pairs of real numbers and t h e class B is
tine s e t of real numbers i t s e l f . There is a p o i n t concerning
notation that should be made. Instead of w r i t i n g the real number
a s s o c i a t e d w i t h t h e ordered pair ( a , b ) In f u n c t i o n n o t a t i o n , say
S[(a,b)j , where S ( s t a n d i n g for "sum") is the function j u a t
described, we use the usual nofation and write a + k .

15-2. I n t e r n a l Operation.
Let us t r y to abstract what is algebraicaLly essential in t h e
example of a d d i t i o n of real numbers. Suppose t h a t A is an
a r b i t r a r y non-empty set of elements, the n a t u r e of which need not
concern us. Suppose f u r t h e r t h a t there i s given a f u n c t l o n which
is d e f l n e d for the ordered pairs (a,b) , where a f A and b E A ,
whlch asaigns to each such ordered p a i r a member of A Such a .
f u n c t i o n is called an i n t e r n a l operation in A . (It is c a l l e d
11
i n t e r n a l " because t h e cumponents a and b of the i n p u t (a,b)
a r e drawn from A and t h e o u t p u t assigned by the function is also
a member of A . Hence, the operation i n question does not involve
data taken o u t s i d e of A .)

There is also a n o t i o n of an external operation and, i n d e e d ,


an example is t o be found in t h e algebra of vectors when one
considers real m u l t i p l e s o r a given v e c t o r so that input Is an
ordered p a i r of the form (real number, vector) and o u t p u t is a
v e c t o r . Here we go ~ u t s l d ethe domain of vectors t o s p e c i f y the
-
i n p u t -hem e 'externa1 'I.
However, in this c h a p t e r we s h a l l consider only i n t e r n a l
operations and for that reason we s h a l l henceforth simply say
II
o p e r a t i o n I' rather than "internal o p e r a t i o n 'I, As it is customary,
we shall usually denote an operation by a m u l t i p l i c a t i o n s i g n .
and the element assigned t o t h e ordered p a i r (a,b] by a b .
when we a r e concerned w i t h a single operation. Ne s h a l l also wr1:e
f o r "a b" dhen there is no doubt about t h e meaning. We
s h a l l have occasion later to d e a l w i t h two o p e r a t i o n s and then we
shall usually use + and to denote the two operations.
If we are concerned w i t h finite s e t s A , we may specify w i t h
the a i d of a multiplication table how a given o p e r a t i o n a c t s in
the same way that we l i s t e d t h e sum and product of certain im-
portant pairs of natural numbers w i t h t h e aid of addition and
m u l t i p l L c a t i o n t a b l e s in elementary arithmetic. The procedure is
to use a square table marking rows by t h e elements of t h e s e t A
and columns by t h e elements of the s e t A . The row markings are
indicated at the left of t h e body of the t a b l e and the column
markings a r e indicated above the body of t h e table. Given
a,b A , in the space In the body of the table belonging t o t h e
,
row marked "at' and the column marked "b" we record the element
a s s o c i a t e d w i t h (a,b) by t h e operation .
Here is a simple example: Let A = {0,1) and l e t denote -
conventional multiplication in the r e a l number system. Then the
operation . may be tabulated as follows:

Suppose t h a t we c o n s i d e r a s e t A consisting of two distinct


elements a and b and we ask in how many ways can we specify an
operatlon in A .
This amounts to constructing i n all p o s s i b l e
ways two-by-two square tables in each space of which is recorded
an element of A .
Here are some:

A,a
a
h
a
a
a
a ,
al;
a

b b
;
h ,

There are 16 such o p e r a t i o n s In


a 4a , aa

A
b

.
a
a

b ,
a<ia a

b
a

b
;
a .
Exercises 15-2
1. List the remaining 12 o p e r a t i o n s in A .
2. Let A = 1 i, 1 - 1 and let .
denote c o n v e n t i o n a l
multiplication for complex numbers. Show t h a t is an .
o p e r a t i o n in A and c o n s t r u c t the table for , .

It is of interest t o n o t e t h a t , i f 2 A is a flnlte set con-


.ining n elements, then there are nn d i s t i n c t operDationsin
. o r n = 2 , we have z4 = 16 distinct operations in A ;
f a r n = 3 , we have 39 = 19,683 distinct o p e r a t i o n s in A ) .
W e s h a l l be interested in studying the composite o b j e c t
consisting of a non-empty s e t A and one o r two o p e r a t i o n s i n A .
Precisely, the term "composite o b j e c t ' is to be taken h e r e t o mean
either an ordered p a i r of the form ( A , - ) where is an
o p e r a t i o n in A o r an ordered t r i p l e of the form ( A , + , )
where + and . are o p e r a t i o n s in A . Such a composite o b j e c t
is c a l l e d an a l g e b r a i c s t r u c t u r e with one operation (or two
operations respectively). A n example of a s t r u c t u r e with one
o p e r a t i o n is given by t a k i n g A as the s e t of integers and as .
t h e customary a d d i t i o n . An example of a s t r u c t u r e w i t h t w o
operations is given by t a k i n g A as the s e t of real numbers and
+ and . respectively a s the customary addition and multlplicat1on
f o r the reals. Another example of a s t r u c t u r e with two o p e r a t i o n s
I s g i v e n by taking A as the s e t of r e a l numbers, as the
customary multiplication for the reals, and + a s the customary
a d d i t i o n f o r t h e reals.
Now it t u r n s out t h a t the i n t e r e s t i n g s t r u c t u r e s are t h o s e
which are s u b j e c t to varlous laws. We saw that the number
systems which we s t u d i e d earlier were structures w i t h t w o opera-
t i o n s which respected such laws a s the commutative laws, the
associative laws, and the distributive la^. If we wished to take
I n t o a c c o u n t s t r u c t u r e a which are n o t mbject to any restrictions
or laws, we would be faced w i t h many different k i n d s of structures
having very few properties in common. We cannot hope to find
interestfng results which would be v a l i d f o r all structurea w l t h
a g i v e n s e t A and w i t h a given number of operations.

On occasion, instead of referring t o the structure "(A, )"


or " ( A , + , ) I' we s h a l l use t h e less fomnal "A together w i t h
the o p e r a t i o n " or "A together w i t h the operations + and
" , etc.
11
respectively, as w e l l as "A and the operatlon
We s h a l l concentrate on two important s t r u c t u r e s which
permeate elementary algebra--the g r o u p and the field. Our -
interest w i l l center principally on the n o t i o n of a field which
embraces three of t h e important number systems which we have met
so far--the systems of the rationals, the reals, and the complex
numbers.

15-3. Group.
Suppose t h a t we consider a structure w l t h one o p e r a t i o n
(A, .) , The example which we cited above, where A I s the s e t
of integers and - is the customary a d d i t i o n , has the following
two properties:
(1) The a s s o c l a t l v e law for a d d i t i o n is satisfied.
( 2 ) Given integers a, b, there exists a unique integer x
satisfying a + x = b and there exists an integer y
satisfying y + a = b .
(We ignore d e l i b e r a t e l y the question of the e q u a l i t y of x
and y for a reason which w i l l become clear presently.) If we
ask f o r s t r u c t u r e s w i t h one o p e r a t l o n whlch share t h e s e l i s t e d
properties w i t h t h i s speclal s t r u c t u r e , we are led t o the very

Eaec. 15-31
i m p o r t a n t s t r u c t u r e s w i t h one operatfan c a l l e d groups. They
appear throughout mathematics in many different g u i s e s , The s t u d y
of groups as such is an i n s t a n c e of algebra -- a t I t s pupeat.
S p ' e c i f i c a l l y (A, -
) is said to be a group provided t h a t t h e
following two conditions are satisfied:

G 1. The 09eration Is assoclatlve. That is, g i v e n


elements a , b, c in A , we have

G 2. Given elements a, b in A , each of t h e equations

and

has a u n i q u e solution I n A .
It is tn be observed t h a t M e have not r e q u i r e d t h a t the
operation -
be commutative. In f a c t , we s h a l l meet examples
where
a .b
.
s b a
-
does not satisfy the commutative law which a s s e r t s t h a t
for all a, b € A .
This is why it was important
in d e f i n i n g t h e n o t i o n of operation to have as our input an
ordered pair of elements of A .
The order in which the components
are assigned may very well be e s s e n t i a l . If the o p e r a t i o n .
s a t i s f i e s the commutative law, the group is c a l l e d commutative or,
as is more u s u a l , a b e l i a n , I n honor of the g r e a t Norwegian
mathematician N. H, Abel (1802-1829)who did pioneer work in the
t h e o r y of groups.
Let us consider some examples of groups drawn from o u r
e a r i i e r experience. In these examples the operations a r e t h e
standard ones o f the number systems so t h a t the groups In q u e s t i o n
are necessarily a b e l i a n . We s h a l l c o n s i d e r an example of a non-
abelian group later (Section 15-5).
mample 15-3a. A = s e t of integers; the operation - is the
conventional a d d i t i o n + .
The second
postulate states that t h e equation a 4- x = b,
where a and b are i n t e g e r s , has a unique
integral solut Ion.
-
Example 15-3b. A = set of real numbers d i f f e r e n t from zero;
is t h e conventional m u l t f p l i c a t i o n .
&ample 15-3c. A = s e t of v e c t o r s in 3-space; . i s t h e
addition @ .

Verify that each of the c i t e d examples s a t i s f i e s the group


postulates G 1 , G 2 . Show that t h e f o l l o w i n g are a l s o
examples of groups:
Fxample 15-3d. A is the set of nth roots of 1, where n I s
a p o s i t i v e integer, and is the conventional
m u l t i p l i c a t i o n f o r complex numbers. Here it is
to b e observed that A h a s j u s t n elements.
-
Example 15-3e. A is t h e set of p o s i t i v e rational numbers;
is the conventional multiplication.
2. In what way doea the f o l l o ~ i n gf a i l to yield an example of a
group: A = set of a l l complex numbers and is the con-
ventlonal multiglicatlon?
3. Let A denote the s e t of real numbers of the form
a + b f i where a and b are integers and let be the
conventional addition. Verify t h a t is an o p e r a t i o n in A
and that the group postulates a r e satisfied,
4. L e t A denote t h e s e t of real numbers d i f f e r e n t from z e r o o f
the form a + b f l where a and b are r a t i o n a l and l e t -
be the c o n v e n t i o n a l r n u l t l p l i c a t i o n . Verify that is an
operation in A and t h a t the group p o s t u l a t e s a r e satisfied,
-
15-4. -.
Some General Properties -
of Gpoups.
We have seen in o u r earlier work w i t h number systems t h a t an
important role was played by the notions of additive identity,
a d d i t i v e i n v e r s e , multiplicative i d e n t i t y , m u l t i p l i c a t i v e i n v e r s e .
The c o u n t e r p a r t s of these notions appear in general group t h e o r y as
we s h a l l now s e e . We must not forget t h a t t h e commutative law
-
need n o t be in effect f o r an arbitrary group!
I d e n t i t y element, Here we a r e concerned w i t h t h e q u e s t i o n
whether t h e r e is an element e in A which has t h e p r o p e r t y Chat
a e = e a = a for all. elements a E A In each of the.
c i t e d examples of S e c t i o n 15-3 t h e r e is precfsely one elemenk w i t h
t h i s p r o p e r t y , Thus in Example 1, the integer 0 is t h e unLque
element having the s t a t e d property; In Example 2, it is 1 ; in
Example 3, ft I s the zero v e c t o r ( 0 , 0 , 0 ) ; i n Example 4 , I t is 1;
in Example 5 , It Is 1. We now t u r n t o t h e s i t u a t i o n f o r an a r b i t r a r y
group and a proof o f t h e following theorem:
Theorem 15-4a: Given the group consisting o r t h e s e t A
and operatLon .
, t h e r e is a unlque element e of A
whfch s a t i s f i e s the following condition:

for all a f A ,

(The element e is c a l l e d t h e identity element of the g r o u p .


Note how t h l s is in agreement with earller usage. )
- Proof : We f l x an element b f A .
That
---
there I s a t most one element e having t h e stated p r o p e r t y
f o l l o w s from t h e f a c t t h a t e is a s o l u t i o n o f t h e equation
b . x = b which has p r e c i s e l y one s o l u t i o n .
Naw l e t e -
denote the s o l u t i o n of b x = b and l e t us
verify t h a t a .
e = a f o r a l l a in A .
Given a E A , let c
satisfy c . b = a .
That is, c is t h e unique s o l u t i o n of
y b = a . Our reason f o r Introducihg c is that, if xe w r i t e
a as c * b ,
we are in a p o s i t i o n to r e l a t e the product ae
(which we should like to show is equal t o a ) to the p r o d u c t
[ s e c . 15-41
b e about which we have information. Specifically,

The proof of the theorem w i l l be complete when we show that


we also have e .
a = a for all a in A .
Given a € A , let
d denote t h e unique s o l u t i o n of the equation y a = a .
In .
order t o r e l a t e d and e , we i n t r o d u c e f t h e u n i q u e solution
of the equation a .
x = e (thereby linking theaementa a and
e) . From d .
a = a and a .
f = e , we have

From the a s s o c i a t i v e law and af = e , we have

Taken together these e q u a l i t i e s yield

Now e s a t i s f i e s the equation y e = e .


( ~ e c a l lt h a t
ae = a for a l l a in A , in particular f o r a = e This .
yields e e = e . ) Since e and d both satisfy the
equation y .
e = e and s i n c e t h i s equation has a unique s o l u t i o n ,
e = d . Hence on taking account of t h e relatlon d a = a , we
have e .a = a . The proof of the theorem is now complete,

The n o t a t i o n e w i l l be reserved for the identity element.

Inverse element. Qiven af A , let us consider the two


equat Ions
a . x = e and y . a = e .
Since we do n o t have the commutative law at o u r disposal, it
is n o t o b v i o u s t h a t t h e s o l u t i o n s x and y of these respective
equations a r e e q u a l . Let u s see whether, i n spite of the non-
a v a i l a b i l i t y of the commutative law, x = y . Let us m u l t i p l y
each s i d e of a x = e on t h e left by y . We o b t a i n

Using t h e associative l a w and the basic p r o p e r t y of t h e identity,


we obtain
( y m a ) . x = y .

Hence

Since

&e c o n c l u d e t h a t x = y . The common solution of a x = e


and y .a = e is called simply the inverse of a . It is
denoted a-' .

Exercises 15-4
1. Determine t h e i n v e r s e element of an a r b i t r a r y element for
each of t h e groups examined in S e c t i o n 15-3, The answer
1 s ta be s t a t e d i n terms of the special interpretation o f a
group given by the example. Thus in Example 15-3a, the answer
is "the i n v e r s e of a is - a " .
2. Show that a" - b Is the s o l u t i o n of a - x = b and that
b .a" is the solution o f y . a = b .
3 . Which of t h e multiplication tables considered in S e c t i o n 1 5 - 2
s a t i s f y t h e g r o u p requirements? In case of f a i l u r e , s t a t e
t h e reason. In the case(s) ,)here a g r o u p is specified,
e x h i b i t the i d e n t i t y element and t h e i n v e r s e of each element.
4. Let A d e n o t e a non-empty s e t , and an operation in A .
S ~ O JtIh a t t h e r e is at; most one element e < A such t h a t
a . e = e . a = a forall a f A .
5. Let A denote a non-empty s e t , and an operation in A .
Suppose t h a t . s a t i s f i e s the associative law. Suppose that
t h e r e exists an element e A such t h a t a e = e a = a
for all a A . h he element e is unique by Exercise 4.)
Suppose t h a t f o r each a A , there e x i s t s x A such that
a . x = e and that there e x i s t s y A such t h a t y a = e .
Show t h a t A together w i t h is a group. H i n t : With x
satisfying a x = e and y satisfying y a = e , show
that a . z = b is satisfied by x . b , and, by multiplying
e a c h side by y , t h a t the only possible s o l u t i o n is y b .
Hence conclude t h a t there Is precisely one s o l u t i o n . Treat
t h e remaining case s i m i l a r l y .
6. C o n s t r u c t multiplication tables f o r o p e r a t i o n s in a s e t A
of three elements so that the group postulates G 1 and G 2
are s a t i s f i e d . Hint: We may assume that one of t h e elements
is e , t h e i d e n t i t y , and we may call one of the remaining
elements a and the o t h e r b . The c o n s t r u c t i o n of a
multipllcation table can be carried out in only one way when
account is taken of the n a t u r e of the identity element and
the group postulates.

15-5. An ---
Example of a Non-Abelian Group.
It is n o t hard to give an example of a group which is n o t
a b e l i a n by means of a specifically constructed m u l t i p l i c a t i o n
t a b l e . H,wever, there is greater interest i n constructing an
example which is meaningful in terms of o u r earlier experience and
which at the same time is important in terms of o u r future study
of mathematics. The elements which we consider are the non-
constant l i n e a r functions; t h a t is, the functions 2 deflned f o r
all real numbers by the formulas of the form

where a and B are real numbers and a # 0 .


Our s e t A I s
taken t o be the s e t whose elements are the functions / .
It should be observed that a given linear function I s
defined by precisely one formula of the form 15-ga. That is, If

for a l l real x , then a = and ~7 = d . This fs seen by f i r s t


setting x = 0 and i n f e r r i n g that& =oT and then that a: = 71
Composition. Suppose t h a t we are given non-constant linear
functions J and m where / ( x ) = ct x + @ and m(x) =r" x +d .
It is often of interest to construct a function from the given
functfons 1 and m in the following manner. S t a r t i n g w i t h
input x our f i r s t f b n c t i o n -/ yields o u t p u t /(x) .Suppose
that we now use/ (x) a s input with the f u n c t l o n m .The o u t p u t
is m ( y ( x ) ). We see t h a t for each r e a l x the quantity
m(/(x)) is unambiguously specified. Thus we have a f u n c t l o n
determined by the requirement t h a t to each real x there I s
assigned m ( / ( x ) ) . This f u n c t i o n is called tho composition of -
m and/. It is denoted by m ,/. Let u s determine
rn[R(x))explicitly. We have

This computation shows t h a t the function m o / is a non-


constant linear function, for the coefficient of x in the l a s t
l i n e of Formula 15-5b is n o t zero. The rule which assigns to the
ordered p a i r (m,/) of non-constant l i n e a r functions the composi-
tion f u n c t l o n m / I s an operation in A . By analogy with
what we d i d w i t h sum a n d p d u c t , we denote the o p e r a t i o n of
cornpositlon by . Let us pause to consfder a numerical example
before we c o n t i n u e o u r study of t h e structure we have just i n t r o -
duced.
Thus, suppose
y ( x >= a +1 and m(x) = -a+ 3 .
We have f c r rn :
/(m(x)) = a(x) + 1 = 2(-a + 3) + 1 = -4x + 7 .
We have f o r m 1:

This example shows that w i t h the s p e c i f i c choices made f o r


1and rn , re have
lorn + m e R.
We recall that two functions which have the same i n p u t s e t s (i.e.,
domain) are d i f f e r e n t If they a s s i g n d i f f e r e n t o u t p u t s for some
member of their c o m n input s e t . 1n our example ,/ m and
-
m o 1 assign different o u t p u t s f o r each r e a l x .
Hence t h e y a r e
d i s t i n c t functions.
This example shows us t h a t the commutative law does - n o t hold
f o r the o p e r a t i o n of composition of ( n o n - c o n s t a n t ) l i n e a r
functions.
How d o we show that the s t r u c t u r e consisting of the non-
c o n s t a n t linear functions together w i t h the operation o f composi-
tion is a group? We simply v e r i f y that Q 1 and G 2 a r e
f u l f i l l e d w i t h t h e o p e r a t i o n of composition.

1
Q .
Suppose that 1,rn, and n a r e three given (nan-
constant) linear functions, Given x as i n p u t , ,
/ (men>
the 1o u a for Input m e n(x) , 1 . e . . the
assigns as output -
o u t s u t for i n n u t
m(n(x)) . Given x as i n p u t (,/*rn)o n
assigns as o u t p u t -
the mR8 o u t p u t fop input n(x) , that is,

Ro m(n(x))
- b r input rn(n(x)) . Hence f o r
But Y o m ( n ( x ) ) is t h e / o u t p u t f-
each r e a l x as I n p u t , 1 (m n) and ( y o
0 m) O n assign the
same o u t p u t . Hence the functions 1 ( m a n) and (lo m) a n a r e
equal. The associative law G 1 is v e r i f i e d for composit~on.
G 2. Given two members of A ,/ and m , we ask:
Is there a member n s a t i s f y i n g

is there just one s u c h member? Let us t r y to approach the


q u e s t i o n in an exploratory way. Let

Suppose that
n(x) = h x +p ( h f 0)
satlsfies 15-5c. From 1 5 5 b we have

Hence if R o n = m , we
have, u s i n g t h e fact t h a t a l i n e a r
f u n c t l o n may be represented by only one formula of the form 1 5 - 3 ,

Hence

We conclude t h a t there is a t most one such member n On.


the other hand, if we take and p as g i v e n by 15-5d, the
function n defined by

does satisfy 15-5c. Hence 15-5c has a u n i q u e s o l u t i o n .


The treatment of the o t h e r equation, no,/ = rn , where /
and m a r e g i v e n members of A , is similar. Thus we see that
----
the set of non-constant linear f'unctlons t o g e t h e r w i t h the --
-
operation of composftion - is-a-non-abelian & r o u ~ .
1. F'urnish the d e t a i l s concerning the equation n. / = m ,
where R and rn a r e given members of A .
2 Determine the i d e n t i t y element of t h e group which we have
studied in t h i s s e c t i o n .
3. Determine t h e i n v e r s e of1 R if (x) = a x +/3 , (1 # 0 .
4. Show by dlrec t computation t h a t n = 1-1* m satisfies
1,n = rn and that n = m e satisfies no/= rn where
R ( x ) = a x + p a n d m(x)=f x + d , u # 0 , f f 0 .
. Show that l o m = m. 1 f o r the functions of Exerclse 4 i f
and only if (a - I)& = ( f - 1)B .
6. Let A denote the set of ordered p a i r s of real numbers w i t h
non-zero first components. Given (a,b) , ( c , d ) in A ,
--
let ( a , b ) .(c,d) be defined as ( a ~ ,a d + b) .
Show that
A , . is a group. What is t h e identity element? What is
the i n v e r s e of the element ( a , b ) of A ? Is there any
r e l a t i o n between t h i s group and the group of non-constant
l i n e a r functions t r e a t e d in t h i s section? Hint: Use No. 5
of Zxercises 15-4.
7. Suppose that A I s the s e t of ordered pairs of r a t i o n a l
numbers with non-zero f i r s t components and that .I s
d e f i n e d as in Exercise 6 . Show t h a t ( A , - ) Is a group.
Show that a corresponding result holds when A is t h e set
of ordered pairs of complex numbers with non-zero f i r s t
components and again is defined as in Exercise 6 .

-
15-6. -
Field.
We now turn to t h e consideration of an algebraic s t r u c t u r e
which is present I n v e r y many areas of mathematical study. We
refer to the n o t i o n of a f i e l d . Once the d e f i n i t i o n of a field
Is s t a t e d , it will be clear that each of the following number
systems is a field:

Esec. 15-61
(a) The rationals with the usual addltion and m u l t i p l i c a t i o n .
(b) The reals w i t h the usual a d d i t i o n and multiplication.
( c ) The complex numbers w i t h the u s u a l a d d i t i o n and
multiplication.
Let denote a s e t c o n t a i n i n g more than one member. L e t +
A ---
and a denote two operations in A .
Then ( A , +, ) is called a

a field provided that t h e f o l l o w i n g postulates are satisfled:


F 1. The s t r u c t u r e (A, +) is a n abellan group.
(The identity element of t h i s group is called "zero",
and is denoted by " 0 " i n accordance w i t h t h e usage
employed f o r t h e number systems which we have s t u d i e d
earlier; the i n v e r s e o f the element a is denoted by
-a , and the s o l u t i o n of a + x = b by b a). -
P 2. L e t B denote t h e s e t obtafned from A by the
removal of t h e element 0 .
It I s required
( 1 ) that .
be an o p e r a t i o n in B--i.e., if
bl , b 2 E B , then bl .
b 2 E B ; and

(2) that the s t r u c t u r e ) be an abelian g r o u p .


(B,
h he identity element of this group is called
"one1'and is denoted by "1" .
khen we speak of
. as an operation in B , we a c t u a l l y refer,
not to the full operation .
I n A , b u t rather
to the f u n c t l o n obtained from .
by restricting
attention to inputs of the form ( b l , b 2 ) where
bl and bp are members of B .)
P 3. The two distributive laws

hold, a , b, and c being a r b i t r a r y elements of A .


Some remarks are i n o r d e r .
Given a f i e l d ( A , +, . ) , it is sometimes c o n v e n i e n t in
order to avoid unnecessarily clumsy modes of expression to u s e the
phrase "the field A " and to mean elther
(1) the s e t A or ,
( 2 ) the f i e l d in the strict sense ( A , +, ) .
Which meaning is intended will be clear from context. When we
speak of t h e elements of the field, we mean of course the elements
of A .
We s h a l l also agree to write, as is usual, "ab" for
'la b 'I .
Of course, it is p o s s i b l e to state the required p o s t u l a t e s
in alternative form and in d e t a i l . The group concept, however,
permits us to separate off in i n d i v i d u a l compartments a description
of t h e a c t i o n of each of the given operations + and . . It is
now c l e a r t h a t if the two operations are to be i n t e r r e l a t e d in a
s e r i o u s s o r t of way, some c o n d i t i o n p e r t a i n i n g to both + and
must be in effect, In the postulates which we have l i s t e d , it is
F 3 which links + and - , In particular, it is n a t u r a l to
t u r n to F 3 to see how 0 a c t s in multiplication.

W e have
O + O = O ,

and hence if a is an apbitrary element of A ,

and

Applying the distributive laws, we obtain

and

relations which s t a t e t h a t $0 and Oa are each the zero of A ;


P o s t u l a t e F 2 pertains only t o B . Are t h e c o m u t a t i v e
a n d associative laws in e f f e c t f o r .
in A ? The only c a s e t h a t
need concern us Is when one of the given elements is z e r o , but
then see that the two laws are in effect, for each s i d e is
z e r o if one of the g i v e n elements is.
Since 1 0 = 0 and a = a , a # 0 , we see t h a t 1
1
is a n i d e n t i t y element for in A. .
The element 1 i s the
only element in A with this p r o p e r t y . If e €A satisfies
a -e = a for all a A , ire have

and

Hence

Consider e q u a t i o n a . x = b . If a = 0 and b # 0 , t h e n
t h e r e is no solution. If a = 0 and b = 0 , then every element
of A is a s o l u t i o n . Suppose t h a t a # 0 . Here we see, using
the same argument t h a t we used in t h e study of a group, that if
a 0 , the equation has the unique s o l u t i o n a-I .
b .Again,
f o l l o r r i n g o u r earlier p r a c t i c e f o r number systems, we s h a l l d e n o t e
t h e solution of a . x = b , a f a , by ab .
bie nou see t h a t the identities and theorems xhich were ob-
tained for t h e r a t i o n a l number system, the r e a l number system, or
the complex number system, and whose p r o o f s depended only on the
s t r u c t u r a l laws which h o l d f o r an arbitrary f i e l d , c o n t i n u e to h o l d
for a n a r b i t r a r y f i e l d . Thus, i f a, b, c , d are members of an
a r b i t r a r y field and b # 0 and d f O , then

[sec. 15-63
Exercises 13-6
1. Verify that E q u a t i o n 15-6a h o l d s f o r an arbitrary field.
2. Given that a, b, c , d are elements o f a field znd t h a t
b # o , c f o , d f o .
s~o;, t h a t (a/b)/c = a/bc and t h a t
(a/b ) / ( c / d ) = ad/bc .
3. Show that if a, b, c, d , e , f a r e arbitrary elements of a
f i e l d and ae - bd f 0 , then the system of e q u a t i o n s

has a unique s o l u t i o n ( x , y ) ;ihose components a r e elements


of t h e f i e l d . G i v e explicit f o m u l a s f o r the solution.
4. L e t A c o n s i s t of the numbers 0, 1, 2 . Let an o p e r a t i o n
+ be defined i n A by t h e requirement t h a t if a, b f A ,
then a + b is to be the remainder obtained when the number
a + b (+ b e i n g the c o n v e n t i o n a l addition) i s d i v i d e d by 3 .
Thus if a = 2 and b = 2 , then a + b is the remainder
o b t a i n e d when 4 = 2 + 2 is d i v i d e d by 3 ; i . e . , 1 .
Similarly, let an operation . be d e f i n e d in A by the
requirement t h a t , if a, b A , then a b is t o be the
remainder when t h e number ab (reference being made t o
conventional multiplication) i s d i v i d e d by 3 . Display the
tables for + and . . Verify t h a t the s t r u c t u r e ( A , +, . )
is a field. This exercise yields an example of a field which
has precisely 3 elements.
5. Let A consist of two distinct elements a, b .
Let + and
be t h e o p e r a t i o n s in A given by t h e folloiving tables.
Show t h a t the s t r u c t u r e (A, +, ) is a f i e l d .
Specify the additive identity and t h e multigllcative i d e n t i t y
of t h i s f i e l d .

15-7. S u b f i e l d .
Given a ffeld whose elements constitute a s e t . A , It is
n a t u r a l t o consider s u b s e t s B of A which taken together with
+ and . make up a field; t h a t is, subsets B which have the
f o l l o w i n g two properties:
(1) When + and . a r e restricted to ordered pairs ( b l , b 2 ) ,
whose components a r e in B , they define operations in B .
( 2 ) B together with + and so r e s t r i c t e d is a field.
Such a subset B of A is c a l l e d a s u b f i e l d of A .
Of course,
one can also call such a B taken together w i t h i t s two operations
a s u b f i e l d of t h e given f l e l d . The meaning which is intended will
be c l e a r from context.
Wlth this notion we can proceed to find o u t something about
the architecture of the complex number system. Let Q denote
t h e s e t of rational numbers; let R denote the s e t of real
numbers, and l e t C denote the set; of complex numbers. We k n o w
t h a t Q is a subset of R and that R i s a subset of C ; in
t h e n o t a t i o n of the theory of s e t s ,

BCRCC .
We may ask whether there are any intermediate subfields between
R and C or between Q and R , and whether t h e r e is any sub-
field of the complex number system which is a proper p a r t of Q .
Suppose t h a t A is a s u b f i e l d of the complex number system
which c o n t a i n s R . Suppose t h a t A contains an element n o t
already in R . Then such an element m u s t be of t h e form a + bl
xhere a and b a r e real and b f 0 . Since a A ,
( a + bi) - a = b i c A . .
S i n c e b c ,~ ~ E A Hence given
a r b i t r a r y r e a l numbers c and d , we have d l € A and t h e r e f o r e
.
c t- d i C ~ T h a t is, CCA. Hence A = C .
We a r e l e d t o the
f o l l o w i n g conclusion:

-
Theorem 15-7a: If A is a s u b f i e l d of the complex number
system c o n t a i n i n g R , then e i t h e r A = R or A = C .
This theorem s t a t e s t h a t there is no s u b f i e l d o f the complex
number system bihich c o n t a i n s R as a proper subset and at the
same time is a proper subset of C .
A second result t h a t is easy to obtain Is t h e f o l l o w i n g :

-
Theorem 15-7b: E v e r y s u b f i e l d of the complex number system
contains Q ,

Proof: L e t A denote a subfleld of t h e complex number


system. W e n o t e that if a and b belong t o A a n d b # O ,
then a f A
6
. Now 1 A , It is a consequence of t h e additive
c l o s u r e o f A and the brell o r d e r property of t h e n a t u r a l number
system t h a t e v e r y n a t u r a l number is a member of A , Suppose t h a t
there a r e one or more n a t u r a l numbers n o t in A and l e t m be
the minimal member o f t h e s e t o f n a t u r a l numbers n o t in A (the
w e l l order property assures us t h e r e is s u c h a minimal member).
Then rn - 1 is a member of A , but our hypothesis tells us rn
is n o t . Since m = (rn - 1) + 1 and m -1 and 1 are in A ,
it follows from the a d d i t i v e closure of A that m itself i s in
A . This contradiction proves t h a t the s e t of n a t u r a l numbers
n o t in A is empty. It now f o l l o w s that every integer i s a
member of A , since f o r each n a t u r a l number n , -n 1s a member
of A . Since c o n t a i n s the quotients of i t s members, it
A
follows t h a t A c o n t a i n s every quotient of t h e form
9
where p and q are integers and q # 0 .
T h i s says t h a t every
r a t i o n a l number is a member of A .
In other words, Q C A .
The theorem is established.
S u b f i e l d s intermediate t o - Q and
R .
There is a v a s t
h i e r a r c h y of s u b f i e l d s between Q and R .
T h e i r study is a
large undertaking. We s h a l l c o n t e n t ourselves to see that certain
intermediate f i e l d s can be exhibited in a simple way,
Let A denote the s e t of real numbers of the form

where a and b are both r a t i o n a l . What can be said about -sum


and product of elements of A ? Given a, b, c, d r a t i o n a l , we
see t h a t

and s i n c e a + c and b + d are rational, we have

Similarly,
(a + b e ) . (c + dfl) = (ac + ad) + (ad + bc)fl,
and s i n c e ac .t 2bd and ad + bc are rational, we have

Suppose t h a t a + b@ = 0 where a and b are r a t i o n a l .


Then b = 0 , otherwise f l would be r a t i o n a l . It follows that
also a = 0 .
Therefore, member a + b f l of A (a and b -
rational) -
is equal to zero if and onlx a 0 and b = 0 .
2
.
7 - - -

2
This implies that if a + b f l # 0 , then a - 2b # 0 Other-
wise we should have

so that e i t h e r a .t = o or a + ( - b ) J Z = 0 From .
a + (-b) fl = 0 , we have a = 0 and -b = 0 and consequently
a + bfl= 0 . That is, if a
2
- 2b 2 = 0, then a + bfl= 0 ,
W e n o w have by a familiar rationalization method, f o r

This t e l l s us that the q u o t i e n t of two members of A I s also


a member of A .
It is now easy to verify t h a t A I s a s u b f i e l d of the real
number system. We leave the d e t a i l s as an exercise.

Exercises 15-7
1. Show t h a t A is a s u b f i e l d of the real number system.
2. L e t B denote the s e t of real numbers of the form a + b f l
where a and b are r a t i o n a l . Show that B is a s u b f i e l d
of t h e real number system.
+3. Show t h a t the only r e a l numbers belonging to both A and B
are rational. In p a r t i c u l a r , f l does n o t belong to A .
--
Hence, A is intermediate In t h e strict sense to & and R-- .
That is, Q is a proper part of A , and A l a a proper p a r t
of R.

References :
1. Birkhoff, G a r r e t t and Saunders MacLane, A Survey of -
Modern A l ~ e b r a(rev. e d . ) , Macmillan Company.
2. Books cited in the bibliography of Reference 1 above.
INDEX
The reference is t o the page on which the term occurs.
Abel, N. H., 849
a b s c i s s a , 120, 125
absolute value, 3 6 , 37, 131, 275, 278, 279
cf a product, 688
a b s t r a c t algebra, 844
addition, 20, 40, 55, 64, 75
addition formulas,
functions or composite angles, 605
addition of v e c t o r s , 647
additive inverse, 258, 277
additive inverae in vectors,
addition, 637
algebraic equations, 170
algebraic s t r u c t u r e s , 844, 847
analytic geometry, 119
applications,
of vectors t o physics, 660 665
Archimedes1 property, 20, 56, 64, 7 6
Argand diagram, 276, 277, 707
arithmetic means, 747
arithmetic progression, 740
a r i t h m e t i c sequence, 739
arrangements, 826
associative law f o r vectors,
addition, 636, 637
associativity, 6 , 21, 40, 64, 75
asymptotes, 345
axis of a graph, 205, 213, 215, 315
binomial product , 89
binomial square, 89
blnomial theorem, 783, 818
cancellatlon property, 7
center,
ellipse, 340
hyperbola, 351
change of base, 512
charac terls t i c , 488
c i r c l e , 333
c i r c u l a r permutations, 802
closure p r o p e r t 5, 6 , 21, 27, 40, 64, 75
coefficients,
collinear, 649
&, 252, 289

combination of the n elements taken m at a time, 806, 809


combinations, 783, 805
common difference, 139
common f a c t o r , 89
common logarithm function, 456
commutative group, 849
commutative l a w f o r , 635
a d d i t i o n , 636
addition of vectors, 635
commutative property, 6, 21, 40, 64, 75
completing the square, 217, 224
complex fraction, 97
complex numbers, 225, 251, 256, 683
addition, multiplication, s u b t r a c t i o n , 256
argument, 693
c o n s t r u c t i o n o f , 298
division, 264
graphical representation, 275
p r o p e r t f es , 253
real part, 262
standard form, 261, 691
complex numbers and vectors, 684
complex solutions, 291, 293
components, 644
x-c omponent , 646
y-component, 646
composition, 183, 184
computation with logarithms, 505
conlc, 326
direc trix, 328
eccentricity, 328
focus, 328
conjugate , 282
c o n s i s t e n t systems, 365
c o n s t a n t function, 167, 200
convergent sequence , 756
converse, 3, 29, 192
coordinates, 125
coordinate axes, 119
coordinate geometry, 119
c o o r d i n a t e sys tern, 1x9, 409
c o s i n e f u n c t i o n , 561, 566, 569
counting problems, 783
cube r o o t s , 711
decimal, 66
degree of a polynomial, 289
deMoivre Theorem, 695
density, 57, 64, 76
dependent sys tems , 365
dichotomy, 6 , 20, 40, 63, 74, 75
difference of cubes, 89
difference of squares, 89
g;l;nts,
d i r e c t e d line se
addition,
629, 630
equivalent, 630
discriminant, 227, 241, 251, 272
distance, 629
d i s t a n c e formula, 124, 127
three-dimensions , 411
e
distributive law for multi l l c a t i o n of a vector by a scalar,
d i s t r i b u t i v i t y , 21, 41, 6 , 76
d i v e r g e n t sequence, 757
638

d i v i s i o n , 64, 75
domain, 165, 166, 170, 173, 176, 182, 183, 184
dot product, 656
double root, 231
dumrny variable, 734
e, base of natural logarithm, 511
element,
identity , 851
inverse, 852
elimination, 266
ellipse, 330
center, 335
major axis, 335
rnlnor axfs, 335
v e r t i c e s , 335
equality , 5
equations,
trigonometric, 612
equation xn = a, 77
equation y = ax2 + bx + c , 222, 243
equilibrium, 667
equivalent angles, 552
e q u i v a l e n t equations, 222, 226, 233,
inequalities, 370, 422
equivalent paths, 548
equivalent sys terns, 371, 448
exponential f u n c t i o n s , 520
exponents, laws of', 529
external o p e r a t i o n , 845
extraneous r o o t s , 233, 234
f a c t o r i n g , solution by, 228
f a c t o r s , 86-89
field, 858
f o c a l chord, 359
f u n c t i o n , 165, 166, 182, 183, 844
composition 183
inverse, 183, 523
linear, 189, 191, 193, 196, 201, 220
notation, 168
y = ax2 + bx + c, 217
2
y = a ( x - k) , 211
y = a ( x - k ) 2 + p , 214
2
y = ax + C , 209
2
y = ax , 206
2
y = x , 204
fundamental theorem o f algebra, 290
general t e n , 734
geometric progressions 749
geometric sequence, - 748
geometric series, 748
golden rectangle, 245
graph, 140, 141, 173, 420
graphs of the trigonometric functions, 590
group, 848
harmonic mean, 780
h o r i z o n t a l line, 197
hyperbola, 330, 342
asymptotes, 345
center, 351
conjugate axis, 343
transverse a x i s , 343
identities, 612
identity,
a d d l t i v e , 24
multiplicative 6
Identity element, 651
identity function, 167
imaginary, 254
part, 262
unit, 253
i n c o n s i s t e n t s stems, 364
inequa1ity , 1 5
graph of, 140, 145
i n f i n i t e decimal, 72, 774
i n f i n i t e geometric series, 774
initial point, 630
inner product , 655
i n t e g e r s , 1, 24, 42
intercept form 313
intercepts, 145
internal operation 845
interpolation, 498-501, 579
invera e ,
a d d l t i v e , 25
multiplicative, 48
of a l i n e a r function, 190, 191
inverse element, 852
inverse functions, 183, 184
inverse variation, 348
irrational, 73
decimal, 73, 74
Isomorphic, 680, 685
law of cosines, 594, 658
l a w of sines, 598
limit of a sequence, 754
limits - theorems on, 758
linear combination, 374, 449
linear equation, 310
logarithm function, 455
graph of, 456
p r o p e r t i e s of, 474
logarithm function with base a , 511
logarithms of n! , 813
major axis, 334
mantissa, 488
mathematical s y s tern, 678
m a x i m u m p o i n t , - 210-
mid-point formula, 127, 128
m i n i m u m p o i n t , 209
minor axis, ,334
m u l t i p l i c a t i o n , 20, 40, 55, 64, 75
multiplication by scalars, 648
multiplication formulas,
identities for, 700
multiplicative inverse, 48, 264, 266
natural logarithm function, 456
n a t u r a l numbers, 1, 4, 22, 33
n f a c t o r i a l , 799
non-abelian group, 854, 857
nth r o o t s o f u n l t y , 718
nth t e r m of a geometric sequence, 749
nth term of an arithmetic sequence 740
one-to-one correspondence, 120, 169, 680, 783
ordered m-tuples, 788, 792
ordered p a i r s , 789
ordered p a r t i t i o n of a s e t , 831
ordered triple, 791
o r d e r property, 13, 31, 53
ordinate, 120, 125
origin, 119
parabola, 205, 221, 315
axis, 315
direc trix, 315
focus, 315
latus rectum, 324
standard form, 318
vertex, 315
p a r a l l e l , 133, 134
p a r a l l e l rays, 629
partial sums, 765
partitions, 826
Pascalis Theorem 818
p e m t a t i a n s , 783, 79 5
plane, equation o f , 415
point-s lope f o m , 305
p o l a r form, 683, 692
polynomials, 86, 87, 88, 289
prescribed values, 196, 220
product ,
dot, 656
mer, 655
projectLon, 120
properties of groups, 851
Pythagorean Theorem, 125
quadrant, 122
quadratic equation, 203, 243, 252
quadratic equatLons wlth complex c o e f f i c i e n t s , 707
quadratic formula, 228
quadratic i n e q u a l i t i e s , 238
q u o t f e n t , 43
radian measure, 556, 559
range, 165, 166, 173, 176, 183, 184
r a t t o n a l expressions, 95
numbers, 1
number system, 43, 65
solution, 79
real number system, 1
r e r l e c tion, 146, 283
r e f l e x i v i t y , 6, 20, 40, 63, 75
repeating decimal, 67, 69
resultant, 665
Riemann, Bernhard, 727
Riemann surface, 727
r o o t , 224; 229
roots of order n, 710
scalars, 644
s e l e c t i o n s with repetition, 834
sequences and s e r i e s , 731
arithmetic sequence, 739
convergent sequence, 756
divergent sequence, 757
f i n i t e sequence, 731
f i n i t e s e r i e s , 732
geometric sequence, 748
geometric series, 748
I n f i n i t e geometric series, 774
i n f i n l t e sequence, 731
inf inl te s e r i e s , 732
nth t a n of a geometric sequence, 749
nth term of an arithmetic sequence, 740
serles for,
cos x, 771
ex, 771
sin x, 771
sets s a t i s f y i n g geometric conditions, 156
sigma notation, "x", 745
signed angles, 550, 551
sine f u n c t i o n , 561, 566, 569
s l o p e , 130, 132, 192, 303
of parallel l i n e s , 1 3 3
of perpendicular l i n e s , 134
s l o p e - i n t e r c e p t form, 306
solution of an equation, 224
s o l u t i o n set,
of equation or i n e q u a l i t y , 361
of a system, 363
o r three f i r e t degree e q u a t i o n s , 437, 422, 426
speclal angles,
trigonom tric functions of, 573, 574
square roots, 701
standard form, 262
standard form f o r log x, 488
s tandard p o s i t i o n ,
the decimal p o i n t , 496
straight l i n e , 303
structure, 1, 2
s u b f i e l d , 863
subfielas intermediate to Q and R, 865
subsets of a f i n i t e s e t , 806
subtraction, 35
sum of a f i n i t e geometric aeries, 751
sum of an i n f i n i t e serfes , 764, 766
sum o l arithmetic series, 743
sum of cubes, 89
summation notation, 734
sum sn of a f l n i t e aeries, 735
symmetric, 283
symmetry, 6 , 20, 40, 64, 75 146-148
system of equations, 361, 381, 389, 398, 422
table o f common logarithms, 464, 502 503
t a b l e of trigonometric f u n c t i o n s , 578, 580
tangent function, 561, 566
terminal point, 630
terminating decimal, 66, 67
Theorem of deMolvre, 695
trace, 420
t r a n s i t i v i t y , 6, 13, 20, 40, 55, 64, 75
triangle inequality, 62, 280
t rlangulat i o n ,
method of s o l u t i o n , 432
trichotomy, 13, 16, 55, 74
trigonometr2c functions, 561, 566, 583
two-dimensional v e c t o r space, 682
variation,
dfrect, 308
vectors in three dimensions, 661
v e c t o r space, 681
velocity, 674
vertex, 205, 213, 220
v e r t i c a l line, 133, 176, 197
well order property, 20, 40
work, 672
zero element for vector,
addition, 636
zero factorial, 799
zero of a polynomlal, 291
zero v e c t o r , 648

You might also like